Advanced Med Surg Final Exam

Réussis tes devoirs et examens dès maintenant avec Quizwiz!

When caring for a patient with pulmonary hypertension, which parameter is most appropriate for the nurse to monitor to evaluate the effectiveness of the treatment?

Pulmonary vascular resistance (PVR)

A 37-year-old patient is being admitted with a diagnosis of Cushing syndrome. Which findings will the nurse expect during the assessment?

Purplish streaks on the abdomen

The urgent care center protocol for tick bites includes the following actions. Which action will the nurse take first when caring for a patient with a tick bite?

Use tweezers to remove any remaining ticks

The nurse notes that a patients cardiac monitor shows that every other beat is earlier than expected, has no visible P wave, and has a QRS complex that is wide and bizarre in shape. How will the nurse document the rhythm?

Ventricular bigeminy

A patient who is orally intubated and receiving mechanical ventilation is anxious and is fighting the ventilator. Which action should the nurse take next?

Verbally coach the patient to breathe with the ventilator

During the change of shift report a nurse is told that a patient has an occluded left posterior cerebral artery. The nurse will anticipate that the patient may have

Visual deficits

A nurse is assessing a patient who is receiving a nitroprusside (Nipride) infusion to treat cardiogenic shock. Which finding indicates that the medication is effective?

Warm, pink, and dry skin

Following assessment of a patient with pneumonia, the nurse identifies a nursing diagnosis of ineffective airway clearance. Which assessment data best supports this diagnosis?

Weak, nonproductive cough effort

When does a sub acute subdural hematoma occur?

Within 48 hours to 2 weeks

An older patient is receiving standard multidrug therapy for tuberculosis (TB). The nurse should notify the health care provider if the patient exhibits which finding?

Yellow-tinged skin

The female pt is having whole brain radiation for brain metastasis. She is concerned about how she will look when she loses her hair. What is the best response by the nurse to this pt.

You can get a wig now to match your hair so you will not look different

A patient scheduled for a total laryngectomy and radical neck dissection for cancer of the larynx asks the nurse, Will I be able to talk normally after surgery? What is the best response by the nurse?

You will have a permanent opening into your neck, and you will need to have rehabilitation for some type of voice restoration.

A 50-year-old man complains of recurring headaches. He describes these as sharp, stabbing, and located around his left eye. He also reports that his left eye seems to swell and get teary when these headaches occur. Based on this history, you suspect that he has a. cluster headaches. b. tension headaches. c. migraine headaches. d. medication overuse headaches.

a Rationale: Cluster headaches involve repeated headaches that can occur for weeks to months, followed by periods of remission. The pain of cluster headache is sharp and stabbing; the intense pain lasts a few minutes to 3 hours. Cluster headaches can occur every other day and as often as eight times a day. The clusters occur with regularity, usually occurring at the same time each day and during the same seasons of the year. Typically, a cluster lasts 2 weeks to 3 months, and the patient then goes into remission for months to years. The pain usually is located around the eye and radiates to the temple, forehead, cheek, nose, or gums. Other manifestations may include swelling around the eye, lacrimation (tearing), facial flushing or pallor, nasal congestion, and constriction of the pupil. During the headache, the patient is often agitated and restless, unable to sit still or relax.

A nurse is discussing hemodialysis with a newly licensed nurse. The nurse should identify that hemodialysis is contraindicated for which of the following clients?

a client who cannot receive anticoagulants

During the assessment of a 25-year-old patient with infective endocarditis (IE), the nurse would expect to find

a new regurgitant murmur.

Four victims of an automobile crash are brought by ambulance to the emergency department. The triage nurse determines that the victim who has the highest priority for treatment is the one with

a sucking chest wound

Social effects of a chronic neurologic disease include (select all that apply) a. divorce. b. job loss. c. depression. d. role changes. e. loss of self-esteem.

a, b, c, d, e Rationale: Social problems related to chronic neurologic disease may include changes in roles and relationships (e.g., divorce, job loss, role changes); other psychologic problems (e.g., depression, loss of self-esteem) also may have social effects.

Which nursing dx is most appropriate for a patient experiencing myelosupression secondary to chemo for cancer tx?

Risk for infection

A 20 year old patient arrives in the ED several hours after taking 25-30 acetaminophen (Tylenol) tablets. Which action will the nurse plan to take?

Give N-acetylcysteine (Mucomyst)

The nurse is monitoring a patient for increased ICP following a head injury. What are manifestations of increased ICP (select all that apply)?

-Fever -Oriented to name only -Right pupil dilated greater than left pupil -Decorticate posturing to painful stimulus

The patient with diabetes has a blood glucose level of 248 mg/dL. Which manifestations in the patient would the nurse understand as being related to this blood glucose level? (select all that apply)

-Headache -Abdominal cramps -Increase in urination -Weakness and fatigue

Which question during the assessment of a diabetic patient will help the nurse identify autonomic neuropathy?

"Do you feel bloated after eating?"

A 45-year-old male patient with suspected acromegaly is seen at the clinic. To assist in making the diagnosis, which question should the nurse ask?

"Do you have to wear larger shoes now?"

The nurse is assessing a 22-year-old patient experiencing the onset of symptoms of type 1 diabetes. Which question is most appropriate for the nurse to ask?

"Have you lost weight lately?"

The nurse determines a need for additional instruction when the patient with newly diagnosed type 1 diabetes says which of the following?

"I can choose any foods, as long as I use enough insulin to cover the calories."

A 38-year-old male patient is admitted to the hospital in Addisonian crisis. Which patient statement supports a nursing diagnosis of ineffective self-health management related to lack of knowledge about management of Addison's disease?

"I had the stomach flu earlier this week, so I couldn't take the hydrocortisone."

Which statement by the patient indicates a need for additional instruction in administering insulin?

"I need to rotate injection sites among my arms, legs, and abdomen each day."

The nurse determines that additional instruction is needed for a 60-year-old patient with chronic syndrome of inappropriate antidiuretic hormone (SIADH) when the patient says which of the following?

"I need to shop for foods low in sodium and avoid adding salt to food."

After the nurse has finished teaching a patient who has a new prescription for exenatide (Byetta), which patient statement indicates that the teaching has been effective?

"I should take my daily aspirin at least an hour before the Byetta."

The nurse has been teaching a patient with type 2 diabetes about managing blood glucose levels and taking glipizide (Glucotrol). Which patient statement indicates a need for additional teaching?

"My diabetes won't cause complications because I don't need insulin."

A patient who is recovering from an acute myocardial infarction (AMI) asks the nurse about when sexual intercourse can be resumed. Which response by the nurse is BEST?

"Sexual activity uses about as much energy as climbing two flights of stairs."

A 29-year-old woman with systemic lupus erythematosus has been prescribed 2 weeks of high-dose prednisone therapy. Which information about the prednisone is most important for the nurse to include?

"The prednisone dose should be decreased gradually rather than stopped suddenly."

Which question will the nurse in the endocrine clinic ask to help determine a patient's risk factors for goiter?

"What medications are you taking?"

The patient has been diagnosed with a cerebral concussion. What should the nurse expect to see in this patient?

-Headache -retrograde amnesia -transient reduction in LOC

Which instructions by the nurse are beneficial to a patient with acute pyelonephritis? (select all that apply)

-"Drink at least eight glasses of fluid every day." -"Take the full course of antibiotics to ensure that the bacteria are eradicated."

The nurse is reviewing the medical records for five patients who are scheduled for their yearly physical examinations in September. Which patients should receive the inactivated influenza vaccination (select all that apply)?

-A 76-year-old nursing home resident -A 36-year-old female patient who is pregnant -A 30-year-old patient who takes corticosteroids for rheumatoid arthritis

Which conditions characterize critical limb ischemia? (select all that apply)

-Arterial leg ulcers. -Gangrene of the leg.

When preparing to cool a patient who is to begin therapeutic hypothermia, which intervention will the nurse plan to do?

-Assist with edotracheal intubation -Insert an indwelling urinary catheter -Begin continuous cardiac monitoring

Which information will be included when the nurse is teaching self-management to a patient who is receiving peritoneal dialysis

-Avoid commercial salt substitutes. -Take phosphate-binders with each meal. -Choose high-protein foods for most meals.

A nurse is planning preoperative care for a client who has Phenochromocytoma. Which of the following interventions should the nurse anticipate as being the priority?

-Avoid palpating the abdomen (greatest risk to client is injury from hypertensive crisis. Palpation can cause a sudden release of catecholamines, causing hypertensive crisis).

A nurse is reviewing the laboratory reports of a client who has acute kidney injury (AKI). Which of the following findings should the nurse expect? (Select all that apply.)

-BUN 30 -Urine output of 40 mL in past 3 hrs -Hematocrit 30%

To monitor for complications in a patient with type 2 diabetes, which tests will the nurse in the diabetic clinic schedule at least annually (select all that apply)?

-Blood Pressure -Serum Creatinine -Urine for microalbuminuria -Monofilament testing of the foot

A nurse is preparing response for botulism as a bioterrorism agent. The nurse should prepare the protocol based on which of the following information? (select all that apply)

-Botulism can produce paralysis within 12 to 72 hours following exposure -vomiting and diarrhea are expected findings following exposure

The clinic nurse is teaching a patient with acute sinusitis. Which interventions should the nurse plan to include in the teaching session (select all that apply)?

-Decongestants can be used to relieve swelling. -Taking a hot shower will increase sinus drainage and decrease pain. -Saline nasal spray can be made at home and used to wash out secretions. -You will be more comfortable if you keep your head in an upright position.

The nurse recognizes the presence of Cushing's triad in the patient with which vital sign changes?

-Decreased pulse (bradycardia) -irregular respiration -widened pulse pressure

What is an appropriate respiratory goal with increased ICP?

-Deep breathing -promote CO2 elimination

The nurse is caring for a patient admitted to the hospital with a head injury who requires frequent neurologic assessment. Which components are assessed using the Glasgow Coma Scale (GCS) (select all that apply.)?

-Eye opening -Best verbal response -Best motor response

A nurse is caring for a client who has ingested a toxic agent. Which of the following actions should the nurse plan to take? (Select all that apply.) A. Induce vomiting. B. Instill activated charcoal. C. Perform a gastric lavage with aspiration. D. Administer syrup of ipecac. E. Infuse IV fluids.

-Instill activated charcoal -Perform gastric lavage with aspiration -Infuse IV fluids

Which components are able to change to adapt to small increases in intracranial pressure (ICP) (select all that apply)?

-Intravascular blood -brain tissue -cerebrospinal fluid

A patient has been diagnosed with urinary tract calculi that are high in uric acid. Which foods will the nurse teach the patient to avoid

-Liver -Chicken

A dehydrated patient is receiving a hypertonic solution. Which assessments must be done to avoid adverse risks associated with these solutions

-Lung sounds -Blood pressure -Serum sodium level

The patient and his family are upset that the pt is going through procedures to dx cancer. What nursing actions should the nurse use first to facilitate their coping with this situation (select all that apply)

-Maintain hope -Exhibit a caring attitude -Be available to listen to fears and concerns

A nurse is teaching a client who has an autoimmune disease about the adverse effects of long-term corticosteroid therapy. Which of the following effects should the nurse include? (Select All) A Osteoporosis B Moon-shaped face C Increased risk of infection D Hearing loss E Weight loss

-Osteoporosis -Moon-shaped face -Increased risk of infection

Most important nursing intervention for a client with a Pulmonary Embolism (PE) is what?

-Oxygen therapy -anticoagulation -opioids -continuous ECG monitoring

What factors decrease cerebral blood flow (select all that apply)?

-PaCO2 of 30 mm Hg -Decreased mean arterial pressure (MAP) -Increased ICP

The ED nurse is initiating therapeutic hypothermia in a patient who has been resuscitated after a cardiac arrest. Which actions in the hypothermia protocol can be delegated to an experienced licensed practical/vocational nurse (LPN/LVN)?

-Place cooling blanket above and below patient -Give acetaminophen (Tylenol) 650 mg per nasogastric tube -Insert rectal temperature probe and attach to cooling blanket control panel

A pt with suspected neurogenic shock after a diving accident has arrived in the ED. A cervical collar is in place. Which action should the nurse take?

-Prepare to administer atropine IV -Obtain baseline body temp -Provide high-flow oxygen (100%) by non-rebreather mask -Prepare for emergent intubation and mechanical ventilation

A patient with suspected neurogenic shock after a diving accident has arrived in the emergency department. A cervical collar is in place. Which actions should the nurse take (select all that apply)?

-Prepare to administer atropine IV. -Obtain baseline body temperature. -Provide high-flow oxygen (100%) by non-rebreather mask. -Prepare for emergent intubation and mechanical ventilation.

Patient having increased ICP, what would be an appropriate respiratory goal? (select all that apply)

-Promote CO2 elimination -Deep breathing

Which actions should the nurse initiate to reduce the risk for ventilator-associated pneumonia (VAP) (select all that apply)?

-Provide a sedation holiday daily. -Elevate the head of the bed to at least 30. -Give prescribed pantoprazole (Protonix). -Provide oral care with chlorhexidine (0.12%) solution daily.

While performing patient teaching regarding hypercalcemia, which statements are appropriate

-Renal calculi may occur as a complication of hypercalcemia. -Weight-bearing exercises can help keep calcium in the bones. -The patient should increase daily fluid intake to 3000 to 4000 mL.

The surgery area calls the transfer report for a 68 year old, postmenopausal, female patient who smokes and takes hormone therapy. She is returning to the floor after a lengthy hip replacement surgery. Which factors present in this patient increase her risk for developing venous thromboembolism (VTE) related to Virchow's triad? (select all that apply)

-Smoking -IV therapy -Estrogen therapy -Orthopedic surgery

A patient is admitted to the emergency department complaining of sudden onset shortness of breath and is diagnosed with a possible pulmonary embolus. How should the nurse prepare the patient for diagnostic testing to confirm the diagnosis?

-Start an IV so contrast media can be given -be aware of allergy to shell fish or iodine r/t contrast media

You are caring for a patient with an advanced stage of breast cancer. The cancer has metastasized. You enter the room and find the patient struggling to breath. You note that this patient's jugular veins are distended. What would you suspect is happening with this patient?

-Superior vena cava syndrome (SVCS)

Which medications are indicated for arterial thrombus if less than 14 days? (Select all that apply).

-TPA (Alteplase) -Unfractionated heparin -Urokinase

a 19 year old is brought to the ED with multiple lacerations an tissue avulsion of the left hand. When asked about tetanus immunization, the patient denies having any previous vaccinations. The nurse will anticipate giving...

-Tetanus immunoglobulin (TIG) -Tetanus-diphtheria toxoid Pertussis vaccine

A patient with respiratory failure has arterial pressure based cardiac output (APCO) monitoring and is receiving mechanical ventilation with peak end-expiratory pressure (PEEP) of 12 cm H2O. Which information indicates that a change in the ventilator settings may be required?

-The arterial pressure is 90/46.

You are caring for a patient admitted with diabetes mellitus, malnutrition, and a massive GI bleed. In analyzing the morning lab results, the nurse understands that a potassium level of 5.5 mEq/L could be caused by which factors in this patient

-The potassium level may be increased if the patient has nephropathy. -There may be excess potassium being released into the blood as a result of massive blood transfusion. -The potassium level may be increased because of dehydration that accompanies high blood glucose levels.

What are the manifestations of Acute Coronary Syndrome? (Select all that apply)

-Unstable angina -STEMI -Non-STEMI

Which preventive actions by the nurse will help limit the development of systemic inflammatory response syndrome (SIRS) in patients admitted to the hospital (select all that apply)?

-Use aseptic technique when caring for invasive lines or devices. -Ambulate postoperative patients as soon as possible after surgery. -Remove indwelling urinary catheters as soon as possible after surgery. -Administer prescribed antibiotics within 1 hour for patients with possible sepsis.

While caring for a patient with metastatic bone cancer, which clinical manifestations would alert the nurse to the possibility of hypercalcemia in this patient

-Weakness -Depressed reflexes

Your assessing a patient with Guilian Barre Syndrome what do you expect?

-Weakness in legs -Leg and feet numbness

Which factors will the nurse consider when calculating the CURB-65 score for a patient with pneumonia (select all that apply)? a. Age b. Blood pressure c. Respiratory rate d. Oxygen saturation e. Presence of confusion f. Blood urea nitrogen (BUN) level

-a. Age -b. Blood pressure -c. Respiratory rate -e. Presence of confusion -f. Blood urea nitrogen (BUN) level

The nurse is caring for a patient who has an intraortic balloon pump (IABP) following a massive heart attack. When assessing the patient, the nurse notices blood backing up into the IABP catheter. In which order should the nurse take the following actions? (Put a comma and a space between each answer choice [A, B, C, D.) a. Ensure that the IABP console has turned off. b. Assess the patients vital signs and orientation. c. Obtain supplies for insertion of a new IABP catheter. d. Notify the health care provider of the IABP malfunction.

-a. Ensure that the IABP console has turned off. -b. Assess the patients vital signs and orientation. -d. Notify the health care provider of the IABP malfunction. -c. Obtain supplies for insertion of a new IABP catheter.

The nurse notes new onset confusion in an older patient who is normally alert and oriented. In which order should the nurse take the following actions? (Put a comma and a space between each answer choice [A, B, C, D].) a. Obtain the oxygen saturation. b. Check the patients pulse rate. c. Document the change in status. d. Notify the health care provider.

-a. Obtain the oxygen saturation. -Check the patients pulse rate. -d. Notify the health care provider. -c. Document the change in status.

When preparing to defibrillate a patient. In which order will the nurse perform the following steps?(Put a comma and a space between each answer choice [A, B, C, D, E].) a. Turn the defibrillator on. b. Deliver the electrical charge. c. Select the appropriate energy level. d. Place the paddles on the patients chest. e. Check the location of other staff and call out all clear.

-a. Turn the defibrillator on. -c. Select the appropriate energy level. -d. Place the paddles on the patients chest. -e. Check the location of other staff and call out all clear. -b. Deliver the electrical charge.

The nurse assumes care of a patient who just returned from surgery for a total laryngectomy and radical neck dissection and notes the following problems. In which order should the nurse address the problems? (Put a comma and a space between each answer choice [A, B, C, D].)

-a.The patient is in a side-lying position with the head of the bed flat. -b. The patient is coughing blood-tinged secretions from the tracheostomy -d. The wound drain in the neck incision contains 200 mL of bloody drainage. -c. The nasogastric (NG) tube is disconnected from suction and clamped off.

A nurse in the emergency department is preparing to care for a client who received multiple injuries during a fight. The nurse should plan to base his primary survey on which of the following? (Select all that apply.) airway cervical spine disability exposure urinary output

-airway -cervical spine -disability -exposure

What are the clinical manifestations of acute pyelonephritis?

-chills and flank pain

What to include in a disaster preparedness kit? (select all that apply)

-clean clothing -personal ID -matches -prescription medications

What is the treatment for a patient with traumatic brain injury (TBI)?

-craniotomy -rest -diuretics -rehab

When assisting with oral intubation of a patient who is having respiratory distress, in which order will the nurse take these actions? (Put a comma and a space between each answer choice [A, B, C, D, E].) a. Obtain a portable chest-x-ray. b. Position the patient in the supine position. c. Inflate the cuff of the endotracheal tube after insertion. d. Attach an end-tidal CO2 detector to the endotracheal tube. e. Oxygenate the patient with a bag-valve-mask device for several minutes.

-e. Oxygenate the patient with a bag-valve-mask device for several minutes. -b. Position the patient in the supine position. -c. Inflate the cuff of the endotracheal tube after insertion. -d. Attach an end-tidal CO2 detector to the endotracheal tube. -a. Obtain a portable chest-x-ray.

A nurse in the emergency department is implementing a plan of care for a conscious client who has a suspected cervical cord injury. Which of the following immediate interventions should the nurse implement? (Select all that apply.)

-hypotension -absence of bowel sounds -weakened gag reflex

What strategies do you add to your plan of care with a patient with Guillian Barre Syndrome ?

-positive feedback -emotional support -comfort -relaxation

a nurse is reinforcing teaching with a client who has diabetes. Which of the following should the nurse include in the teaching? Select all that apply

-reduce cholesterol and saturated fat intake -increase physical activity and daily exercise -enroll in a smoking cessation program -maintain optimal blood pressure to prevent kidney damage

A nurse in the emergency department is caring for a client who fell through the ice on a pond and is unresponsive and breathing slowly. Which of the following actions should the nurse take? (Select all that apply.) A. Remove wet clothing. B. Maintain normal room temperature. C. Apply warm blankets. D. Apply a heat lamp. E. Infuse warmed IV fluids.

-remove wet clothing -Apply warm blankets -Apply a heat lamp -Infuse warmed IV fluids

A nurse is planning a staff education session regarding biological weapons of mass destruction. Which of the following should he include in the session? (select all that apply)

-small pox -anthrax -botulism

Which characteristics describe the anticoagulant warfarin. (select all that apply)

-taken orally -Vitamin K is the antidote. -Dosage monitored using international normalized ratio (INR).

a nurse is performing teaching with a client who has newly diagnosed type 2 diabetes. The nurse should recognize that the client understands the teaching when he identifies which of the following manifestations of hypoglycemia? Select all that apply

-vertigo -tachycardia -moist, clammy skin

After receiving change-of-shift report about the following four patients, which patient should the nurse assess first?

A 70-year-old female taking levothyroxine (Synthroid) who has an irregular pulse of 134

A patient is to receive methylprednisolone (Solu-Medrol) 100 mg. The label on the medication states: methylprednisolone 125 mg in 2 mL. How many milliliters will the nurse administer?

1.6

10. Which type of seizure is most likely to cause death for the patient? a. Subclinical seizures b. Myoclonic seizures c. Psychogenic seizures d. Tonic-clonic status epilepticus

10. d. Tonic-clonic status epilepticus is most dangerous because the continuous seizing can cause respiratory insufficiency, hypoxemia, cardiac dysrhythmia, hyperthermia, and systemic acidosis, which can all be fatal. Subclinical seizures may occur in a patient who is sedated, so there is no physical movement. Myoclonic seizures may occur in clusters and have a sudden, excessive jerk of the body that may hurl the person to the ground. Psychogenic seizures are psychiatric in origin and diagnosed with videoelectroencephalography (EEG) monitoring. They occur in patients with a history of emotional abuse or a specific traumatic episode.

A patient receives aspart (NovoLog) insulin at 8:00 AM. Which time will it be most important for the nurse to monitor for symptoms of hypoglycemia?

10:00 AM

A 46-year-old patient with a head injury opens the eyes to verbal stimulation, curses when stimulated, and does not respond to a verbal command to move but attempts to push away a painful stimulus. The nurse records the patient's Glasgow Coma Scale score as?

11

11. A patient admitted to the hospital following a generalized tonic-clonic seizure asks the nurse what caused the seizure. What is the best response by the nurse? a. "So many factors can cause epilepsy that it is impossible to say what caused your seizure." b. "Epilepsy is an inherited disorder. Does anyone else in your family have a seizure disorder?" c. "In seizures, some type of trigger causes sudden, abnormal bursts of electrical brain activity." d. "Scar tissue in the brain alters the chemical balance, creating uncontrolled electrical discharges."

11. c. A seizure is a paroxysmal, uncontrolled discharge of neurons in the brain, which interrupts normal function, but the factor that causes the abnormal firing is not clear. Seizures may be precipitated by many factors and although scar tissue may make the brain neurons more likely to fire, it is not the usual cause of seizures. Epilepsy is established only by a pattern of spontaneous, recurring seizures.

12. A patient with a seizure disorder is being evaluated for surgical treatment of the seizures. The nurse recognizes that what is one of the requirements for surgical treatment? a. Identification of scar tissue that is able to be removed b. An adequate trial of drug therapy that had unsatisfactory results c. Development of toxic syndromes from long-term use of antiseizure drugs d. The presence of symptoms of cerebral degeneration from repeated seizures

12. b. Most patients with seizure disorders maintain seizure control with medications but if surgery is considered, three requirements must be met: the diagnosis of epilepsy must be confirmed, there must have been an adequate trial with drug therapy without satisfactory results, and the electroclinical syndrome must be defined. The focal point must be localized but the presence of scar tissue is not required.

13. The nurse teaches the patient taking antiseizure drugs that this method is most commonly used to measure compliance and monitor for toxicity. a. A daily seizure log b. Urine testing for drug levels c. Blood testing for drug levels d. Monthly electroencephalography (EEG)

13. c. Serum levels of antiseizure drugs are monitored regularly to maintain therapeutic levels of the drug, above which patients are likely to experience toxic effects and below which seizures are likely to occur. Many newer drugs do not require drug level monitoring because of large therapeutic ranges. A daily seizure log and urine testing for drug levels will not measure compliance or monitor for toxicity. EEGs have limited value in diagnosis of seizures and even less value in monitoring seizure control.

14. Priority Decision: When teaching a patient with a seizure disorder about the medication regimen, what is it most important for the nurse to emphasize? a. The patient should increase the dosage of the medication if stress is increased. b. Most over-the-counter and prescription drugs are safe to take with antiseizure drugs. c. Stopping the medication abruptly may increase the intensity and frequency of seizures. d. If gingival hypertrophy occurs, the drug should be stopped and the health care provider notified.

14. c. If antiseizure drugs are discontinued abruptly, seizures can be precipitated. Missed doses should be made up if the omission is remembered within 24 hours and patients should not adjust medications without professional guidance because this also can increase seizure frequency and may cause status epilepticus. Antiseizure drugs have numerous interactions with other drugs and the use of other medications should be evaluated by health professionals. If side effects occur, the physician should be notified and drug regimens evaluated.

Following a generalized tonic-clonic seizure, the patient is tired and sleepy. What care should the nurse provide? a. Suction the patient before allowing him to rest. b. Allow the patient to sleep as long as he feels sleepy. c. Stimulate the patient to increase his level of consciousness. d. Check the patient's level of consciousness every 15 minutes for an hour.

16. b. In the postictal phase of generalized tonic-clonic seizures, patients are usually very tired and may sleep for several hours and the nurse should allow the patient to sleep as long as necessary. Suctioning is performed only if needed and decreased level of consciousness is not a problem postictally unless a head injury has occurred during the seizure.

17. During the diagnosis and long-term management of a seizure disorder, what should the nurse recognize as one of the major needs of the patient? a. Managing the complicated drug regimen of seizure control b. Coping with the effects of negative social attitudes toward epilepsy c. Adjusting to the very restricted lifestyle required by a diagnosis of epilepsy d. Learning to minimize the effect of the condition in order to obtain employment

17. b. One of the most common complications of a seizure disorder is the effect it has on the patient's lifestyle. This is because of the social stigma attached to seizures, which causes patients to hide their diagnosis and to prefer not to be identified as having epilepsy. Medication regimens usually require only once- or twice-daily dosing and the major restrictions of lifestyle usually involve driving and high-risk environments. Job discrimination against the handicapped is prevented by federal and state laws and patients only need to identify their disease in case of medical emergencies.

18. A patient at the clinic for a routine health examination mentions that she is exhausted because her legs bother her so much at night that she cannot sleep. The nurse questions the patient further about her leg symptoms with what knowledge about restless legs syndrome? a. The condition can be readily diagnosed with EMG. b. Other more serious nervous system dysfunctions may be present. c. Dopaminergic agents are often effective in managing the symptoms. d. Symptoms can be controlled by vigorous exercise of the legs during the day.

18. c. Restless legs syndrome that is not related to other pathologic processes, such as diabetes mellitus or rheumatic disorders, may be caused by a dysfunction in the basal ganglia circuits that use the neurotransmitter dopamine, which controls movements. Dopamine precursors and dopamine agonists, such as those used for parkinsonism, are effective in managing sensory and motor symptoms. Polysomnography studies during sleep are the only tests that have diagnostic value and although exercise should be encouraged, excessive leg exercise does not have an effect on the symptoms.

19. Which chronic neurologic disorder involves a deficiency of the neurotransmitters acetylcholine and γ aminobutyric acid (GABA) in the basal ganglia and extrapyramidal system? a. Myasthenia gravis b. Parkinson's disease c. Huntington's disease d. Amyotrophic lateral sclerosis (ALS)

19. c. Huntington's disease (HD) involves deficiency of acetylcholine and γ-aminobutyric acid (GABA) in the basal ganglia and extrapyramidal system that causes the opposite symptoms of parkinsonism. Myasthenia gravis involves autoimmune antibody destruction of cholinergic receptors at the neuromuscular junction. Amyotrophic lateral sclerosis (ALS) involves degeneration of motor neurons in the brainstem and spinal cord.

The following four patients arrive in the emergency department (ED) after a motor vehicle collision. Which patient has the highest priority?

A 74-year-old with palpitations and chest pain

A 44-year-old patient is unable to void after having an open loop resection and fulguration of the bladder. Which nursing action should be implemented first?

Assist the patient to soak in a 15-minute sitz bath.

A patient with bacterial pneumonia has rhonchi and thick sputum. What is the nurses most appropriate action to promote airway clearance?

Assist the patient to splint the chest when coughing.

A patient who is having an acute exacerbation of multiple sclerosis has a prescription for methylprednisolone (Solu-Medrol) 160 mg IV. The label on the vial reads: methylprednisolone 125 mg in 2 mL. How many mL will the nurse administer?

2.56. With a concentration of 125 mg/2 mL, the nurse will need to administer 2.56 mL to obtain 160 mg of methylprednisolone.

When planning the care of a patient with dehydration, what urine output would the nurse instruct the unlicensed assistive personnel to report?

20 mL for 2 consecutive hours

20. A 38-year-old woman has newly diagnosed multiple sclerosis (MS) and asks the nurse what is going to happen to her. What is the best response by the nurse? a. "You will have either periods of attacks and remissions or progression of nerve damage over time." b. "You need to plan for a continuous loss of movement, sensory functions, and mental capabilities." c. "You will most likely have a steady course of chronic progressive nerve damage that will change your personality." d. "It is common for people with MS to have an acute attack of weakness and then not to have any other symptoms for years."

20. a. Most patients with multiple sclerosis (MS) have remissions and exacerbations of neurologic dysfunction or a relapsing-remitting initial course followed by progression with or without occasional relapses, minor remissions, and plateaus that progressively cause loss of motor, sensory, and cerebellar functions. Intellectual function generally remains intact but patients may experience anger, depression, or euphoria. A few people have chronic progressive deterioration and some may experience only occasional and mild symptoms for several years after onset.

When planning care for stable adult patients, the oral intake that is adequate to meet daily fluid needs is

2000 to 3000 mL.

21. During assessment of a patient admitted to the hospital with an acute exacerbation of MS, what should the nurse expect to find? a. Tremors, dysphasia, and ptosis b. Bowel and bladder incontinence and loss of memory c. Motor impairment, visual disturbances, and paresthesias d. Excessive involuntary movements, hearing loss, and ataxia

21. c. Specific neurologic dysfunction of MS is caused by destruction of myelin and replacement with glial scar tissue at specific areas in the nervous system. Motor, sensory, cerebellar, and emotional dysfunctions, including paresthesias as well as patchy blindness, blurred vision, pain radiating along the dermatome of the nerve, ataxia, and severe fatigue, are the most common manifestations of MS. Constipation and bladder dysfunctions, short-term memory loss, sexual dysfunction, anger, and depression or euphoria may also occur. Excessive involuntary movements and tremors are not seen in MS.

After change-of-shift report, which patient should the nurse assess first?

23-year-old with type 1 diabetes who has a blood glucose of 40 mg/dL

23. Mitoxantrone (Novantrone) is being considered as treatment for a patient with progressive-relapsing MS. The nurse explains that a disadvantage of this drug compared with other drugs used for MS is what? a. It must be given subcutaneously every day. b. It has a lifetime dose limit because of cardiac toxicity. c. It is an anticholinergic agent that causes urinary incontinence. d. It is an immunosuppressant agent that increases the risk for infection.

23. b. Mitoxantrone (Novantrone) cannot be used for more than 2 to 3 years because it is an antineoplastic drug that causes cardiac toxicity, leukemia, and infertility. It is a monoclonal antibody given IV monthly when patients have inadequate responses to other drugs. It increases the risk of progressive multifocal leukoencephalopathy.

The nurse receives change-of-shift report on the oncology unit. Which patient should the nurse assess first?

24-year-old patient who received neck radiation and has blood oozing from the neck

Priority Decision: A patient with MS has a nursing diagnosis of self-care deficit related to muscle spasticity and neuromuscular deficits. In providing care for the patient, what is most important for the nurse to do? a. Teach the family members how to care adequately for the patient's needs. b. Encourage the patient to maintain social interactions to prevent social isolation. c. Promote the use of assistive devices so the patient can participate in self-care activities. d. Perform all activities of daily living (ADLs) for the patient to conserve the patient's energy.

24. c. The main goal in care of the patient with MS is to keep the patient active and maximally functional and promote self-care as much as possible to maintain independence. Assistive devices encourage independence while preserving the patient's energy. No care activity that the patient can do for himself or herself should be performed by others. Involvement of the family in the patient's care and maintenance of social interactions are also important but are not the priority in care.

A patient was admitted for a paracentesis to remove ascites fluid. Five liters of fluid was removed. Which IV solution may be used to pull fluid into the intravascular space after the paracentesis?

25% albumin solution

25. A patient with newly diagnosed MS has been hospitalized for evaluation and initial treatment of the disease. Following discharge teaching, the nurse realizes that additional instruction is needed when the patient says what? a. "It is important for me to avoid exposure to people with upper respiratory infections." b. "When I begin to feel better, I should stop taking the prednisone to prevent side effects." c. "I plan to use vitamin supplements and a high-protein diet to help manage my condition." d. "I must plan with my family how we are going to manage my care if I become more incapacitated."

25. b. Corticosteroids used in treating acute exacerbations of MS should not be abruptly stopped by the patient because adrenal insufficiency may result and prescribed tapering doses should be followed. Infections may exacerbate symptoms and should be avoided and high-protein diets with vitamin supplements are advocated. Long-term planning for increasing disability is also important.

26. The classic triad of manifestations associated with Parkinson's disease is tremor, rigidity, and bradykinesia. What is a consequence related to rigidity? a. Shuffling gait b. Impaired handwriting c. Lack of postural stability d. Muscle soreness and pain

26. d. The degeneration of dopamine-producing neurons in the substantia nigra of midbrain and basal ganglia lead to this triad of signs. Muscle soreness, pain, and slowness of movement are patient function consequences related to rigidity. Shuffling gait, lack of postural stability, absent arm swing while walking, absent blinking, masked facial expression, and difficulty initiating movement are all related to bradykinesia. Impaired handwriting and hand activities are related to the tremor of Parkinson's disease (PD).

27. A patient with a tremor is being evaluated for Parkinson's disease. The nurse explains to the patient that Parkinson's disease can be confirmed by a. CT and MRI scans. b. relief of symptoms with administration of dopaminergic agents. c. the presence of tremors that increase during voluntary movement. d. cerebral angiogram that reveals the presence of cerebral atherosclerosis.

27. b. Although clinical manifestations are characteristic in PD, no laboratory or diagnostic tests are specific for the condition. A diagnosis is made when at least two of the three signs of the classic triad are present and it is confirmed with a positive response to antiparkinsonian medication. Research regarding the role of genetic testing and MRI to diagnose PD is ongoing. Essential tremors increase during voluntary movement whereas the tremors of PD are more prominent at rest.

28. Which observation of the patient made by the nurse is most indicative of Parkinson's disease? a. Large, embellished handwriting b. Weakness of one leg resulting in a limping walk c. Difficulty rising from a chair and beginning to walk d. Onset of muscle spasms occurring with voluntary movement

28. c. The bradykinesia of PD prevents automatic movements and activities such as beginning to walk, rising from a chair, or even swallowing saliva cannot be executed unless they are consciously willed. Handwriting is affected by the tremor and results in the writing trailing off at the end of words. Specific limb weakness and muscle spasms are not characteristic of PD.

29. A patient with Parkinson's disease is started on levodopa. What should the nurse explain about this drug? a. It stimulates dopamine receptors in the basal ganglia. b. It promotes the release of dopamine from brain neurons. c. It is a precursor of dopamine that is converted to dopamine in the brain. d. It prevents the excessive breakdown of dopamine in the peripheral tissues.

29. c. Peripheral dopamine does not cross the blood-brain barrier but its precursor, levodopa, is able to enter the brain, where it is converted to dopamine, increasing the supply that is deficient in PD. Other drugs used to treat PD include bromocriptine, which stimulates dopamine receptors in the basal ganglia, and amantadine, which blocks the reuptake of dopamine into presynaptic neurons. Carbidopa is an agent that is usually administered with levodopa to prevent the levodopa from being metabolized in peripheral tissues before it can reach the brain.

A 68-year-old female patient admitted to the hospital with dehydration is confused and incontinent of urine. Which nursing action will be best to include in the plan of care?

Assist the patient to the bathroom every 2 hours during the day.

a nurse is caring for a client with diabetes mellitus who receives 25 units of NHP insulin every morning if her blood glucose level is above 200 mg/dl. The nurse monitors this client for hypoglycemia because NPH insulin peaks how long after administration?

4-12 hr

The oxygen saturation (SpO2) for a patient with left lower lobe pneumonia is 90%. The patient has rhonchi, a weak cough effort, and complains of fatigue. Which action is a priority for the nurse to take?

Assist the patient with staged coughing.

An intraaortic balloon pump (IABP) is being used for a patient who is in cardiogenic shock. Which assessment data indicate to the nurse that the goals of treatment with the IABP are being met?

Cardiac output (CO) of 5 L/min

A patient complains of gas pains and abdominal distention two days after a small bowel resection. Which nursing action is best to take? a. Encourage the patient to ambulate. b. Instill a mineral oil retention enema. c. Administer the ordered IV morphine sulfate. d. Offer the ordered promethazine (Phenergan) suppository.

A

Which serum potassium result best supports the rationale for administering a stat dose of IV potassium chloride 20 mEq in 200 mL of normal saline over 2 hours?

3.1 mEq/L

31. A patient with myasthenia gravis is admitted to the hospital with respiratory insufficiency and severe weakness. When is a diagnosis of cholinergic crisis made? a. The patient's respiration is impaired because of muscle weakness. b. Administration of edrophonium (Tensilon) increases muscle weakness. c. Administration of edrophonium (Tensilon) results in improved muscle contractility. d. EMG reveals decreased response to repeated stimulation of muscles.

31. b. The reduction of the acetylcholine (ACh) effect in myasthenia gravis (MG) is treated with anticholinesterase drugs, which prolong the action of ACh at the neuromuscular synapse, but too much of these drugs will cause a cholinergic crisis with symptoms very similar to those of MG. To determine whether the patient's manifestations are due to a deficiency of ACh or to too much anticholinesterase drug, the anticholinesterase drug edrophonium chloride (Tensilon) is administered. If the patient is in cholinergic crisis, the patient's symptoms will worsen; if the patient is in a myasthenic crisis, the patient will improve.

Priority Decision: During care of a patient in myasthenic crisis, maintenance of what is the nurse's first priority for the patient? a. Mobility b. Nutrition c. Respiratory function d. Verbal communication

32. c. The patient in myasthenic crisis has severe weakness and fatigability of all skeletal muscles, affecting the patient's ability to breathe, swallow, talk, and move. However, the priority of nursing care is monitoring and maintaining adequate ventilation.

33. When providing care for a patient with ALS, the nurse recognizes what as one of the most distressing problems experienced by the patient? a. Painful spasticity of the face and extremities b. Retention of cognitive function with total degeneration of motor function c. Uncontrollable writhing and twisting movements of the face, limbs, and body d. Knowledge that there is a 50% chance the disease has been passed to any offspring

33. b. In ALS there is gradual degeneration of motor neurons with extreme muscle wasting from lack of stimulation and use. However, cognitive function is not impaired and patients feel trapped in a dying body. Chorea manifested by writhing, involuntary movements is characteristic of HD. As an autosomal dominant genetic disease, HD also has a 50% chance of being passed to each offspring.

34. In providing care for patients with chronic, progressive neurologic disease, what is the major goal of treatment that the nurse works toward? a. Meet the patient's personal care needs. b. Return the patient to normal neurologic function. c. Maximize neurologic functioning for as long as possible. d. Prevent the development of additional chronic diseases.

34. c. Many chronic neurologic diseases involve progressive deterioration in physical or mental capabilities and have no cure, with devastating results for patients and families. Health care providers can only attempt to alleviate physical symptoms, prevent complications, and assist patients in maximizing function and self-care abilities for as long as possible.

The nurse is caring for a patient with mitral regurgitation. Referring to the figure below, where should the nurse listen to best hear any murmur that the patient has?

4

A patient who has acute glomerulonephritis is hospitalized with hyperkalemia. Which information will the nurse monitor to evaluate the effectiveness of the prescribed calcium gluconate IV?

Cardiac rhythm

4. What drug therapy is included for acute migraine and cluster headaches that appears to alter the pathophysiologic process for these headaches? a. β-Adrenergic blockers such as propranolol (Inderal) b. Serotonin antagonists such as methysergide (Sansert) c. Tricyclic antidepressants such as amitriptyline (Elavil) d. Specific serotonin receptor agonists such as sumatriptan (Imitrex)

4. d. Triptans (sumatriptan [Imitrex]) affect selected serotonin receptors that decrease neurogenic inflammation of the cerebral blood vessels and produce vasoconstriction. Both migraine headaches and cluster headaches appear to be related to vasodilation of cranial vessels and drugs that cause vasoconstriction are useful in treatment of migraine and cluster headaches. Methysergide blocks serotonin receptors in the central and peripheral nervous systems and is used for prevention of migraine and cluster headaches. β adrenergic blockers and tricyclic antidepressants are used prophylactically for migraine headaches but are not effective for cluster headaches.

A patient has a junctional escape rhythm on the monitor. The nurse will expect the patient to have a heart rate of _____ beats/minute.

40 to 60

The nurse monitors a patient after chest tube placement for a hemopneumothorax. The nurse is most concerned if which assessment finding is observed?

400 mL of blood in the collection chamber

5. What is a nursing intervention that is appropriate for the patient with a nursing diagnosis of anxiety related to lack of knowledge of the etiology and treatment of headache? a. Help the patient to examine lifestyle patterns and precipitating factors. b. Administer medications as ordered to relieve pain and promote relaxation. c. Provide a quiet, dimly lit environment to reduce stimuli that increase muscle tension and anxiety. d. Support the patient's use of counseling or psychotherapy to enhance conflict resolution and stress reduction.

5. a. When the anxiety is related to a lack of knowledge about the etiology and treatment of a headache, helping the patient to identify stressful lifestyle patterns and other precipitating factors and ways of avoiding them are appropriate nursing interventions for the anxiety. Interventions that teach alternative therapies to supplement drug therapy also give the patient some control over pain and are appropriate teaching regarding treatment of the headache. The other interventions may help to reduce anxiety generally but they do not address the etiologic factor of the anxiety.

When analyzing an electrocardiographic (ECG) rhythm strip of a patient with a regular heart rhythm, the nurse counts 30 small blocks from one R wave to the next. The nurse calculates the patients heart rate as ____.

50

A patients vital signs are pulse 87, respirations 24, and BP of 128/64 mm Hg and cardiac output is 4.7 L/min. The patients stroke volume is _____ mL. (Round to the nearest whole number.)

54

6. Delegation Decision: The nurse is preparing to admit a newly diagnosed patient experiencing tonic-clonic seizures. What could the nurse delegate to unlicensed assistive personnel (UAP)? a. Complete the admission assessment. b. Explain the call system to the patient. c. Obtain the suction equipment from the supply cabinet. d. Place a padded tongue blade on the wall above the patient's bed.

6. c. The unlicensed assistive personnel (UAP) is able to obtain equipment from the supply cabinet or department. The RN may need to provide a list of necessary equipment and should set up the equipment and ensure proper functioning. The RN is responsible for the initial history and assessment as well as teaching the patient about the room's call system. Padded tongue blades are no longer used and no effort should be made to place anything in the patient's mouth during a seizure.

The result of a patient's creatinine clearance test is 60 mL/min. The nurse equates this finding to a glomerular filtration rate (GFR) of _____ mL/min. 30 60 120 240

60 The creatinine clearance approximates the GFR.

After change-of-shift report, which patient will the nurse assess first?

60-year-old with hyperosmolar hyperglycemic syndrome who has poor skin turgor and dry oral mucosa

After change-of-shift report, which patient should the nurse assess first?

64-year-old with lung cancer and tracheal deviation after subclavian catheter insertion

A patient with an intracranial problem does not open his eyes to any stimulus, has no verbal response except moaning and muttering when stimulated, and flexes his arm in response to painful stimuli. What should the nurse record as the patient's GCS score?

7

7. How do generalized seizures differ from focal seizures? a. Focal seizures are confined to one side of the brain and remain focal in nature. b. Generalized seizures result in loss of consciousness whereas focal seizures do not. c. Generalized seizures result in temporary residual deficits during the postictal phase. d. Generalized seizures have bilateral synchronous epileptic discharges affecting the whole brain at onset of the seizure.

7. d. Generalized seizures have bilateral synchronous epileptic discharge affecting the entire brain at onset of the seizure. Loss of consciousness is also characteristic but many focal seizures also include an altered consciousness. Focal seizures begin in one side of the brain but may spread to involve the entire brain. Focal seizures that start with a local focus and spread to the entire brain, causing a secondary generalized seizure, are associated with a transient residual neurologic deficit postictally known as Todd's paralysis.

You are caring for a patient receiving D5W at a rate of 125 mL/hr. During the 4:00 PM assessment of the patient, you determine that 500 mL is left in the present IV bag. At what time should the nurse anticipate hanging the next bag of D5W?

8:00 PM

9. The patient is diagnosed with complex focal seizures. Which characteristics are related to complex focal seizures (select all that apply)? a. Formerly known as grand mal seizure b. Often accompanied by incontinence or tongue or cheek biting c. Psychomotor seizures with repetitive behaviors and lip smacking d. Altered memory, sexual sensations, and distortions of visual or auditory sensations e. Loss of consciousness and stiffening of the body with subsequent jerking of extremities f. Often involves behavioral, emotional, and cognitive functions with altered consciousness

9. c, d, f. Complex focal seizures are psychomotor seizures with automatisms such as lip smacking. They cause altered consciousness or loss of consciousness producing a dreamlike state and may involve behavioral, emotional, or cognitive experiences without memory of what was done during the seizure. In generalized tonic-clonic seizures (previously known as grand mal seizures) there is loss of consciousness and stiffening of the body with subsequent jerking of extremities. Incontinence or tongue or cheek biting may also occur.

A patient in the oliguric phase after an acute kidney injury has had a 250 mL urine output and an emesis of 100 mL in the past 24 hours. What is the patients fluid restriction for the next 24 hours?

950 mL (add 600 to the existing output)

A 22-year-old patient is being seen in the clinic with increased secretion of the anterior pituitary hormones. The nurse would expect the laboratory results to show a. increased urinary cortisol. b. decreased serum thyroxine. c. elevated serum aldosterone levels. d. low urinary catecholamines excretion.

A

A 34-year-old female patient with a new ileostomy asks how much drainage to expect. The nurse explains that after the bowel adjusts to the ileostomy, the usual drainage will be about _____ cups. a. 2 b. 3 c. 4 d. 5

A

A 50-year-old female patient calls the clinic to report a new onset of severe diarrhea. The nurse anticipates that the patient will need to a. collect a stool specimen. b. prepare for colonoscopy. c. schedule a barium enema. d. have blood cultures drawn.

A

A 51-year-old male patient has a new diagnosis of Crohn's disease after having frequent diarrhea and a weight loss of 10 pounds (4.5 kg) over 2 months. The nurse will plan to teach about a. medication use. b. fluid restriction. c. enteral nutrition. d. activity restrictions.

A

A 51-year-old woman with Crohn's disease who is taking infliximab (Remicade) calls the nurse in the outpatient clinic about new symptoms. Which symptom is most important to communicate to the health care provider? a. Fever b. Nausea c. Joint pain d. Headache

A

A female patient is awaiting surgery for acute peritonitis. Which action will the nurse include in the plan of care? a. Position patient with the knees flexed. b. Avoid use of opioids or sedative drugs. c. Offer frequent small sips of clear liquids. d. Assist patient to breathe deeply and cough.

A

The nurse admitting a patient with acute diverticulitis explains that the initial plan of care is to a. administer IV fluids. b. give stool softeners and enemas. c. order a diet high in fiber and fluids. d. prepare the patient for colonoscopy.

A

The nurse is caring for a 63-year-old with a possible pituitary tumor who is scheduled for a computed tomography (CT) scan with contrast. Which information about the patient is most important to discuss with the health care provider before the test? a. History of renal insufficiency b. Complains of chronic headache c. Recent bilateral visual field loss d. Blood glucose level of 134 mg/dL

A

Which nursing action will be included in the plan of care for a 27-year-old male patient with bowel irregularity and a new diagnosis of irritable bowel syndrome (IBS)? a. Encourage the patient to express concerns and ask questions about IBS. b. Suggest that the patient increase the intake of milk and other dairy products. c. Educate the patient about the use of alosetron (Lotronex) to reduce symptoms. d. Teach the patient to avoid using nonsteroidal antiinflammatory drugs (NSAIDs).

A

Physiological Integrity 21. Which assessment finding in a patient admitted with acute decompensated heart failure (ADHF) requires the most immediate action by the nurse? a. Oxygen saturation of 88% b. Weight gain of 1 kg (2.2 lb) c. Heart rate of 106 beats/minute d. Urine output of 50 mL over 2 hours

A A decrease in oxygen saturation to less than 92% indicates hypoxemia. The nurse should administer supplemental oxygen immediately to the patient. An increase in apical pulse rate, 1-kg weight gain, and decreases in urine output also indicate worsening heart failure and require nursing actions, but the low oxygen saturation rate requires the most immediate nursing action. DIF: Cognitive Level: Apply (application) REF: 769-770 OBJ: Special Questions: Prioritization TOP: Nursing Process: Assessment MSC:

Which patient in the ear, nose, and throat (ENT) clinic should the nurse assess first?

A 23-year-old who is complaining of a sore throat and has a muffled voice

The following four patients arrive in the emergency department (ED) after a motor vehicle collision. Which patient has the lowest priority?

A 37-year-old with a misaligned left leg with intact pulses

The nurse receives change-of-shift report on the following four patients. Which patient should the nurse assess first?

A 46-year-old patient on bed rest who is complaining of sudden onset of shortness of breath

Physiological Integrity 19. A patient in the intensive care unit with acute decompensated heart failure (ADHF) complains of severe dyspnea and is anxious, tachypneic, and tachycardic. All of the following medications have been ordered for the patient. The nurse's priority action will be to a. give IV morphine sulfate 4 mg. b. give IV diazepam (Valium) 2.5 mg. c. increase nitroglycerin (Tridil) infusion by 5 mcg/min. d. increase dopamine (Intropin) infusion by 2 mcg/kg/min.

A Morphine improves alveolar gas exchange, improves cardiac output by reducing ventricular preload and afterload, decreases anxiety, and assists in reducing the subjective feeling of dyspnea. Diazepam may decrease patient anxiety, but it will not improve the cardiac output or gas exchange. Increasing the dopamine may improve cardiac output, but it will also increase the heart rate and myocardial oxygen consumption. Nitroglycerin will improve cardiac output and may be appropriate for this patient, but it will not directly reduce anxiety and will not act as quickly as morphine to decrease dyspnea. DIF: Cognitive Level: Analyze (analysis) REF: 774 OBJ: Special Questions: Prioritization TOP: Nursing Process: Implementation MSC:

Physiological Integrity 14. Which action should the nurse include in the plan of care when caring for a patient admitted with acute decompensated heart failure (ADHF) who is receiving nesiritide (Natrecor)? a. Monitor blood pressure frequently. b. Encourage patient to ambulate in room. c. Titrate nesiritide slowly before stopping. d. Teach patient about home use of the drug.

A Nesiritide is a potent arterial and venous dilator, and the major adverse effect is hypotension. Because the patient is likely to have orthostatic hypotension, the patient should not be encouraged to ambulate. Nesiritide does not require titration and is used for ADHF but not in a home setting. DIF: Cognitive Level: Apply (application) REF: 774 TOP: Nursing Process: Planning MSC:

Physiological Integrity 20. After receiving change-of-shift report on a heart failure unit, which patient should the nurse assess first? a. A patient who is cool and clammy, with new-onset confusion and restlessness b. A patient who has crackles bilaterally in the lung bases and is receiving oxygen. c. A patient who had dizziness after receiving the first dose of captopril (Capoten) d. A patient who is receiving IV nesiritide (Natrecor) and has a blood pressure of 100/62

A The patient who has "wet-cold" clinical manifestations of heart failure is perfusing inadequately and needs rapid assessment and changes in management. The other patients also should be assessed as quickly as possible but do not have indications of severe decreases in tissue perfusion. DIF: Cognitive Level: Analyze (analysis) REF: 770 OBJ: Special Questions: Prioritization; Multiple Patients TOP: Nursing Process: Assessment MSC:

Physiological Integrity 6. During a visit to a 78-year-old with chronic heart failure, the home care nurse finds that the patient has ankle edema, a 2-kg weight gain over the past 2 days, and complains of "feeling too tired to get out of bed." Based on these data, the best nursing diagnosis for the patient is a. activity intolerance related to fatigue. b. disturbed body image related to weight gain. c. impaired skin integrity related to ankle edema. d. impaired gas exchange related to dyspnea on exertion.

A The patient's statement supports the diagnosis of activity intolerance. There are no data to support the other diagnoses, although the nurse will need to assess for other patient problems. DIF: Cognitive Level: Apply (application) REF: 770 TOP: Nursing Process: Diagnosis MSC:

Despite a high dosage, a male patient who is taking nifedipine (Procardia XL) for antihypertensive therapy continues to have blood pressures over 140/90 mmHg. What should the nurse do next? A Assess his adherence to therapy. B Ask him to make an exercise plan. C Instruct him to use the DASH diet. D Request a prescription for a thiazide diuretic.

A Assess his adherence to therapy. A long-acting calcium-channel blocker such as nifedipine causes vascular smooth muscle relaxation resulting in decreased SVR and arterial BP and related side effects. The patient data the nurse has about this patient is very limited, so the nurse needs to assess his adherence to therapy.

Which information will the nurse include when teaching a patient how to avoid chronic constipation (select all that apply)? a. Many over-the-counter (OTC) medications can cause constipation. b. Stimulant and saline laxatives can be used regularly. c. Bulk-forming laxatives are an excellent source of fiber. d. Walking or cycling frequently will help bowel motility. e. A good time for a bowel movement may be after breakfast.

A C D E

Which statements will the nurse include when teaching a patient who is scheduled for oral glucose tolerance testing in the outpatient clinic (select all that apply)? a. "You will need to avoid smoking before the test." b. "Exercise should be avoided until the testing is complete." c. "Several blood samples will be obtained during the testing." d. "You should follow a low-calorie diet the day before the test." e. "The test requires that you fast for at least 8 hours before testing."

A C E

The nurse has received change-of-shift report about the following patients on the progressive care unit. Which patient should the nurse see first?

A patient whose implantable cardioverter-defibrillator (ICD) fired two times today who has a dose of amiodarone (Cordarone) due

Following an earthquake, patients are triaged by emergency medical personnel and are transported to the hospital. Which of these patients will the nurse need to assess first?

A patient with a red tag

A patient will need vascular access for hemodialysis. Which statement by the nurse accurately describes an advantage of a fistula over a graft?

A fistula is much less likely to clot.

Which menu choice by the patient with diverticulosis is best for preventing diverticulitis? a. Navy bean soup and vegetable salad b. Whole grain pasta with tomato sauce c. Baked potato with low-fat sour cream d. Roast beef sandwich on whole wheat bread

A low fat and low fiber diet

The nurse on a medical-surgical unit identifies which patient as having the highest risk for metabolic alkalosis?

A patient with nasogastric tube suction

The ED triage nurse is assessing four victims involved in a motor vehicle collision. Which patient has the highest priority for treatment?

A patient with paradoxic chest movements

After receiving change-of-shift report on a medical unit, which patient should the nurse assess first?

A patient with septicemia who has intercostal and suprasternal retractions

The nurse teaches a patient with hypertension that uncontrolled hypertension may damage organs in the body primarily by which mechanism? A Hypertension promotes atherosclerosis and damage to the walls of the arteries. Hypertension causes direct pressure on organs, Bresulting in necrosis and replacement of cells with scar tissue. C Hypertension causes thickening of the capillary membranes, leading to hypoxia of organ systems. D Hypertension increases blood viscosity, which contributes to intravascular coagulation and tissue necrosis distal to occlusions.

A Hypertension promotes atherosclerosis and damage to the walls of the arteries. Hypertension is a major risk factor for the development of atherosclerosis by mechanisms not yet fully known. However, once atherosclerosis develops, it damages the walls of arteries and reduces circulation to target organs and tissues.

The nurse is caring for a patient with hypertension who is scheduled to receive a dose of esmolol (Brevibloc). The nurse should withhold the dose and consult the prescribing physician for which vital sign taken just before administration? A Pulse 48 B Respirations 24 C Blood pressure 118/74 D Oxygen saturation 93%

A Pulse 48 Because esmolol is a β1-adrenergic blocking agent, it can cause hypotension and bradycardia as adverse effects. The nurse should withhold the dose and consult with the health care provider for parameters regarding pulse rate limits.

When conducting a primary survey on a trauma patient, which of the following is considered one of the priority elements of the primary survey?

A brief neurologic assessment

In a multiple-trauma victim, which assessment finding signals the most serious and life-threatening condition?

A deviated trachea

When caring for a patient who is hospitalized with active tuberculosis (TB), the nurse observes a student nurse who is assigned to take care of a patient. Which action, if performed by the student nurse, would require an intervention by the nurse?

A surgical face mask is applied before visiting the patient.

A patient with Parkinson's disease is admitted to the hospital for treatment of pneumonia. Which nursing interventions will be included in the plan of care (select all that apply)? a. Use an elevated toilet seat. b. Cut patient's food into small pieces. c. Provide high-protein foods at each meal. d. Place an armchair at the patient's bedside. e. Observe for sudden exacerbation of symptoms.

A, B, D Because the patient with Parkinson's has difficulty chewing, food should be cut into small pieces. An armchair should be used when the patient is seated so that the patient can use the arms to assist with getting up from the chair. An elevated toilet seat will facilitate getting on and off the toilet. High-protein foods will decrease the effectiveness of L-dopa. Parkinson's is a steadily progressive disease without acute exacerbations.

A 27-year-old patient who has been treated for status epilepticus in the emergency department will be transferred to the medical nursing unit. Which equipment should the nurse have available in the patient's assigned room (select all that apply)? a. Side-rail pads b. Tongue blade c. Oxygen mask d. Suction tubing e. Urinary catheter f. Nasogastric tube

A, C, D The patient is at risk for further seizures, and oxygen and suctioning may be needed after any seizures to clear the airway and maximize oxygenation. The bed's side rails should be padded to minimize the risk for patient injury during a seizure. Use of tongue blades during a seizure is contraindicated. Insertion of a nasogastric (NG) tube is not indicated because the airway problem is not caused by vomiting or abdominal distention. A urinary catheter is not required unless there is urinary retention.

Safe and Effective Care Environment MULTIPLE RESPONSE 1. Based on the Joint Commission Core Measures for patients with heart failure, which topics should the nurse include in the discharge teaching plan for a patient who has been hospitalized with chronic heart failure (select all that apply)? a. How to take and record daily weight b. Importance of limiting aerobic exercise c. Date and time of follow-up appointment d. Symptoms indicating worsening heart failure e. Actions and side effects of prescribed medications

A, C, D, E The Joint Commission Core Measures state that patients should be taught about prescribed medications, follow-up appointments, weight monitoring, and actions to take for worsening symptoms. Patients with heart failure are encouraged to begin or continue aerobic exercises such as walking, while self-monitoring to avoid excessive fatigue. DIF: Cognitive Level: Apply (application) REF: 779 TOP: Nursing Process: Planning MSC:

A 48-year-old man was just diagnosed with Huntington's disease. His 20-year-old son is upset about his father's diagnosis. How can the nurse best help this young man? A. Provide emotional and psychologic support. B. Encourage him to get diagnostic genetic testing done. C. Tell him the cognitive deterioration will be treated with counseling. D. Tell him the chorea and psychiatric disorders can be treated with haloperidol (Haldol).

A. The patient's son will first need emotional and psychologic support. He should be taught about diagnostic genetic testing for himself but should decide for himself with a genetic counselor if and when he wants this done. The treatment plan for his father will be determined depending on his father's needs.

The patient with type 1 diabetes mellitus with hypoglycemia is having a seizure. Which medication should the nurse anticipate administering to stop the seizure? A. IV dextrose solution B. IV diazepam (Valium) C. IV phenytoin (Dilantin) D. Oral carbamazepine (Tegretol)

A. This patient's seizure is caused by low blood glucose, so IV dextrose solution should be given first to stop the seizure. IV diazepam, IV phenytoin, and oral carbamazepine would be used to treat seizures from other causes such as head trauma, drugs, and infections.

A 64-year-old patient who has amyotrophic lateral sclerosis (ALS) is hospitalized with pneumonia. Which nursing action will be included in the plan of care? a. Assist with active range of motion (ROM). b. Observe for agitation and paranoia. c. Give muscle relaxants as needed to reduce spasms. d. Use simple words and phrases to explain procedures.

A. ALS causes progressive muscle weakness, but assisting the patient to perform active ROM will help maintain strength as long as possible. Psychotic manifestations such as agitation and paranoia are not associated with ALS. Cognitive function is not affected by ALS, and the patient's ability to understand procedures will not be impaired. Muscle relaxants will further increase muscle weakness and depress respirations.

A hospitalized 31-year-old patient with a history of cluster headache awakens during the night with a severe stabbing headache. Which action should the nurse take first? a. Start the ordered PRN oxygen at 6 L/min. b. Put a moist hot pack on the patient's neck. c. Give the ordered PRN acetaminophen (Tylenol). d. Notify the patient's health care provider immediately.

A. Acute treatment for cluster headache is administration of 100% oxygen at 6 to 8 L/min. If the patient obtains relief with the oxygen, there is no immediate need to notify the health care provider. Cluster headaches last only 60 to 90 minutes, so oral pain medications have minimal effect. Hot packs are helpful for tension headaches but are not as likely to reduce pain associated with a cluster headache.

After change-of-shift report, which patient should the nurse assess first? a. Patient with myasthenia gravis who is reporting increased muscle weakness b. Patient with a bilateral headache described as "like a band around my head" c. Patient with seizures who is scheduled to receive a dose of phenytoin (Dilantin) d. Patient with Parkinson's disease who has developed cogwheel rigidity of the arms

A. Because increased muscle weakness may indicate the onset of a myasthenic crisis, the nurse should assess this patient first. The other patients should also be assessed, but do not appear to need immediate nursing assessments or actions to prevent life-threatening complications.

A 31-year-old woman who has multiple sclerosis (MS) asks the nurse about risks associated with pregnancy. Which response by the nurse is accurate? a. "MS symptoms may be worse after the pregnancy." b. "Women with MS frequently have premature labor." c. "MS is associated with an increased risk for congenital defects." d. "Symptoms of MS are likely to become worse during pregnancy."

A. During the postpartum period, women with MS are at greater risk for exacerbation of symptoms. There is no increased risk for congenital defects in infants born of mothers with MS. Symptoms of MS may improve during pregnancy. Onset of labor is not affected by MS.

Which measure should the nurse prioritize when providing care for a patient with a diagnosis of multiple sclerosis (MS)? A. Vigilant infection control and adherence to standard precautions B. Careful monitoring of neurologic assessment and frequent reorientation C. Maintenance of a calorie count and hourly assessment of intake and output D. Assessment of blood pressure and monitoring for signs of orthostatic hypotension

A. Infection control is a priority in the care of patients with MS, since infection is the most common cause of an exacerbation of the disease. Decreases in cognitive function are less likely, and MS does not typically result in malnutrition, hypotension, or fluid volume excess or deficit.

The nurse advises a patient with myasthenia gravis (MG) to a. perform physically demanding activities early in the day. b. anticipate the need for weekly plasmapheresis treatments. c. do frequent weight-bearing exercise to prevent muscle atrophy. d. protect the extremities from injury due to poor sensory perception.

A. Muscles are generally strongest in the morning, and activities involving muscle activity should be scheduled then. Plasmapheresis is not routinely scheduled, but is used for myasthenia crisis or for situations in which corticosteroid therapy must be avoided. There is no decrease in sensation with MG, and muscle atrophy does not occur because although there is muscle weakness, they are still used.

A patient has been taking phenytoin (Dilantin) for 2 years. Which action will the nurse take when evaluating for adverse effects of the medication? a. Inspect the oral mucosa. b. Listen to the lung sounds. c. Auscultate the bowel tones. d. Check pupil reaction to light.

A. Phenytoin can cause gingival hyperplasia, but does not affect bowel tones, lung sounds, or pupil reaction to light.

A 46-year-old patient tells the nurse about using acetaminophen (Tylenol) several times every day for recurrent bilateral headaches. Which action will the nurse plan to take first? a. Discuss the need to stop taking the acetaminophen. b. Suggest the use of biofeedback for headache control. c. Describe the use of botulism toxin (Botox) for headaches. d. Teach the patient about magnetic resonance imaging (MRI).

A. The headache description suggests that the patient is experiencing medication overuse headache. The initial action will be withdrawal of the medication. The other actions may be needed if the headaches persist.

A patient reports feeling numbness and tingling of the left arm before experiencing a tonic-clonic seizure. The nurse determines that this history is consistent with what type of seizure? a. Focal b. Atonic c. Absence d. Myoclonic

A. The initial symptoms of a focal seizure involve clinical manifestations that are localized to a particular part of the body or brain. Symptoms of an absence seizure are staring and a brief loss of consciousness. In an atonic seizure, the patient loses muscle tone and (typically) falls to the ground. Myoclonic seizures are characterized by a sudden jerk of the body or extremities.

The nurse observes a patient ambulating in the hospital hall when the patient's arms and legs suddenly jerk and the patient falls to the floor. The nurse will first a. assess the patient for a possible head injury. b. give the scheduled dose of divalproex (Depakote). c. document the timing and description of the seizure. d. notify the patient's health care provider about the seizure.

A. The patient who has had a myoclonic seizure and fall is at risk for head injury and should first be evaluated and treated for this possible complication. Documentation of the seizure, notification of the seizure, and administration of antiseizure medications are also appropriate actions, but the initial action should be assessment for injury.

In which patient would it be the most important for the nurse to assess the glossopharyngeal and vagus nerves? A. A 50-year-old woman with lethargy from a drug overdose B. A 40-year-old man with a complete lumbar spinal cord injury C. A 60-year-old man with severe pain from trigeminal neuralgia D. A 30-year-old woman with a high fever and bacterial meningitis

A. A 50-year-old woman with lethargy from a drug overdose The glossopharyngeal and vagus nerves innervate the pharynx and are tested by the gag reflex. It is important to assess the gag reflex in patients who have a decreased level of consciousness, a brainstem lesion, or a disease involving the throat musculature. If the reflex is weak or absent, the patient is in danger of aspirating food or secretions.

When teaching a patient about the most important respiratory defense mechanism distal to the respiratory bronchioles, which topic would the nurse discuss? A. Alveolar macrophages B. Impaction of particles C. Reflex bronchoconstriction D. Mucociliary clearance mechanism

A. Alveolar macrophages Respiratory defense mechanisms are efficient in protecting the lungs from inhaled particles, microorganisms, and toxic gases. Because ciliated cells are not found below the level of the respiratory bronchioles, the primary defense mechanism at the alveolar level is alveolar macrophages.

Which of the following actions will the nurse plan to take first when admitting a patient who has a history of neurogenic bladder as a result of a spinal cord injury? A. Ask about the usual urinary pattern & any measures used for bladder control. B. Assist the patient to the toilet at scheduled times to help ensure bladder emptying. C. Check the patient for urinary incontinence every 2 hours to maintain skin integrity. D. Use intermittent catheterization on a regular schedule to avoid the risk of infection.

A. Ask about the usual urinary pattern & any measures used for bladder control. Before planning any interventions, the nurse should complete the assessment and determine the patient's normal bladder pattern & the usual measures used by the patient at home. All the other responses may be appropriate, but until the assessment is complete, an individualized plan for the patient cannot be developed.

The nurse is completing a health assessment for an obese 62-year-old man who wants to begin a diet and exercise program. Which assessment should the nurse perform to determine the cognitive function of the patient during the physical examination? A. Ask the patient a question such as, "Who were the last three presidents?" B. Determine the level of consciousness, body posture, and facial expressions. C. Observe for signs of agitation, anger, or depression during the health check. D. Request that the patient mimic rapid, alternating movements with both hands.

A. Ask the patient a question such as, "Who were the last three presidents?" Cognition is one component of the mental status examination to determine cerebral functioning. Cognition is assessed by determining orientation, memory, general knowledge, insight, judgment, problem solving, and calculation. A question often used to determine cognition for adults living in the United States is, "Who were the last three presidents?" General appearance and behavior is another component and includes level of consciousness, body posture, and facial expressions. Mood and affect are assessed by observing for agitation, anger, or depression. Cerebellar function is determined by assessing balance and coordination and may include testing rapid alternating movements of the upper and lower extremities.

The nurse cares for a 34-year-old woman after a lumbar puncture. Which action by the nurse is most appropriate? A. Assess for drainage or bleeding from the puncture site. B. Monitor for bladder dysfunction and bowel incontinence. C. Maintain bed rest until lower extremities move normally. D. Check for loss of muscle strength in the upper extremities.

A. Assess for drainage or bleeding from the puncture site. After a lumbar puncture the nurse should monitor the puncture site for drainage or bleeding. Other assessments include headache intensity, meningeal irritation (nuchal rigidity), signs and symptoms of local trauma (e.g., hematoma, pain), neurologic signs, and vital signs. A lumbar puncture does not affect bowel or bladder function or upper extremity muscle strength. Bed rest until lower extremity movement returns is indicated for the patient after spinal anesthesia.

When assessing a patient with a traumatic brain injury, the nurse notes uncoordinated movement of the extremities. How should the nurse document this assessment? A. Ataxia B. Apraxia C. Anisocoria D. Anosognosia

A. Ataxia Ataxia is a lack of coordination of movement, possibly caused by lesions of sensory or motor pathways, cerebellum disorders, or certain medications. Apraxia is the inability to perform learned movements despite having the desire and physical ability to perform them related to a cerebral cortex lesion. Anisocoria is inequality of pupil size from an optic nerve injury. Anosognosia is the inability to recognize a bodily defect or disease related to lesions in the right parietal cortex.

The nurse is obtaining a focused respiratory assessment of a 44-year-old female patient who is in severe respiratory distress 2 days after abdominal surgery. What is most important for the nurse to assess? A. Auscultation of bilateral breath sounds B. Percussion of anterior and posterior chest wall C. Palpation of the chest bilaterally for tactile fremitus D. Inspection for anterior and posterior chest expansion

A. Auscultation of bilateral breath sounds Important assessments obtained during a focused respiratory assessment include auscultation of lung (breath) sounds. Assessment of tactile fremitus has limited value in acute respiratory distress. It is not necessary to assess for both anterior and posterior chest expansion. Percussion of the chest wall is not essential in a focused respiratory assessment.

A frail 82-year-old female patient develops sudden shortness of breath while sitting in a chair. What location on the chest should the nurse begin auscultation of the lung fields? A. Bases of the posterior chest area B. Apices of the posterior lung fields C. Anterior chest area above the breasts D. Midaxillary on the left side of the chest

A. Bases of the posterior chest area Baseline data with the most information is best obtained by auscultation of the posterior chest, especially in female patients because of breast tissue interfering with the assessment or if the patient may tire easily (e.g., shortness of breath, dyspnea, weakness, fatigue). Usually auscultation proceeds from the lung apices to the bases unless it is possible the patient will tire easily. In this case the nurse should start at the bases.

The patient is calling the clinic with a cough. What assessment should be made first before the nurse advises the patient? A. Cough sound, sputum production, pattern B. Frequency, a family history, hematemesis C. Smoking, medications, residence location D. Weight loss, activity tolerance, orthopnea

A. Cough sound, sputum production, pattern The sound of the cough, sputum production and description, as well as pattern of the cough's occurrence (including acute or chronic) and what its occurrence is related to are the first assessments to be made to determine the severity. Frequency of the cough will not provide a lot of information. Family history can help to determine a genetic cause of the cough. Hematemesis is vomiting blood and not as important as hemoptysis. Smoking is an important risk factor for COPD and lung cancer and may cause a cough. Medications may or may not contribute to a cough as does residence location. Weight loss, activity intolerance, and orthopnea may be related to respiratory or cardiac problems, but are not as important when dealing with a cough.

When assessing a patient's sleep-rest pattern related to respiratory health, what should the nurse ask the patient about (select all that apply)? A. Have trouble falling asleep? B. Need to urinate during the night? C. Awaken abruptly during the night? D. Sleep more than 8 hours per night? E. Need to sleep with the head elevated?

A. Have trouble falling asleep? C. Awaken abruptly during the night? E. Need to sleep with the head elevated? The patient with sleep apnea may have insomnia and/or abrupt awakenings. Patients with cardiovascular disease (e.g., heart failure that may affect respiratory health) may need to sleep with the head elevated on several pillows (orthopnea). Sleeping more than 8 hours per night or needing to urinate during the night is not indicative of impaired respiratory health.

A patient who has a neurologic disease that affects the pyramidal tract is likely to manifest what sign? A. Impaired muscle movement. B. Decreased deep tendon reflexes. C. Decreased level of consciousness D. Impaired sensation of touch, pain, and temperature.

A. Impaired muscle movement. Among the most important descending tracts are the corticobulbar and corticospinal tracts, collectively termed the pyramidal tract. These tracts carry volitional (voluntary) impulses from the cortex to the cranial and peripheral nerves. Dysfunction of the pyramidal tract is likely to manifest as impaired movement because of hypertonicity. Diseases affecting the pyramidal tract do not result in changes in LOC, impaired reflexes, or decreased sensation.

A patient with recurrent shortness of breath has just had a bronchoscopy. What is a priority nursing action immediately following the procedure? A. Monitor the patient for laryngeal edema. B. Assess the patient's level of consciousness. C. Monitor and manage the patient's level of pain. D. Assess the patient's heart rate and blood pressure.

A. Monitor the patient for laryngeal edema. Priorities for assessment are the patient's airway and breathing, both of which may be compromised after bronchoscopy by laryngeal edema. These assessment parameters supersede the importance of loss of consciousness (LOC), pain, heart rate, and blood pressure, although the nurse should also be assessing these.

A hospitalized patient with a decreased glomerular filtration rate is scheduled to have an intravenous pyelogram (IVP). Which action will be included in the plan of care? A. Monitor the urine output after the procedure. B. Assist with monitored anesthesia care (MAC). C. Give oral contrast solution before the procedure. D. Insert a large size urinary catheter before the IVP.

A. Monitor the urine output after the procedure. Patients with impaired renal function are at risk for decreased renal function after IVP because the contrast medium used is nephrotoxic, so the nurse should monitor the patient's urine output. MAC sedation and retention catheterization are not required for the procedure. The contrast medium is given intravenously, not orally.

The patient with Parkinson's disease has a pulse oximetry reading of 72%, but he is not displaying any other signs of decreased oxygenation. What is most likely contributing to his low SpO2 level? A. Motion B. Anemia C. Dark skin color D. Thick acrylic nails

A. Motion Motion is the most likely cause of the low SpO2 for this patient with Parkinson's disease. Anemia, dark skin color, and thick acrylic nails as well as low perfusion, bright fluorescent lights, and intravascular dyes may also cause an inaccurate pulse oximetry result. There is no mention of these or reason to suspect these in this question.

A patient with heart failure and type 1 diabetes mellitus is scheduled for a positron emission tomography (PET) of the brain. Which medication prescribed by the health care provider should the nurse expect to administer before the diagnostic study? A. Regular insulin 6 units (SQ) B. Furosemide (Lasix) 20 mg (IV) C. Alprazolam (Xanax) 0.5 mg (PO) D. Ciprofloxacin (Cipro) 500 mg (PO)

A. Regular insulin 6 units (SQ) Patients with type 1 diabetes mellitus must have insulin administered the day of the PET study if glucose metabolism is the focus of the PET. Diuretics should not be administered before the PET scan unless a urinary catheter is inserted. The patient must remain still during the procedure (1 to 2 hours). Sedatives and tranquilizers (e.g., alprazolam) should not be administered before a PET study of the brain because the patient may need to perform mental activities and these medications may affect glucose metabolism. Prophylactic antibiotics are not necessary. Patients are NPO before a PET study of the brain and should not receive oral medications (alprazolam and ciprofloxacin).

The patient's arterial blood gas results show the PaO2 at 65 mmHg and the SaO2 at 80%. What early manifestations should the nurse expect to observe in this patient? A. Restlessness, tachypnea, tachycardia, and diaphoresis B. Unexplained confusion, dyspnea at rest, hypotension, and diaphoresis C. Combativeness, retractions with breathing, cyanosis, and decreased output D. Coma, accessory muscle use, cool and clammy skin, and unexplained fatigue

A. Restlessness, tachypnea, tachycardia, and diaphoresis With inadequate oxygenation, early manifestations include restlessness, tachypnea, tachycardia, and diaphoresis, decreased urinary output, and unexplained fatigue. The unexplained confusion, dyspnea at rest, hypotension, and diaphoresis; combativeness, retractions with breathing, cyanosis, and decreased urinary output; coma, accessory muscle use, cool and clammy skin, and unexplained fatigue occur as later manifestations of inadequate oxygenation.

A patient who is scheduled for an intravenous pyelogram (IVP) gives the nurse the following information. Which information has the most immediate implications for the patient's care? A. The patient describes allergies to shellfish and penicillin. B. The patient has not had anything to eat or drink for 8 hours. C. The patient complains of costovertebral angle (CVA) tenderness. Incorrect D. The patient used a bisacodyl (Dulcolax) tablet the previous night.

A. The patient describes allergies to shellfish and penicillin Iodine-based contrast dye is used during IVP & for many computed tomography (CT) scans. The nurse will need to notify the health care provider before the procedures so that the patient can receive medications such as antihistamines or corticosteroids before the procedures are started. The other information also is important to note & document but does not have immediate implications for the patient's care during the procedures.

The nurse is preparing the patient for a diagnostic procedure to remove pleural fluid for analysis. The nurse would prepare the patient for which test? A. Thoracentesis B. Bronchoscopy C. Pulmonary angiography D. Sputum culture and sensitivity

A. Thoracentesis Thoracentesis is the insertion of a large-bore needle through the chest wall into the pleural space to obtain specimens for diagnostic evaluation, remove pleural fluid, or instill medication into the pleural space.

During the assessment in the ED, the nurse is palpating the patient's chest. Which finding is a medical emergency? A. Trachea moved to the left B. Increased tactile fremitus C. Decreased tactile fremitus D. Diminished chest movement

A. Trachea moved to the left Tracheal deviation is a medical emergency when it is caused by a tension pneumothorax. Tactile fremitus increases with pneumonia or pulmonary edema and decreases in pleural effusion or lung hyperinflation. Diminished chest movement occurs with barrel chest, restrictive disease, and neuromuscular disease.

A patient with an elevated blood urea nitrogen (BUN) and serum creatinine is scheduled for a renal arteriogram. The nurse should question an order from radiology for bowel preparation with the use of A. a Fleet enema. B. a tap-water enema. C. bisacodyl (Dulcolax) tablets. Incorrect D. senna/docusate (Senokot-S).

A. a Fleet enema. High-phosphate enemas, such as Fleet enemas, should be avoided in patients with elevated BUN and creatinine because phosphate cannot be excreted by patients with renal failure. The other medications for bowel evacuation are more appropriate.

A student nurse asks the RN what can be measured by arterial blood gases (ABGs). The RN tells the student that the ABGs can measure (select all that apply). A. acid-base balance. B. oxygenation status. C. acidity of the blood. D. glucose bound to hemoglobin. E. bicarbonate (HCO3-) in arterial blood.

A. acid-base balance. B. oxygenation status. C. acidity of the blood. E. bicarbonate (HCO3-) in arterial blood. Arterial blood gases (ABGs) are measured to determine oxygenation status and acid-base balance. ABG analysis includes measurement of the PaO2, the partial pressure of carbon dioxide in arterial blood (PaCO2), acidity (pH), and bicarbonate (HCO3-) in arterial blood.

A patient's urine dipstick indicates a small amount of protein in the urine. The next action by the nurse should be to A. check which medications the patient is currently taking. B. obtain a clean-catch urine for culture and sensitivity testing. C. ask the patient about any family history of chronic renal failure. D. send a urine specimen to the laboratory to test for ketones and glucose.

A. check which medications the patient is currently taking. Normally the urinalysis will show zero to trace amounts of protein, but some medications may give false-positive readings. The other actions by the nurse may be appropriate, but checking for medications that may affect the dipstick accuracy should be done first.

A result of stimulation of the parasympathetic nervous system is (select all that apply): A. constriction of the bronchi. B. dilation of skin blood vessels. C. increased secretion of insulin. D. increased blood glucose levels. E. relaxation of the urinary sphincters.

A. constriction of the bronchi. B. dilation of skin blood vessels. C. increased secretion of insulin. E. relaxation of the urinary sphincters. Stimulation of the parasympathetic nervous system results in constriction of the bronchi, dilation of blood vessels to the skin, increased secretion of insulin, and relaxation of the urinary sphincter. Stimulation of the sympathetic nervous system results in increased blood glucose levels.

When assessing activity-exercise patterns related to respiratory health, the nurse inquires about: A. dyspnea during rest or exercise. B. recent weight loss or weight gain. C. ability to sleep through the entire night. D. willingness to wear oxygen equipment in public.

A. dyspnea during rest or exercise. In this functional health pattern, determine whether the patient's activity is limited by dyspnea at rest or during exercise.

During assessment of a patient with decreased renal function, which of these medications taken by the patient at home will be of most concern to the nurse? A. ibuprofen (Motrin) B. warfarin (Coumadin) C. folic acid (vitamin B9 ) D. penicillin (Bicillin LA)

A. ibuprofen (Motrin) The nonsteroidal anti-inflammatory medications (NSAIDs) are nephrotoxic and should be avoided in patients with impaired renal function. The nurse also should ask about reasons the patient is taking the other medications, but the medication of most concern is the ibuprofen.

Data regarding mobility, strength, coordination, and activity tolerance are important for the nurse to obtain because: A. many neurologic diseases affect one or more of these areas. B. patients are less able to identify other neurologic impairments. C. these are the first functions to be affected by neurologic diseases. D. aspects of movement are the most important function of the nervous system.

A. many neurologic diseases affect one or more of these areas. Many neurologic disorders affect the patient's mobility, strength, and coordination. These problems can alter the patient's usual activity and exercise patterns.

A patient's eyes jerk while the patient looks to the left. You will record this finding as: A. nystagmus. B. CN VI palsy. C. oculocephalia. D. opthalmic dyskinesia.

A. nystagmus. Nystagmus is defined as fine, rapid jerking movements of the eyes.

To promote the release of surfactant, the nurse encourages the patient to: A. take deep breaths. B. cough five times per hour to prevent alveolar collapse. C. decrease fluid intake to reduce fluid accumulation in the alveoli. D. sit with head of bed elevated to promote air movement through the pores of Kohn.

A. take deep breaths. Surfactant is a lipoprotein that lowers the surface tension in the alveoli. It reduces the amount of pressure needed to inflate the alveoli and decreases the tendency of the alveoli to collapse. Deep breaths stretch the alveoli and promote surfactant secretion.

The health care provider orders a clean-catch urine specimen for culture and sensitivity testing for a patient with a suspected urinary tract infection (UTI). To obtain the specimen, the nurse will plan to A. teach the patient to clean the urethral area, void a small amount into the toilet, and then void into a sterile specimen cup. B. have the patient empty the bladder completely, and then obtain the next urine specimen that the patient is able to void. C. insert a short, small "mini" catheter attached to a collecting container into the urethra and bladder to obtain the specimen. D. clean the area around the meatus with a povidone-iodine (Betadine) swab, and then have the patient void into a sterile container.

A. teach the patient to clean the urethral area, void a small amount into the toilet, and then void into a sterile specimen cup. This answer describes the technique for obtaining a clean-catch specimen. The answer beginning, "insert a short, small, 'mini' catheter attached to a collecting container" describes a technique that would result in a sterile specimen, but a health care provider's order for a catheterized specimen would be required. Using Betadine before obtaining the specimen is not necessary and might result in suppressing the growth of some bacteria. And the technique described in the answer beginning "have the patient empty the bladder completely" would not result in a sterile specimen.

A patient who is having an acute exacerbation of multiple sclerosis has a prescription for methylprednisolone (Solu-Medrol) 160 mg IV. The label on the vial reads: methylprednisolone 125 mg in 2 mL. How many mL will the nurse administer?

ANS: 2.56 With a concentration of 125 mg/2 mL, the nurse will need to administer 2.56 mL to obtain 160 mg of methylprednisolone. DIF: Cognitive Level: Understand (comprehension) REF: 1430-1431 TOP: Nursing Process: Implementation MSC: NCLEX: Physiological Integrity

1. A patient is to receive an infusion of 250 mL of platelets over 2 hours through tubing that is labeled: 1 mL equals 10 drops. How many drops per minute will the nurse infuse?

ANS: 21 To infuse 250 mL over 2 hours, the calculated drip rate is 20.8 drops/minute or 21 drops/minute.

Which action will the nurse include in the plan of care for a 72-year-old woman admitted with multiple myeloma? a. Monitor fluid intake and output. b. Administer calcium supplements. c. Assess lymph nodes for enlargement. d. Limit weight bearing and ambulation.

ANS: A A high fluid intake and urine output helps prevent the complications of kidney stones caused by hypercalcemia and renal failure caused by deposition of Bence-Jones protein in the renal tubules. Weight bearing and ambulation are encouraged to help bone retain calcium. Lymph nodes are not enlarged with multiple myeloma. Calcium supplements will further increase the patient's calcium level and are not used

24. Which action will the nurse include in the plan of care for a 72-year-old woman admitted with multiple myeloma? a. Monitor fluid intake and output. b. Administer calcium supplements. c. Assess lymph nodes for enlargement. d. Limit weight bearing and ambulation.

ANS: A A high fluid intake and urine output helps prevent the complications of kidney stones caused by hypercalcemia and renal failure caused by deposition of Bence-Jones protein in the renal tubules. Weight bearing and ambulation are encouraged to help bone retain calcium. Lymph nodes are not enlarged with multiple myeloma. Calcium supplements will further increase the patient's calcium level and are not used.

A 64-year-old patient who has amyotrophic lateral sclerosis (ALS) is hospitalized with pneumonia. Which nursing action will be included in the plan of care? a. Assist with active range of motion (ROM). b. Observe for agitation and paranoia. c. Give muscle relaxants as needed to reduce spasms. d. Use simple words and phrases to explain procedures.

ANS: A ALS causes progressive muscle weakness, but assisting the patient to perform active ROM will help maintain strength as long as possible. Psychotic manifestations such as agitation and paranoia are not associated with ALS. Cognitive function is not affected by ALS, and the patient's ability to understand procedures will not be impaired. Muscle relaxants will further increase muscle weakness and depress respirations. DIF: Cognitive Level: Apply (application) REF: 1439 TOP: Nursing Process: Planning MSC: NCLEX: Physiological Integrity

A hospitalized 31-year-old patient with a history of cluster headache awakens during the night with a severe stabbing headache. Which action should the nurse take first? a. Start the ordered PRN oxygen at 6 L/min. b. Put a moist hot pack on the patient's neck. c. Give the ordered PRN acetaminophen (Tylenol). d. Notify the patient's health care provider immediately.

ANS: A Acute treatment for cluster headache is administration of 100% oxygen at 6 to 8 L/min. If the patient obtains relief with the oxygen, there is no immediate need to notify the health care provider. Cluster headaches last only 60 to 90 minutes, so oral pain medications have minimal effect. Hot packs are helpful for tension headaches but are not as likely to reduce pain associated with a cluster headache. DIF: Cognitive Level: Apply (application) REF: 1417 OBJ: Special Questions: Prioritization TOP: Nursing Process: Implementation MSC: NCLEX: Physiological Integrity

After change-of-shift report, which patient should the nurse assess first? a. Patient with myasthenia gravis who is reporting increased muscle weakness b. Patient with a bilateral headache described as "like a band around my head" c. Patient with seizures who is scheduled to receive a dose of phenytoin (Dilantin) d. Patient with Parkinson's disease who has developed cogwheel rigidity of the arms

ANS: A Because increased muscle weakness may indicate the onset of a myasthenic crisis, the nurse should assess this patient first. The other patients should also be assessed, but do not appear to need immediate nursing assessments or actions to prevent life-threatening complications. DIF: Cognitive Level: Analyze (analysis) REF: 1438-1439 OBJ: Special Questions: Prioritization; Multiple Patients TOP: Nursing Process: Planning MSC: NCLEX: Safe and Effective Care Environment

25. An appropriate nursing intervention for a patient with non-Hodgkin's lymphoma whose platelet count drops to 18,000/µL during chemotherapy is to a. check all stools for occult blood. b. encourage fluids to 3000 mL/day. c. provide oral hygiene every 2 hours. d. check the temperature every 4 hours.

ANS: A Because the patient is at risk for spontaneous bleeding, the nurse should check stools for occult blood. A low platelet count does not require an increased fluid intake. Oral hygiene is important, but it is not necessary to provide oral care every 2 hours. The low platelet count does not increase risk for infection, so frequent temperature monitoring is not indicated.

An appropriate nursing intervention for a patient with non-Hodgkin's lymphoma whose platelet count drops to 18,000/µL during chemotherapy is to a. check all stools for occult blood. b. encourage fluids to 3000 mL/day. c. provide oral hygiene every 2 hours. d. check the temperature every 4 hours.

ANS: A Because the patient is at risk for spontaneous bleeding, the nurse should check stools for occult blood. A low platelet count does not require an increased fluid intake. Oral hygiene is important, but it is not necessary to provide oral care every 2 hours. The low platelet count does not increase risk for infection, so frequent temperature monitoring is not indicated.

8. A 31-year-old woman who has multiple sclerosis (MS) asks the nurse about risks associated with pregnancy. Which response by the nurse is accurate? a. "MS symptoms may be worse after the pregnancy." b. "Women with MS frequently have premature labor." c. "MS is associated with an increased risk for congenital defects." d. "Symptoms of MS are likely to become worse during pregnancy."

ANS: A During the postpartum period, women with MS are at greater risk for exacerbation of symptoms. There is no increased risk for congenital defects in infants born of mothers with MS. Symptoms of MS may improve during pregnancy. Onset of labor is not affected by MS. DIF: Cognitive Level: Understand (comprehension) REF: 1429 TOP: Nursing Process: Implementation MSC: NCLEX: Health Promotion and Maintenance

2. Which menu choice indicates that the patient understands the nurse's teaching about best dietary choices for iron-deficiency anemia? a. Omelet and whole wheat toast b. Cantaloupe and cottage cheese c. Strawberry and banana fruit plate d. Cornmeal muffin and orange juice

ANS: A Eggs and whole grain breads are high in iron. The other choices are appropriate for other nutritional deficiencies but are not the best choice for a patient with iron-deficiency anemia.

Which menu choice indicates that the patient understands the nurse's teaching about best dietary choices for iron-deficiency anemia? a. Omelet and whole wheat toast b. Cantaloupe and cottage cheese c. Strawberry and banana fruit plate d. Cornmeal muffin and orange juice

ANS: A Eggs and whole grain breads are high in iron. The other choices are appropriate for other nutritional deficiencies but are not the best choice for a patient with iron-deficiency anemia.

Which information obtained by the nurse assessing a patient admitted with multiple myeloma is most important to report to the health care provider? a. Serum calcium level is 15 mg/dL. b. Patient reports no stool for 5 days. c. Urine sample has Bence-Jones protein. d. Patient is complaining of severe back pain.

ANS: A Hypercalcemia may lead to complications such as dysrhythmias or seizures, and should be addressed quickly. The other patient findings will also be discussed with the health care provider, but are not life threatening

45. Which information obtained by the nurse assessing a patient admitted with multiple myeloma is most important to report to the health care provider? a. Serum calcium level is 15 mg/dL. b. Patient reports no stool for 5 days. c. Urine sample has Bence-Jones protein. d. Patient is complaining of severe back pain.

ANS: A Hypercalcemia may lead to complications such as dysrhythmias or seizures, and should be addressed quickly. The other patient findings will also be discussed with the health care provider, but are not life threatening.

6. Which patient statement to the nurse indicates a need for additional instruction about taking oral ferrous sulfate? a. "I will call my health care provider if my stools turn black." b. "I will take a stool softener if I feel constipated occasionally." c. "I should take the iron with orange juice about an hour before eating." d. "I should increase my fluid and fiber intake while I am taking iron tablets."

ANS: A It is normal for the stools to appear black when a patient is taking iron, and the patient should not call the doctor about this. The other patient statements are correct.

Which patient statement to the nurse indicates a need for additional instruction about taking oral ferrous sulfate? a. "I will call my health care provider if my stools turn black." b. "I will take a stool softener if I feel constipated occasionally." c. "I should take the iron with orange juice about an hour before eating." d. "I should increase my fluid and fiber intake while I am taking iron tablets."

ANS: A It is normal for the stools to appear black when a patient is taking iron, and the patient should not call the doctor about this. The other patient statements are correct.

he nurse advises a patient with myasthenia gravis (MG) to a. perform physically demanding activities early in the day. b. anticipate the need for weekly plasmapheresis treatments. c. do frequent weight-bearing exercise to prevent muscle atrophy. d. protect the extremities from injury due to poor sensory perception.

ANS: A Muscles are generally strongest in the morning, and activities involving muscle activity should be scheduled then. Plasmapheresis is not routinely scheduled, but is used for myasthenia crisis or for situations in which corticosteroid therapy must be avoided. There is no decrease in sensation with MG, and muscle atrophy does not occur because although there is muscle weakness, they are still used. DIF: Cognitive Level: Apply (application) REF: 1437 TOP: Nursing Process: Implementation MSC: NCLEX: Physiological Integrity

A 68-year-old male patient who has bladder cancer had a cystectomy with creation of an Indiana pouch. Which topic will be included in patient teaching?

Catheterization technique and schedule

31. Which patient should the nurse assign as the roommate for a patient who has aplastic anemia? a. A patient with chronic heart failure b. A patient who has viral pneumonia c. A patient who has right leg cellulitis d. A patient with multiple abdominal drains

ANS: A Patients with aplastic anemia are at risk for infection because of the low white blood cell production associated with this type of anemia, so the nurse should avoid assigning a roommate with any possible infectious process.

Which patient should the nurse assign as the roommate for a patient who has aplastic anemia? a. A patient with chronic heart failure b. A patient who has viral pneumonia c. A patient who has right leg cellulitis d. A patient with multiple abdominal drains

ANS: A Patients with aplastic anemia are at risk for infection because of the low white blood cell production associated with this type of anemia, so the nurse should avoid assigning a roommate with any possible infectious process.

A patient has been taking phenytoin (Dilantin) for 2 years. Which action will the nurse take when evaluating for adverse effects of the medication? a. Inspect the oral mucosa. b. Listen to the lung sounds. c. Auscultate the bowel tones. d. Check pupil reaction to light.

ANS: A Phenytoin can cause gingival hyperplasia, but does not affect bowel tones, lung sounds, or pupil reaction to light. DIF: Cognitive Level: Apply (application) REF: 1424 TOP: Nursing Process: Evaluation MSC: NCLEX: Physiological Integrity

33. A 19-year-old woman with immune thrombocytopenic purpura (ITP) has an order for a platelet transfusion. Which information indicates that the nurse should consult with the health care provider before obtaining and administering platelets? a. The platelet count is 42,000/L. b. Petechiae are present on the chest. c. Blood pressure (BP) is 94/56 mm Hg. d. Blood is oozing from the venipuncture site.

ANS: A Platelet transfusions are not usually indicated until the platelet count is below 10,000 to 20,000/L unless the patient is actively bleeding. Therefore the nurse should clarify the order with the health care provider before giving the transfusion. The other data all indicate that bleeding caused by ITP may be occurring and that the platelet transfusion is appropriate.

A 19-year-old woman with immune thrombocytopenic purpura (ITP) has an order for a platelet transfusion. Which information indicates that the nurse should consult with the health care provider before obtaining and administering platelets? a. The platelet count is 42,000/L. b. Petechiae are present on the chest. c. Blood pressure (BP) is 94/56 mm Hg. d. Blood is oozing from the venipuncture site.

ANS: A Platelet transfusions are not usually indicated until the platelet count is below 10,000 to 20,000/L unless the patient is actively bleeding. Therefore the nurse should clarify the order with the health care provider before giving the transfusion. The other data all indicate that bleeding caused by ITP may be occurring and that the platelet transfusion is appropriate.

Which syndrome of incomplete spinal cord lesion is described as cord damage common in the cervical region resulting in greater weakness in upper extremities than lower?

Central cord syndrome

A 46-year-old patient tells the nurse about using acetaminophen (Tylenol) several times every day for recurrent bilateral headaches. Which action will the nurse plan to take first? a. Discuss the need to stop taking the acetaminophen. b. Suggest the use of biofeedback for headache control. c. Describe the use of botulism toxin (Botox) for headaches. d. Teach the patient about magnetic resonance imaging (MRI).

ANS: A The headache description suggests that the patient is experiencing medication overuse headache. The initial action will be withdrawal of the medication. The other actions may be needed if the headaches persist. DIF: Cognitive Level: Apply (application) REF: 1418 OBJ: Special Questions: Prioritization TOP: Nursing Process: Planning MSC: NCLEX: Physiological Integrity

16. The nurse caring for a patient with type A hemophilia being admitted to the hospital with severe pain and swelling in the right knee will a. immobilize the joint. b. apply heat to the knee. c. assist the patient with light weight bearing. d. perform passive range of motion to the knee.

ANS: A The initial action should be total rest of the knee to minimize bleeding. Ice packs are used to decrease bleeding. Range of motion (ROM) and weight-bearing exercise are contraindicated initially, but after the bleeding stops, ROM and physical therapy are started.

Gastric lavage and administration of activated charcoal are ordered for an unconscious patient who has been admitted to the ED after ingesting 30 lorazepam (Ativan) tablets. Which action should the nurse plan to do first?

Assist with intubation of the patient

The nurse caring for a patient with type A hemophilia being admitted to the hospital with severe pain and swelling in the right knee will a. immobilize the joint. b. apply heat to the knee. c. assist the patient with light weight bearing. d. perform passive range of motion to the knee.

ANS: A The initial action should be total rest of the knee to minimize bleeding. Ice packs are used to decrease bleeding. Range of motion (ROM) and weight-bearing exercise are contraindicated initially, but after the bleeding stops, ROM and physical therapy are started.

A patient reports feeling numbness and tingling of the left arm before experiencing a tonic-clonic seizure. The nurse determines that this history is consistent with what type of seizure? a. Focal b. Atonic c. Absence d. Myoclonic

ANS: A The initial symptoms of a focal seizure involve clinical manifestations that are localized to a particular part of the body or brain. Symptoms of an absence seizure are staring and a brief loss of consciousness. In an atonic seizure, the patient loses muscle tone and (typically) falls to the ground. Myoclonic seizures are characterized by a sudden jerk of the body or extremities. DIF: Cognitive Level: Understand (comprehension) REF: 1421 TOP: Nursing Process: Assessment MSC: NCLEX: Physiological Integrity

47. The nurse has obtained the health history, physical assessment data, and laboratory results shown in the accompanying figure for a patient admitted with aplastic anemia. Which information is most important to communicate to the health care provider? a. Neutropenia b. Increasing fatigue c. Thrombocytopenia d. Frequent constipation

ANS: A The low white blood cell count indicates that the patient is at high risk for infection and needs immediate actions to diagnose and treat the cause of the leucopenia. The other information may require further assessment or treatment, but does not place the patient at immediate risk for complications.

The nurse observes a patient ambulating in the hospital hall when the patient's arms and legs suddenly jerk and the patient falls to the floor. The nurse will first a. assess the patient for a possible head injury. b. give the scheduled dose of divalproex (Depakote). c. document the timing and description of the seizure. d. notify the patient's health care provider about the seizure.

ANS: A The patient who has had a myoclonic seizure and fall is at risk for head injury and should first be evaluated and treated for this possible complication. Documentation of the seizure, notification of the seizure, and administration of antiseizure medications are also appropriate actions, but the initial action should be assessment for injury. DIF: Cognitive Level: Apply (application) REF: 1423 OBJ: Special Questions: Prioritization TOP: Nursing Process: Implementation MSC: NCLEX: Physiological Integrity

Which action will the nurse need to do when preparing to assist with the insertion of a pulmonary artery catheter?

Attach cardiac monitoring leads before the procedure.

Following a laryngectomy a patient coughs violently during suctioning and dislodges the tracheostomy tube. Which action should the nurse take first?

Attempt to reinsert the tracheostomy tube with the obturator in place.

A patient with Parkinson's disease is admitted to the hospital for treatment of pneumonia. Which nursing interventions will be included in the plan of care (select all that apply)? a. Use an elevated toilet seat. b. Cut patient's food into small pieces. c. Provide high-protein foods at each meal. d. Place an armchair at the patient's bedside. e. Observe for sudden exacerbation of symptoms.

ANS: A, B, D Because the patient with Parkinson's has difficulty chewing, food should be cut into small pieces. An armchair should be used when the patient is seated so that the patient can use the arms to assist with getting up from the chair. An elevated toilet seat will facilitate getting on and off the toilet. High-protein foods will decrease the effectiveness of L-dopa. Parkinson's is a steadily progressive disease without acute exacerbations. DIF: Cognitive Level: Apply (application) REF: 1436-1437 TOP: Nursing Process: Planning MSC: NCLEX: Physiological Integrity

Which information about a patient with Goodpasture syndrome requires the most rapid action by the nurse?

Audible crackles bilaterally over the posterior chest to the midscapular level.

A 27-year-old patient who has been treated for status epilepticus in the emergency department will be transferred to the medical nursing unit. Which equipment should the nurse have available in the patient's assigned room (select all that apply)? a. Side-rail pads b. Tongue blade c. Oxygen mask d. Suction tubing e. Urinary catheter f. Nasogastric tube

ANS: A, C, D The patient is at risk for further seizures, and oxygen and suctioning may be needed after any seizures to clear the airway and maximize oxygenation. The bed's side rails should be padded to minimize the risk for patient injury during a seizure. Use of tongue blades during a seizure is contraindicated. Insertion of a nasogastric (NG) tube is not indicated because the airway problem is not caused by vomiting or abdominal distention. A urinary catheter is not required unless there is urinary retention .DIF: Cognitive Level: Apply (application) REF: 1426 TOP: Nursing Process: Planning MSC: NCLEX: Physiological Integrity

32. Which patient requires the most rapid assessment and care by the emergency department nurse? a. The patient with hemochromatosis who reports abdominal pain b. The patient with neutropenia who has a temperature of 101.8° F c. The patient with sickle cell anemia who has had nausea and diarrhea for 24 hours d. The patient with thrombocytopenia who has oozing after having a tooth extracted

ANS: B A neutropenic patient with a fever is assumed to have an infection and is at risk for rapidly developing sepsis. Rapid assessment, cultures, and initiation of antibiotic therapy are needed. The other patients also require rapid assessment and care but not as urgently as the neutropenic patient.

Which patient requires the most rapid assessment and care by the emergency department nurse? a. The patient with hemochromatosis who reports abdominal pain b. The patient with neutropenia who has a temperature of 101.8° F c. The patient with sickle cell anemia who has had nausea and diarrhea for 24 hours d. The patient with thrombocytopenia who has oozing after having a tooth extracted

ANS: B A neutropenic patient with a fever is assumed to have an infection and is at risk for rapidly developing sepsis. Rapid assessment, cultures, and initiation of antibiotic therapy are needed. The other patients also require rapid assessment and care but not as urgently as the neutropenic patient.

39. After receiving change-of-shift report for several patients with neutropenia, which patient should the nurse assess first? a. 56-year-old with frequent explosive diarrhea b. 33-year-old with a fever of 100.8° F (38.2° C) c. 66-year-old who has white pharyngeal lesions d. 23-year old who is complaining of severe fatigue

ANS: B Any fever in a neutropenic patient indicates infection and can quickly lead to sepsis and septic shock. Rapid assessment and (if prescribed) initiation of antibiotic therapy within 1 hour are needed. The other patients also need to be assessed but do not exhibit symptoms of potentially life-threatening problems.

After receiving change-of-shift report for several patients with neutropenia, which patient should the nurse assess first? a. 56-year-old with frequent explosive diarrhea b. 33-year-old with a fever of 100.8° F (38.2° C) c. 66-year-old who has white pharyngeal lesions d. 23-year old who is complaining of severe fatigue

ANS: B Any fever in a neutropenic patient indicates infection and can quickly lead to sepsis and septic shock. Rapid assessment and (if prescribed) initiation of antibiotic therapy within 1 hour are needed. The other patients also need to be assessed but do not exhibit symptoms of potentially life-threatening problems.

A 22 year old patient who experienced a near downing accident in a local pool, but now is awake and breathing spontaneously, is admitted for observation. Which assessment will be most important for the nurse to take during the observation period?

Auscultate breath sounds

Which collaborative problem will the nurse include in a care plan for a patient admitted to the hospital with idiopathic aplastic anemia? a. Potential complication: seizures b. Potential complication: infection c. Potential complication: neurogenic shock d. Potential complication: pulmonary edema

ANS: B Because the patient with aplastic anemia has pancytopenia, the patient is at risk for infection and bleeding. There is no increased risk for seizures, neurogenic shock, or pulmonary edema

7. Which collaborative problem will the nurse include in a care plan for a patient admitted to the hospital with idiopathic aplastic anemia? a. Potential complication: seizures b. Potential complication: infection c. Potential complication: neurogenic shock d. Potential complication: pulmonary edema

ANS: B Because the patient with aplastic anemia has pancytopenia, the patient is at risk for infection and bleeding. There is no increased risk for seizures, neurogenic shock, or pulmonary edema.

18. A routine complete blood count indicates that an active 80-year-old man may have myelodysplastic syndrome. The nurse will plan to teach the patient about a. blood transfusion b. bone marrow biopsy. c. filgrastim (Neupogen) administration. d. erythropoietin (Epogen) administration.

ANS: B Bone marrow biopsy is needed to make the diagnosis and determine the specific type of myelodysplastic syndrome. The other treatments may be necessary if there is progression of the myelodysplastic syndrome, but the initial action for this asymptomatic patient will be a bone marrow biopsy.

A routine complete blood count indicates that an active 80-year-old man may have myelodysplastic syndrome. The nurse will plan to teach the patient about a. blood transfusion b. bone marrow biopsy. c. filgrastim (Neupogen) administration. d. erythropoietin (Epogen) administration.

ANS: B Bone marrow biopsy is needed to make the diagnosis and determine the specific type of myelodysplastic syndrome. The other treatments may be necessary if there is progression of the myelodysplastic syndrome, but the initial action for this asymptomatic patient will be a bone marrow biopsy.

A patient who was admitted the previous day with pneumonia complains of a sharp pain of 7 (based on 0 to 10 scale) whenever I take a deep breath. Which action will the nurse take next?

Auscultate breath sounds.

27. Which information obtained by the nurse caring for a patient with thrombocytopenia should be immediately communicated to the health care provider? a. The platelet count is 52,000/µL. b. The patient is difficult to arouse. c. There are purpura on the oral mucosa. d. There are large bruises on the patient's back.

ANS: B Difficulty in arousing the patient may indicate a cerebral hemorrhage, which is life threatening and requires immediate action. The other information should be documented and reported but would not be unusual in a patient with thrombocytopenia.

Which information obtained by the nurse caring for a patient with thrombocytopenia should be immediately communicated to the health care provider? a. The platelet count is 52,000/µL. b. The patient is difficult to arouse. c. There are purpura on the oral mucosa. d. There are large bruises on the patient's back.

ANS: B Difficulty in arousing the patient may indicate a cerebral hemorrhage, which is life threatening and requires immediate action. The other information should be documented and reported but would not be unusual in a patient with thrombocytopenia.

14. Which intervention will be included in the nursing care plan for a patient with immune thrombocytopenic purpura (ITP)? a. Assign the patient to a private room. b. Avoid intramuscular (IM) injections. c. Use rinses rather than a soft toothbrush for oral care. d. Restrict activity to passive and active range of motion.

ANS: B IM or subcutaneous injections should be avoided because of the risk for bleeding. A soft toothbrush can be used for oral care. There is no need to restrict activity or place the patient in a private room.

Which intervention will be included in the nursing care plan for a patient with immune thrombocytopenic purpura (ITP)? a. Assign the patient to a private room. b. Avoid intramuscular (IM) injections. c. Use rinses rather than a soft toothbrush for oral care. d. Restrict activity to passive and active range of motion.

ANS: B IM or subcutaneous injections should be avoided because of the risk for bleeding. A soft toothbrush can be used for oral care. There is no need to restrict activity or place the patient in a private room.

11. The nurse notes scleral jaundice in a patient being admitted with hemolytic anemia. The nurse will plan to check the laboratory results for the a. Schilling test. b. bilirubin level. c. stool occult blood test. d. gastric analysis testing.

ANS: B Jaundice is caused by the elevation of bilirubin level associated with red blood cell (RBC) hemolysis. The other tests would not be helpful in monitoring or treating a hemolytic anemia.

The nurse notes scleral jaundice in a patient being admitted with hemolytic anemia. The nurse will plan to check the laboratory results for the a. Schilling test. b. bilirubin level. c. stool occult blood test. d. gastric analysis testing.

ANS: B Jaundice is caused by the elevation of bilirubin level associated with red blood cell (RBC) hemolysis. The other tests would not be helpful in monitoring or treating a hemolytic anemia.

The home health registered nurse (RN) is planning care for a patient with a seizure disorder related to a recent head injury. Which nursing action can be delegated to a licensed practical/vocational nurse (LPN/LVN)? a. Make referrals to appropriate community agencies. b. Place medications in the home medication organizer. c. Teach the patient and family how to manage seizures. d. Assess for use of medications that may precipitate seizures.

ANS: B LPN/LVN education includes administration of medications. The other activities require RN education and scope of practice. DIF: Cognitive Level: Apply (application) REF: 1426 OBJ: Special Questions: Delegation TOP: Nursing Process: Planning MSC: NCLEX: Safe and Effective Care Environment

When caring for a patient with a left arm arteriovenous fistula, which action will the nurse include in the plan of care to maintain the patency of the fistula?

Auscultate for a bruit at the fistula site.

34. Which problem reported by a patient with hemophilia is most important for the nurse to communicate to the physician? a. Leg bruises b. Tarry stools c. Skin abrasions d. Bleeding gums

ANS: B Melena is a sign of gastrointestinal bleeding and requires collaborative actions such as checking hemoglobin and hematocrit and administration of coagulation factors. The other problems indicate a need for patient teaching about how to avoid injury, but are not indicators of possible serious blood loss

Which problem reported by a patient with hemophilia is most important for the nurse to communicate to the physician? a. Leg bruises b. Tarry stools c. Skin abrasions d. Bleeding gums

ANS: B Melena is a sign of gastrointestinal bleeding and requires collaborative actions such as checking hemoglobin and hematocrit and administration of coagulation factors. The other problems indicate a need for patient teaching about how to avoid injury, but are not indicators of possible serious blood loss.

3. A patient who is receiving methotrexate for severe rheumatoid arthritis develops a megaloblastic anemia. The nurse will anticipate teaching the patient about increasing oral intake of a. iron. b. folic acid. c. cobalamin (vitamin B12). d. ascorbic acid (vitamin C).

ANS: B Methotrexate use can lead to folic acid deficiency. Supplementation with oral folic acid supplements is the usual treatment. The other nutrients would not correct folic acid deficiency, although they would be used to treat other types of anemia.

A patient who is receiving methotrexate for severe rheumatoid arthritis develops a megaloblastic anemia. The nurse will anticipate teaching the patient about increasing oral intake of a. iron. b. folic acid. c. cobalamin (vitamin B12). d. ascorbic acid (vitamin C).

ANS: B Methotrexate use can lead to folic acid deficiency. Supplementation with oral folic acid supplements is the usual treatment. The other nutrients would not correct folic acid deficiency, although they would be used to treat other types of anemia.

Which intervention will the nurse include in the plan of care for a patient with primary restless legs syndrome (RLS) who is having difficulty sleeping? a. Teach about the use of antihistamines to improve sleep. b. Suggest that the patient exercise regularly during the day. c. Make a referral to a massage therapist for deep massage of the legs. d. Assure the patient that the problem is transient and likely to resolve.

ANS: B Nondrug interventions such as getting regular exercise are initially suggested to improve sleep quality in patients with RLS. Antihistamines may aggravate RLS. Massage does not alleviate RLS symptoms and RLS is likely to progress in most patients. DIF: Cognitive Level: Apply (application) REF: 1427 TOP: Nursing Process: Planning MSC: NCLEX: Physiological Integrity

Two days after an acute myocardial infarction (MI), a patient complains of stabbing chest pain that increases with a deep breath. Which action will the nurse take first?

Auscultate the heart sounds.

5. An appropriate nursing intervention for a hospitalized patient with severe hemolytic anemia is to a. provide a diet high in vitamin K. b. alternate periods of rest and activity. c. teach the patient how to avoid injury. d. place the patient on protective isolation.

ANS: B Nursing care for patients with anemia should alternate periods of rest and activity to encourage activity without causing undue fatigue. There is no indication that the patient has a bleeding disorder, so a diet high in vitamin K or teaching about how to avoid injury is not needed. Protective isolation might be used for a patient with aplastic anemia, but it is not indicated for hemolytic anemia.

An appropriate nursing intervention for a hospitalized patient with severe hemolytic anemia is to a. provide a diet high in vitamin K. b. alternate periods of rest and activity. c. teach the patient how to avoid injury. d. place the patient on protective isolation.

ANS: B Nursing care for patients with anemia should alternate periods of rest and activity to encourage activity without causing undue fatigue. There is no indication that the patient has a bleeding disorder, so a diet high in vitamin K or teaching about how to avoid injury is not needed. Protective isolation might be used for a patient with aplastic anemia, but it is not indicated for hemolytic anemia.

8. It is important for the nurse providing care for a patient with sickle cell crisis to a. limit the patient's intake of oral and IV fluids. b. evaluate the effectiveness of opioid analgesics. c. encourage the patient to ambulate as much as tolerated. d. teach the patient about high-protein, high-calorie foods.

ANS: B Pain is the most common clinical manifestation of a crisis and usually requires large doses of continuous opioids for control. Fluid intake should be increased to reduce blood viscosity and improve perfusion. Rest is usually ordered to decrease metabolic requirements. Patients are instructed about the need for dietary folic acid, but high-protein, high-calorie diets are not emphasized.

It is important for the nurse providing care for a patient with sickle cell crisis to a. limit the patient's intake of oral and IV fluids. b. evaluate the effectiveness of opioid analgesics. c. encourage the patient to ambulate as much as tolerated. d. teach the patient about high-protein, high-calorie foods.

ANS: B Pain is the most common clinical manifestation of a crisis and usually requires large doses of continuous opioids for control. Fluid intake should be increased to reduce blood viscosity and improve perfusion. Rest is usually ordered to decrease metabolic requirements. Patients are instructed about the need for dietary folic acid, but high-protein, high-calorie diets are not emphasized.

A 73-year-old patient with Parkinson's disease has a nursing diagnosis of impaired physical mobility related to bradykinesia. Which action will the nurse include in the plan of care? a. Instruct the patient in activities that can be done while lying or sitting. b. Suggest that the patient rock from side to side to initiate leg movement. c. Have the patient take small steps in a straight line directly in front of the feet. d. Teach the patient to keep the feet in contact with the floor and slide them forward.

ANS: B Rocking the body from side to side stimulates balance and improves mobility. The patient will be encouraged to continue exercising because this will maintain functional abilities. Maintaining a wide base of support will help with balance. The patient should lift the feet and avoid a shuffling gait. DIF: Cognitive Level: Apply (application) REF: 1437 TOP: Nursing Process: Planning MSC: NCLEX: Physiological Integrity

Which action should the nurse take first when a patient complains of acute chest pain and dyspnea soon after insertion of a centrally inserted IV catheter?

Auscultate the patients breath sounds.

22. A patient with a history of a transfusion-related acute lung injury (TRALI) is to receive a transfusion of packed red blood cells (PRBCs). Which action by the nurse will decrease the risk for TRALI for this patient? a. Infuse the PRBCs slowly over 4 hours. b. Transfuse only leukocyte-reduced PRBCs. c. Administer the scheduled diuretic before the transfusion. d. Give the PRN dose of antihistamine before the transfusion.

ANS: B TRALI is caused by a reaction between the donor and the patient leukocytes that causes pulmonary inflammation and capillary leaking. The other actions may help prevent respiratory problems caused by circulatory overload or by allergic reactions, but they will not prevent TRALI.

A patient with a history of a transfusion-related acute lung injury (TRALI) is to receive a transfusion of packed red blood cells (PRBCs). Which action by the nurse will decrease the risk for TRALI for this patient? a. Infuse the PRBCs slowly over 4 hours. b. Transfuse only leukocyte-reduced PRBCs. c. Administer the scheduled diuretic before the transfusion. d. Give the PRN dose of antihistamine before the transfusion.

ANS: B TRALI is caused by a reaction between the donor and the patient leukocytes that causes pulmonary inflammation and capillary leaking. The other actions may help prevent respiratory problems caused by circulatory overload or by allergic reactions, but they will not prevent TRALI.

Which action will the nurse plan to take for a 40-year-old patient with multiple sclerosis (MS) who has urinary retention caused by a flaccid bladder? a. Decrease the patient's evening fluid intake. b. Teach the patient how to use the Credé method. c. Suggest the use of adult incontinence briefs for nighttime only. d. Assist the patient to the commode every 2 hours during the day.

ANS: B The Credé method can be used to improve bladder emptying. Decreasing fluid intake will not improve bladder emptying and may increase risk for urinary tract infection (UTI) and dehydration. The use of incontinence briefs and frequent toileting will not improve bladder emptying. DIF: Cognitive Level: Apply (application) REF: eNCP 59-3 TOP: Nursing Process: Planning MSC: NCLEX: Physiological Integrity

17. A 28-year-old man with von Willebrand disease is admitted to the hospital for minor knee surgery. The nurse will review the coagulation survey to check the a. platelet count. b. bleeding time. c. thrombin time. d. prothrombin time.

ANS: B The bleeding time is affected by von Willebrand disease. Platelet count, prothrombin time, and thrombin time are normal in von Willebrand disease.

A 28-year-old man with von Willebrand disease is admitted to the hospital for minor knee surgery. The nurse will review the coagulation survey to check the a. platelet count. b. bleeding time. c. thrombin time. d. prothrombin time.

ANS: B The bleeding time is affected by von Willebrand disease. Platelet count, prothrombin time, and thrombin time are normal in von Willebrand disease.

When a 74-year-old patient is seen in the health clinic with new development of a stooped posture, shuffling gait, and pill rolling-type tremor, the nurse will anticipate teaching the patient about a. oral corticosteroids. b. antiparkinsonian drugs. c. magnetic resonance imaging (MRI). d. electroencephalogram (EEG) testing.

ANS: B The diagnosis of Parkinson's is made when two of the three characteristic manifestations of tremor, rigidity, and bradykinesia are present. The confirmation of the diagnosis is made on the basis of improvement when antiparkinsonian drugs are administered. This patient has symptoms of tremor and bradykinesia. The next anticipated step will be treatment with medications. MRI and EEG are not useful in diagnosing Parkinson's disease, and corticosteroid therapy is not used to treat it. DIF: Cognitive Level: Apply (application) REF: 1434 TOP: Nursing Process: Planning MSC: NCLEX: Physiological Integrity

19. Which action will the admitting nurse include in the care plan for a 30-year old woman who is neutropenic? a. Avoid any injections. b. Check temperature every 4 hours. c. Omit fruits or vegetables from the diet. d. Place a "No Visitors" sign on the door.

ANS: B The earliest sign of infection in a neutropenic patient is an elevation in temperature. Although unpeeled fresh fruits and vegetables should be avoided, fruits and vegetables that are peeled or cooked are acceptable. Injections may be required for administration of medications such as filgrastim (Neupogen). The number of visitors may be limited and visitors with communicable diseases should be avoided, but a "no visitors" policy is not needed.

Which action will the admitting nurse include in the care plan for a 30-year old woman who is neutropenic? a. Avoid any injections. b. Check temperature every 4 hours. c. Omit fruits or vegetables from the diet. d. Place a "No Visitors" sign on the door.

ANS: B The earliest sign of infection in a neutropenic patient is an elevation in temperature. Although unpeeled fresh fruits and vegetables should be avoided, fruits and vegetables that are peeled or cooked are acceptable. Injections may be required for administration of medications such as filgrastim (Neupogen). The number of visitors may be limited and visitors with communicable diseases should be avoided, but a "no visitors" policy is not needed.

A nurse is planning preoperative care for a client who has pheochromocytoma. Which of the following interventions should the nurse anticipate as being the priority? A. Use same arm for BP measurement B. Avoid palpating the abdomen C. Manage headaches with analgesics D. Provide a private, darkened room

Avoid palpating the abdomen

A high school teacher who has just been diagnosed with epilepsy after having a generalized tonic-clonic seizure tells the nurse, "I cannot teach anymore, it will be too upsetting if I have a seizure at work." Which response by the nurse is best? a. "You might benefit from some psychologic counseling." b. "Epilepsy usually can be well controlled with medications." c. "You will want to contact the Epilepsy Foundation for assistance." d. "The Department of Vocational Rehabilitation can help with work retraining."

ANS: B The nurse should inform the patient that most patients with seizure disorders are controlled with medication. The other information may be necessary if the seizures persist after treatment with antiseizure medications is implemented. DIF: Cognitive Level: Apply (application) REF: 1422 TOP: Nursing Process: Implementation MSC: NCLEX: Psychosocial Integrity

26. A 30-year-old man with acute myelogenous leukemia develops an absolute neutrophil count of 850/µL while receiving outpatient chemotherapy. Which action by the outpatient clinic nurse is most appropriate? a. Discuss the need for hospital admission to treat the neutropenia. b. Teach the patient to administer filgrastim (Neupogen) injections. c. Plan to discontinue the chemotherapy until the neutropenia resolves. d. Order a high-efficiency particulate air (HEPA) filter for the patient's home.

ANS: B The patient may be taught to self-administer filgrastim injections. Although chemotherapy may be stopped with severe neutropenia (neutrophil count less than 500/µL), administration of filgrastim usually allows the chemotherapy to continue. Patients with neutropenia are at higher risk for infection when exposed to other patients in the hospital. HEPA filters are expensive and are used in the hospital, where the number of pathogens is much higher than in the patient's home environment.

A 30-year-old man with acute myelogenous leukemia develops an absolute neutrophil count of 850/µL while receiving outpatient chemotherapy. Which action by the outpatient clinic nurse is most appropriate? a. Discuss the need for hospital admission to treat the neutropenia. b. Teach the patient to administer filgrastim (Neupogen) injections. c. Plan to discontinue the chemotherapy until the neutropenia resolves. d. Order a high-efficiency particulate air (HEPA) filter for the patient's home.

ANS: B The patient may be taught to self-administer filgrastim injections. Although chemotherapy may be stopped with severe neutropenia (neutrophil count less than 500/µL), administration of filgrastim usually allows the chemotherapy to continue. Patients with neutropenia are at higher risk for infection when exposed to other patients in the hospital. HEPA filters are expensive and are used in the hospital, where the number of pathogens is much higher than in the patient's home environment.

Several patients call the outpatient clinic and ask to make an appointment as soon as possible. Which patient should the nurse schedule to be seen first? a. 44-year-old with sickle cell anemia who says "my eyes always look sort of yellow" b. 23-year-old with no previous health problems who has a nontender lump in the axilla c. 50-year-old with early-stage chronic lymphocytic leukemia who reports chronic fatigue d. 19-year-old with hemophilia who wants to learn to self-administer factor VII replacement

ANS: B The patient's age and presence of a nontender axillary lump suggest possible lymphoma, which needs rapid diagnosis and treatment. The other patients have questions about treatment or symptoms that are consistent with their diagnosis but do not need to be seen urgently

38. Several patients call the outpatient clinic and ask to make an appointment as soon as possible. Which patient should the nurse schedule to be seen first? a. 44-year-old with sickle cell anemia who says "my eyes always look sort of yellow" b. 23-year-old with no previous health problems who has a nontender lump in the axilla c. 50-year-old with early-stage chronic lymphocytic leukemia who reports chronic fatigue d. 19-year-old with hemophilia who wants to learn to self-administer factor VII replacement

ANS: B The patient's age and presence of a nontender axillary lump suggest possible lymphoma, which needs rapid diagnosis and treatment. The other patients have questions about treatment or symptoms that are consistent with their diagnosis but do not need to be seen urgently.

ollowing a thymectomy, a 62-year-old male patient with myasthenia gravis receives the usual dose of pyridostigmine (Mestinon). An hour later, the patient complains of nausea and severe abdominal cramps. Which action should the nurse take first? a. Auscultate the patient's bowel sounds. b. Notify the patient's health care provider. c. Administer the prescribed PRN antiemetic drug. d. Give the scheduled dose of prednisone (Deltasone).

ANS: B The patient's history and symptoms indicate a possible cholinergic crisis. The health care provider should be notified immediately, and it is likely that atropine will be prescribed. The other actions will be appropriate if the patient is not experiencing a cholinergic crisis. DIF: Cognitive Level: Apply (application) REF: 1438-1439 OBJ: Special Questions: Prioritization TOP: Nursing Process: Implementation MSC: NCLEX: Physiological Integrity

35. A patient with septicemia develops prolonged bleeding from venipuncture sites and blood in the stools. Which action is most important for the nurse to take? a. Avoid venipunctures. b. Notify the patient's physician. c. Apply sterile dressings to the sites. d. Give prescribed proton-pump inhibitors.

ANS: B The patient's new onset of bleeding and diagnosis of sepsis suggest that disseminated intravascular coagulation (DIC) may have developed, which will require collaborative actions such as diagnostic testing, blood product administration, and heparin administration. The other actions also are appropriate, but the most important action should be to notify the physician so that DIC treatment can be initiated rapidly.

A patient with septicemia develops prolonged bleeding from venipuncture sites and blood in the stools. Which action is most important for the nurse to take? a. Avoid venipunctures. b. Notify the patient's physician. c. Apply sterile dressings to the sites. d. Give prescribed proton-pump inhibitors.

ANS: B The patient's new onset of bleeding and diagnosis of sepsis suggest that disseminated intravascular coagulation (DIC) may have developed, which will require collaborative actions such as diagnostic testing, blood product administration, and heparin administration. The other actions also are appropriate, but the most important action should be to notify the physician so that DIC treatment can be initiated rapidly.

28. The nurse is planning to administer a transfusion of packed red blood cells (PRBCs) to a patient with blood loss from gastrointestinal hemorrhage. Which action can the nurse delegate to unlicensed assistive personnel (UAP)? a. Verify the patient identification (ID) according to hospital policy. b. Obtain the temperature, blood pressure, and pulse before the transfusion. c. Double-check the product numbers on the PRBCs with the patient ID band. d. Monitor the patient for shortness of breath or chest pain during the transfusion.

ANS: B UAP education includes measurement of vital signs. UAP would report the vital signs to the registered nurse (RN). The other actions require more education and a larger scope of practice and should be done by licensed nursing staff members.

A hospitalized patient complains of a bilateral headache, 4/10 on the pain scale, that radiates from the base of the skull. Which prescribed PRN medications should the nurse administer initially? a. Lorazepam (Ativan) b. Acetaminophen (Tylenol) c. Morphine sulfate (Roxanol) d. Butalbital and aspirin (Fiorinal)

ANS: B The patient's symptoms are consistent with a tension headache, and initial therapy usually involves a nonopioid analgesic such as acetaminophen, which is sometimes combined with a sedative or muscle relaxant. Lorazepam may be used in conjunction with acetaminophen but would not be appropriate as the initial monotherapy. Morphine sulfate and butalbital and aspirin would be more appropriate for a headache that did not respond to a nonopioid analgesic. DIF: Cognitive Level: Apply (application) REF: 1414 OBJ: Special Questions: Prioritization TOP: Nursing Process: Implementation MSC: NCLEX: Physiological Integrity

The health care provider is considering the use of sumatriptan (Imitrex) for a 54-year-old male patient with migraine headaches. Which information obtained by the nurse is most important to report to the health care provider? a. The patient drinks 1 to 2 cups of coffee daily. b. The patient had a recent acute myocardial infarction. c. The patient has had migraine headaches for 30 years. d. The patient has taken topiramate (Topamax) for 2 months.

ANS: B The triptans cause coronary artery vasoconstriction and should be avoided in patients with coronary artery disease. The other information will be reported to the health care provider, but none of it indicates that sumatriptan would be an inappropriate treatment. DIF: Cognitive Level: Apply (application) REF: 1416 OBJ: Special Questions: Prioritization TOP: Nursing Process: Assessment MSC: NCLEX: Physiological Integrity

A patient has a nursing diagnosis of risk for ineffective cerebral tissue perfusion related to cerebral tissue to cerebral edema. What is an appropriate nursing intervention for the patient?

Avoid positioning the patient with neck and hip flexion

A 68-year-old woman with acute myelogenous leukemia (AML) asks the nurse whether the planned chemotherapy will be worth undergoing. Which response by the nurse is appropriate? a. "If you do not want to have chemotherapy, other treatment options include stem cell transplantation." b. "The side effects of chemotherapy are difficult, but AML frequently goes into remission with chemotherapy." c. "The decision about treatment is one that you and the doctor need to make rather than asking what I would do." d. "You don't need to make a decision about treatment right now because leukemias in adults tend to progress quite slowly."

ANS: B This response uses therapeutic communication by addressing the patient's question and giving accurate information. The other responses either give inaccurate information or fail to address the patient's question, which will discourage the patient from asking the nurse for information

21. A 68-year-old woman with acute myelogenous leukemia (AML) asks the nurse whether the planned chemotherapy will be worth undergoing. Which response by the nurse is appropriate? a. "If you do not want to have chemotherapy, other treatment options include stem cell transplantation." b. "The side effects of chemotherapy are difficult, but AML frequently goes into remission with chemotherapy." c. "The decision about treatment is one that you and the doctor need to make rather than asking what I would do." d. "You don't need to make a decision about treatment right now because leukemias in adults tend to progress quite slowly."

ANS: B This response uses therapeutic communication by addressing the patient's question and giving accurate information. The other responses either give inaccurate information or fail to address the patient's question, which will discourage the patient from asking the nurse for information.

The nurse is planning to administer a transfusion of packed red blood cells (PRBCs) to a patient with blood loss from gastrointestinal hemorrhage. Which action can the nurse delegate to unlicensed assistive personnel (UAP)? a. Verify the patient identification (ID) according to hospital policy. b. Obtain the temperature, blood pressure, and pulse before the transfusion. c. Double-check the product numbers on the PRBCs with the patient ID band. d. Monitor the patient for shortness of breath or chest pain during the transfusion.

ANS: B UAP education includes measurement of vital signs. UAP would report the vital signs to the registered nurse (RN). The other actions require more education and a larger scope of practice and should be done by licensed nursing staff members

The nurse will assess a 67-year-old patient who is experiencing a cluster headache for a. nuchal rigidity. b. unilateral ptosis. c. projectile vomiting. d. throbbing, bilateral facial pain.

ANS: B Unilateral eye edema, tearing, and ptosis are characteristic of cluster headaches. Nuchal rigidity suggests meningeal irritation, such as occurs with meningitis. Although nausea and vomiting may occur with migraine headaches, projectile vomiting is more consistent with increased intracranial pressure (ICP). Unilateral sharp, stabbing pain, rather than throbbing pain, is characteristic of cluster headaches. DIF: Cognitive Level: Understand (comprehension) REF: 1414 TOP: Nursing Process: Assessment MSC: NCLEX: Physiological Integrity

7. When obtaining a health history and physical assessment for a 36-year-old female patient with possible multiple sclerosis (MS), the nurse should a. assess for the presence of chest pain. b. inquire about urinary tract problems. c. inspect the skin for rashes or discoloration. d. ask the patient about any increase in libido.

ANS: B Urinary tract problems with incontinence or retention are common symptoms of MS. Chest pain and skin rashes are not symptoms of MS. A decrease in libido is common with MS. DIF: Cognitive Level: Apply (application) REF: 1429 TOP: Nursing Process: Assessment MSC: NCLEX: Physiological Integrity

Which action for a patient with neutropenia is appropriate for the registered nurse (RN) to delegate to a licensed practical/vocational nurse (LPN/LVN)? a. Assessing the patient for signs and symptoms of infection b. Teaching the patient the purpose of neutropenic precautions c. Administering subcutaneous filgrastim (Neupogen) injection d. Developing a discharge teaching plan for the patient and family

ANS: C Administration of subcutaneous medications is included in LPN/LVN education and scope of practice. Patient education, assessment, and developing the plan of care require RN level education and scope of practice

37. Which action for a patient with neutropenia is appropriate for the registered nurse (RN) to delegate to a licensed practical/vocational nurse (LPN/LVN)? a. Assessing the patient for signs and symptoms of infection b. Teaching the patient the purpose of neutropenic precautions c. Administering subcutaneous filgrastim (Neupogen) injection d. Developing a discharge teaching plan for the patient and family

ANS: C Administration of subcutaneous medications is included in LPN/LVN education and scope of practice. Patient education, assessment, and developing the plan of care require RN level education and scope of practice.

46. When a patient with splenomegaly is scheduled for splenectomy, which action will the nurse include in the preoperative plan of care? a. Discourage deep breathing to reduce risk for splenic rupture. b. Teach the patient to use ibuprofen (Advil) for left upper quadrant pain. c. Schedule immunization with the pneumococcal vaccine (Pneumovax). d. Avoid the use of acetaminophen (Tylenol) for 2 weeks prior to surgery.

ANS: C Asplenic patients are at high risk for infection with Pneumococcus and immunization reduces this risk. There is no need to avoid acetaminophen use before surgery, but nonsteroidal antiinflammatory drugs (NSAIDs) may increase bleeding risk and should be avoided. The enlarged spleen may decrease respiratory depth and the patient should be encouraged to take deep breaths

When a patient with splenomegaly is scheduled for splenectomy, which action will the nurse include in the preoperative plan of care? a. Discourage deep breathing to reduce risk for splenic rupture. b. Teach the patient to use ibuprofen (Advil) for left upper quadrant pain. c. Schedule immunization with the pneumococcal vaccine (Pneumovax). d. Avoid the use of acetaminophen (Tylenol) for 2 weeks prior to surgery.

ANS: C Asplenic patients are at high risk for infection with Pneumococcus and immunization reduces this risk. There is no need to avoid acetaminophen use before surgery, but nonsteroidal antiinflammatory drugs (NSAIDs) may increase bleeding risk and should be avoided. The enlarged spleen may decrease respiratory depth and the patient should be encouraged to take deep breaths.

4. A 52-year-old patient has a new diagnosis of pernicious anemia. The nurse determines that the patient understands the teaching about the disorder when the patient states, "I a. need to start eating more red meat and liver." b. will stop having a glass of wine with dinner." c. could choose nasal spray rather than injections of vitamin B12." d. will need to take a proton pump inhibitor like omeprazole (Prilosec)."

ANS: C Because pernicious anemia prevents the absorption of vitamin B12, this patient requires injections or intranasal administration of cobalamin. Alcohol use does not cause cobalamin deficiency. Proton pump inhibitors decrease the absorption of vitamin B12. Eating more foods rich in vitamin B12 is not helpful because the lack of intrinsic factor prevents absorption of the vitamin.

A 52-year-old patient has a new diagnosis of pernicious anemia. The nurse determines that the patient understands the teaching about the disorder when the patient states, "I a. need to start eating more red meat and liver." b. will stop having a glass of wine with dinner." c. could choose nasal spray rather than injections of vitamin B12." d. will need to take a proton pump inhibitor like omeprazole (Prilosec)."

ANS: C Because pernicious anemia prevents the absorption of vitamin B12, this patient requires injections or intranasal administration of cobalamin. Alcohol use does not cause cobalamin deficiency. Proton pump inhibitors decrease the absorption of vitamin B12. Eating more foods rich in vitamin B12 is not helpful because the lack of intrinsic factor prevents absorption of the vitamin.

Which assessment is most important for the nurse to make regarding a patient with myasthenia gravis? a. Pupil size b. Grip strength c. Respiratory effort d. Level of consciousness

ANS: C Because respiratory insufficiency may be life threatening, it will be most important to monitor respiratory function. The other data also will be assessed but are not as critical. DIF: Cognitive Level: Apply (application) REF: 1438-1439 OBJ: Special Questions: Prioritization TOP: Nursing Process: Assessment MSC: NCLEX: Physiological Integrity

While the nurse is transporting a patient on a stretcher to the radiology department, the patient begins having a tonic-clonic seizure. Which action should the nurse take? a. Insert an oral airway during the seizure to maintain a patent airway. b. Restrain the patient's arms and legs to prevent injury during the seizure. c. Time and observe and record the details of the seizure and postictal state. d. Avoid touching the patient to prevent further nervous system stimulation.

ANS: C Because the diagnosis and treatment of seizures frequently are based on the description of the seizure, recording the length and details of the seizure is important. Insertion of an oral airway and restraining the patient during the seizure are contraindicated. The nurse may need to move the patient to decrease the risk of injury during the seizure. DIF: Cognitive Level: Apply (application) REF: 1422 TOP: Nursing Process: Implementation MSC: NCLEX: Physiological Integrity

A 49-year-old patient with multiple sclerosis (MS) is to begin treatment with glatiramer acetate (Copaxone). Which information will the nurse include in patient teaching? a. Recommendation to drink at least 4 L of fluid daily b. Need to avoid driving or operating heavy machinery c. How to draw up and administer injections of the medication d. Use of contraceptive methods other than oral contraceptives

ANS: C Copaxone is administered by self-injection. Oral contraceptives are an appropriate choice for birth control. There is no need to avoid driving or drink large fluid volumes when taking glatiramer. DIF: Cognitive Level: Apply (application) REF: 1430 TOP: Nursing Process: Implementation MSC: NCLEX: Physiological Integrity

10. Which instruction will the nurse plan to include in discharge teaching for the patient admitted with a sickle cell crisis? a. Take a daily multivitamin with iron. b. Limit fluids to 2 to 3 quarts per day. c. Avoid exposure to crowds when possible. d. Drink only two caffeinated beverages daily.

ANS: C Exposure to crowds increases the patient's risk for infection, the most common cause of sickle cell crisis. There is no restriction on caffeine use. Iron supplementation is generally not recommended. A high-fluid intake is recommended

Which instruction will the nurse plan to include in discharge teaching for the patient admitted with a sickle cell crisis? a. Take a daily multivitamin with iron. b. Limit fluids to 2 to 3 quarts per day. c. Avoid exposure to crowds when possible. d. Drink only two caffeinated beverages daily.

ANS: C Exposure to crowds increases the patient's risk for infection, the most common cause of sickle cell crisis. There is no restriction on caffeine use. Iron supplementation is generally not recommended. A high-fluid intake is recommended.

A 40-year-old patient is diagnosed with early Huntington's disease (HD). When teaching the patient, spouse, and children about this disorder, the nurse will provide information about the a. use of levodopa-carbidopa (Sinemet) to help reduce HD symptoms. b. prophylactic antibiotics to decrease the risk for aspiration pneumonia. c. option of genetic testing for the patient's children to determine their own HD risks. d. lifestyle changes of improved nutrition and exercise that delay disease progression.

ANS: C Genetic testing is available to determine whether an asymptomatic individual has the HD gene. The patient and family should be informed of the benefits and problems associated with genetic testing. Sinemet will increase symptoms of HD because HD involves an increase in dopamine. Antibiotic therapy will not reduce the risk for aspiration. There are no effective treatments or lifestyle changes that delay the progression of symptoms in HD. DIF: Cognitive Level: Apply (application) REF: 1440 TOP: Nursing Process: Implementation MSC: NCLEX: Physiological Integrity

Nursing staff on a hospital unit are reviewing rates of hospital-acquired infections (HAI) of the urinary tract. Which nursing action will be most helpful in decreasing the risk for HAI in patients admitted to the hospital?

Avoiding unnecessary urinary catheterizations

The nurse determines that teaching about management of migraine headaches has been effective when the patient says which of the following? a. "I can take the (Topamax) as soon as a headache starts." b. "A glass of wine might help me relax and prevent a headache." c. "I will lie down someplace dark and quiet when the headaches begin." d. "I should avoid taking aspirin and sumatriptan (Imitrex) at the same time."

ANS: C It is recommended that the patient with a migraine rest in a dark, quiet area. Topiramate (Topamax) is used to prevent migraines and must be taken for several months to determine effectiveness. Aspirin or other nonsteroidal antiinflammatory medications can be taken with the triptans. Alcohol may precipitate migraine headaches. DIF: Cognitive Level: Apply (application) REF: 1416 | 1419 TOP: Nursing Process: Evaluation MSC: NCLEX: Physiological Integrity

A 71-year-old patient had an abdominal-perineal resection for colon cancer. Which nursing action is most important to include in the plan of care for the day after surgery? a. Teach about a low-residue diet. b. Monitor output from the stoma. c. Assess the perineal drainage and incision. d. Encourage acceptance of the colostomy stoma.

C

You have a 74 male who slipped and fell and had bruising right flank, peritoneal bleeding about two weeks ago. He woke up confused, disoriented and with left sided weakness. What happened to this patient?

Cerebral Vascular Accident (CVA)

23. A 54-year-old woman with acute myelogenous leukemia (AML) is considering treatment with a hematopoietic stem cell transplant (HSCT). The best approach for the nurse to assist the patient with a treatment decision is to a. emphasize the positive outcomes of a bone marrow transplant. b. discuss the need for adequate insurance to cover post-HSCT care. c. ask the patient whether there are any questions or concerns about HSCT. d. explain that a cure is not possible with any other treatment except HSCT.

ANS: C Offering the patient an opportunity to ask questions or discuss concerns about HSCT will encourage the patient to voice concerns about this treatment and also will allow the nurse to assess whether the patient needs more information about the procedure. Treatment of AML using chemotherapy is another option for the patient. It is not appropriate for the nurse to ask the patient to consider insurance needs in making this decision.

A 54-year-old woman with acute myelogenous leukemia (AML) is considering treatment with a hematopoietic stem cell transplant (HSCT). The best approach for the nurse to assist the patient with a treatment decision is to a. emphasize the positive outcomes of a bone marrow transplant. b. discuss the need for adequate insurance to cover post-HSCT care. c. ask the patient whether there are any questions or concerns about HSCT. d. explain that a cure is not possible with any other treatment except HSCT.

ANS: C Offering the patient an opportunity to ask questions or discuss concerns about HSCT will encourage the patient to voice concerns about this treatment and also will allow the nurse to assess whether the patient needs more information about the procedure. Treatment of AML using chemotherapy is another option for the patient. It is not appropriate for the nurse to ask the patient to consider insurance needs in making this decision.

42. Which finding about a patient with polycythemia vera is most important for the nurse to report to the health care provider? a. Hematocrit 55% b. Presence of plethora c. Calf swelling and pain d. Platelet count 450,000/L

ANS: C The calf swelling and pain suggest that the patient may have developed a deep vein thrombosis, which will require diagnosis and treatment to avoid complications such as pulmonary embolus. The other findings will also be reported to the health care provider but are expected in a patient with this diagnosis.

Which finding about a patient with polycythemia vera is most important for the nurse to report to the health care provider? a. Hematocrit 55% b. Presence of plethora c. Calf swelling and pain d. Platelet count 450,000/L

ANS: C The calf swelling and pain suggest that the patient may have developed a deep vein thrombosis, which will require diagnosis and treatment to avoid complications such as pulmonary embolus. The other findings will also be reported to the health care provider but are expected in a patient with this diagnosis.

Which action will the nurse include in the plan of care for a patient who has thalassemia major? a. Teach the patient to use iron supplements. b. Avoid the use of intramuscular injections. c. Administer iron chelation therapy as needed. d. Notify health care provider of hemoglobin 11g/dL.

ANS: C The frequent transfusions used to treat thalassemia major lead to iron toxicity in patients unless iron chelation therapy is consistently used. Iron supplementation is avoided in patients with thalassemia. There is no need to avoid intramuscular injections. The goal for patients with thalassemia major is to maintain a hemoglobin of 10 g/dL or greater

40. Which action will the nurse include in the plan of care for a patient who has thalassemia major? a. Teach the patient to use iron supplements. b. Avoid the use of intramuscular injections. c. Administer iron chelation therapy as needed. d. Notify health care provider of hemoglobin 11g/dL.

ANS: C The frequent transfusions used to treat thalassemia major lead to iron toxicity in patients unless iron chelation therapy is consistently used. Iron supplementation is avoided in patients with thalassemia. There is no need to avoid intramuscular injections. The goal for patients with thalassemia major is to maintain a hemoglobin of 10 g/dL or greater.

A patient admitted with dermal ulcers who has a history of a T3 spinal cord injury tells the nurse, I have a pounding headache and I feel sick to my stomach. Which action should the nurse take first?

Assess the Blood Pressure

A 22-year-old patient seen at the health clinic with a severe migraine headache tells the nurse about having other similar headaches recently. Which initial action should the nurse take? a. Teach about the use of triptan drugs. b. Refer the patient for stress counseling. c. Ask the patient to keep a headache diary. d. Suggest the use of muscle-relaxation techniques.

ANS: C The initial nursing action should be further assessment of the precipitating causes of the headaches, quality, and location of pain, etc. Stress reduction, muscle relaxation, and the triptan drugs may be helpful, but more assessment is needed first. DIF: Cognitive Level: Apply (application) REF: 1419 OBJ: Special Questions: Prioritization TOP: Nursing Process: Implementation MSC: NCLEX: Physiological Integrity

36. A patient with possible disseminated intravascular coagulation arrives in the emergency department with a blood pressure of 82/40, temperature 102° F (38.9° C), and severe back pain. Which physician order will the nurse implement first? a. Administer morphine sulfate 4 mg IV. b. Give acetaminophen (Tylenol) 650 mg. c. Infuse normal saline 500 mL over 30 minutes. d. Schedule complete blood count and coagulation studies.

ANS: C The patient's blood pressure indicates hypovolemia caused by blood loss and should be addressed immediately to improve perfusion to vital organs. The other actions also are appropriate and should be rapidly implemented, but improving perfusion is the priority for this patient.

A patient with possible disseminated intravascular coagulation arrives in the emergency department with a blood pressure of 82/40, temperature 102° F (38.9° C), and severe back pain. Which physician order will the nurse implement first? a. Administer morphine sulfate 4 mg IV. b. Give acetaminophen (Tylenol) 650 mg. c. Infuse normal saline 500 mL over 30 minutes. d. Schedule complete blood count and coagulation studies.

ANS: C The patient's blood pressure indicates hypovolemia caused by blood loss and should be addressed immediately to improve perfusion to vital organs. The other actions also are appropriate and should be rapidly implemented, but improving perfusion is the priority for this patient.

29. A postoperative patient receiving a transfusion of packed red blood cells develops chills, fever, headache, and anxiety 35 minutes after the transfusion is started. After stopping the transfusion, what action should the nurse take? a. Draw blood for a new crossmatch. b. Send a urine specimen to the laboratory. c. Administer PRN acetaminophen (Tylenol). d. Give the PRN diphenhydramine (Benadryl).

ANS: C The patient's clinical manifestations are consistent with a febrile, nonhemolytic transfusion reaction. The transfusion should be stopped and antipyretics administered for the fever as ordered. A urine specimen is needed if an acute hemolytic reaction is suspected. Diphenhydramine (Benadryl) is used for allergic reactions. This type of reaction does not indicate incorrect crossmatching

A patient with a tracheostomy has a new order for a fenestrated tracheostomy tube. Which action should the nurse include in the plan of care in collaboration with the speech therapist?

Assess the ability to swallow before using the fenestrated tube.

A postoperative patient receiving a transfusion of packed red blood cells develops chills, fever, headache, and anxiety 35 minutes after the transfusion is started. After stopping the transfusion, what action should the nurse take? a. Draw blood for a new crossmatch. b. Send a urine specimen to the laboratory. c. Administer PRN acetaminophen (Tylenol). d. Give the PRN diphenhydramine (Benadryl).

ANS: C The patient's clinical manifestations are consistent with a febrile, nonhemolytic transfusion reaction. The transfusion should be stopped and antipyretics administered for the fever as ordered. A urine specimen is needed if an acute hemolytic reaction is suspected. Diphenhydramine (Benadryl) is used for allergic reactions. This type of reaction does not indicate incorrect crossmatching.

The physician orders intracranial pressure (ICP) readings every hour for a 23-year-old male patient with a traumatic brain injury from a motor vehicle crash. The patient's ICP reading is 21 mm Hg. It is most important for the nurse to take which action?

Assess the patient's level of consciousness

The nurse is preparing to teach a 43-year-old man who is newly diagnosed with type 2 diabetes about home management of the disease. Which action should the nurse take first?

Assess the patient's perception of what it means to have diabetes mellitus.

A patient has just arrived on the unit after a thyroidectomy. Which action should the nurse take first?

Assess the patient's respiratory effort.

A patient who has been receiving a heparin infusion and warfarin (Coumadin) for a deep vein thrombosis (DVT) is diagnosed with heparin-induced thrombocytopenia (HIT) when her platelet level drops to 110,000/µL. Which action will the nurse include in the plan of care? a. Use low-molecular-weight heparin (LMWH) only. b. Administer the warfarin (Coumadin) at the scheduled time. c. Teach the patient about the purpose of platelet transfusions. d. Discontinue heparin and flush intermittent IV lines using normal saline.

ANS: D All heparin is discontinued when the HIT is diagnosed. The patient should be instructed to never receive heparin or LMWH. Warfarin is usually not given until the platelet count has returned to 150,000/µL. The platelet count does not drop low enough in HIT for a platelet transfusion, and platelet transfusions increase the risk for thrombosis

12. A patient who has been receiving a heparin infusion and warfarin (Coumadin) for a deep vein thrombosis (DVT) is diagnosed with heparin-induced thrombocytopenia (HIT) when her platelet level drops to 110,000/µL. Which action will the nurse include in the plan of care? a. Use low-molecular-weight heparin (LMWH) only. b. Administer the warfarin (Coumadin) at the scheduled time. c. Teach the patient about the purpose of platelet transfusions. d. Discontinue heparin and flush intermittent IV lines using normal saline.

ANS: D All heparin is discontinued when the HIT is diagnosed. The patient should be instructed to never receive heparin or LMWH. Warfarin is usually not given until the platelet count has returned to 150,000/µL. The platelet count does not drop low enough in HIT for a platelet transfusion, and platelet transfusions increase the risk for thrombosis.

Which medication taken by a patient with restless legs syndrome should the nurse discuss with the patient? a. Multivitamin (Stresstabs) b. Acetaminophen (Tylenol) c. Ibuprofen (Motrin, Advil) d. Diphenhydramine (Benadryl)

ANS: D Antihistamines can aggravate restless legs syndrome. The other medications will not contribute to restless legs syndrome. DIF: Cognitive Level: Apply (application) REF: 1427 TOP: Nursing Process: Implementation MSC: NCLEX: Physiological Integrity

9. Which statement by a patient indicates good understanding of the nurse's teaching about prevention of sickle cell crisis? a. "Home oxygen therapy is frequently used to decrease sickling." b. "There are no effective medications that can help prevent sickling." c. "Routine continuous dosage narcotics are prescribed to prevent a crisis." d. "Risk for a crisis is decreased by having an annual influenza vaccination."

ANS: D Because infection is the most common cause of a sickle cell crisis, influenza, Haemophilus influenzae, pneumococcal pneumonia, and hepatitis immunizations should be administered. Although continuous dose opioids and oxygen may be administered during a crisis, patients do not receive these therapies to prevent crisis. Hydroxyurea (Hydrea) is a medication used to decrease the number of sickle cell crises.

Which statement by a patient indicates good understanding of the nurse's teaching about prevention of sickle cell crisis? a. "Home oxygen therapy is frequently used to decrease sickling." b. "There are no effective medications that can help prevent sickling." c. "Routine continuous dosage narcotics are prescribed to prevent a crisis." d. "Risk for a crisis is decreased by having an annual influenza vaccination."

ANS: D Because infection is the most common cause of a sickle cell crisis, influenza, Haemophilus influenzae, pneumococcal pneumonia, and hepatitis immunizations should be administered. Although continuous dose opioids and oxygen may be administered during a crisis, patients do not receive these therapies to prevent crisis. Hydroxyurea (Hydrea) is a medication used to decrease the number of sickle cell crises.

41. Which patient information is most important for the nurse to monitor when evaluating the effectiveness of deferoxamine (Desferal) for a patient with hemochromatosis? a. Skin color b. Hematocrit c. Liver function d. Serum iron level

ANS: D Because iron chelating agents are used to lower serum iron levels, the most useful information will be the patient's iron level. The other parameters will also be monitored, but are not the most important to monitor when determining the effectiveness of deferoxamine.

Which patient information is most important for the nurse to monitor when evaluating the effectiveness of deferoxamine (Desferal) for a patient with hemochromatosis? a. Skin color b. Hematocrit c. Liver function d. Serum iron level

ANS: D Because iron chelating agents are used to lower serum iron levels, the most useful information will be the patient's iron level. The other parameters will also be monitored, but are not the most important to monitor when determining the effectiveness of deferoxamine.

Which information about a 60-year-old patient with MS indicates that the nurse should consult with the health care provider before giving the prescribed dose of dalfampridine (Ampyra)? a. The patient has relapsing-remitting MS. b. The patient walks a mile a day for exercise. c. The patient complains of pain with neck flexion. d. The patient has an increased serum creatinine level.

ANS: D Dalfampridine should not be given to patients with impaired renal function. The other information will not impact whether the dalfampridine should be administered. DIF: Cognitive Level: Apply (application) REF: 1431 TOP: Nursing Process: Assessment MSC: NCLEX: Physiological Integrity

A 76-year-old patient is being treated with carbidopa/levodopa (Sinemet) for Parkinson's disease. Which information is most important for the nurse to report to the health care provider? a. Shuffling gait b. Tremor at rest c. Cogwheel rigidity of limbs d. Uncontrolled head movement

ANS: D Dyskinesia is an adverse effect of the Sinemet, indicating a need for a change in medication or decrease in dose. The other findings are typical with Parkinson's disease. DIF: Cognitive Level: Apply (application) REF: 1435 OBJ: Special Questions: Prioritization TOP: Nursing Process: Planning MSC: NCLEX: Physiological Integrity

20. Which laboratory test will the nurse use to determine whether filgrastim (Neupogen) is effective for a patient with acute lymphocytic leukemia who is receiving chemotherapy? a. Platelet count b. Reticulocyte count c. Total lymphocyte count d. Absolute neutrophil count

ANS: D Filgrastim increases the neutrophil count and function in neutropenic patients. Although total lymphocyte, platelet, and reticulocyte counts also are important to monitor in this patient, the absolute neutrophil count is used to evaluate the effects of filgrastim.

Which laboratory test will the nurse use to determine whether filgrastim (Neupogen) is effective for a patient with acute lymphocytic leukemia who is receiving chemotherapy? a. Platelet count b. Reticulocyte count c. Total lymphocyte count d. Absolute neutrophil count

ANS: D Filgrastim increases the neutrophil count and function in neutropenic patients. Although total lymphocyte, platelet, and reticulocyte counts also are important to monitor in this patient, the absolute neutrophil count is used to evaluate the effects of filgrastim.

A 62-year-old patient who has Parkinson's disease is taking bromocriptine (Parlodel). Which information obtained by the nurse may indicate a need for a decrease in the dose? a. The patient has a chronic dry cough. b. The patient has four loose stools in a day. c. The patient develops a deep vein thrombosis. d. The patient's blood pressure is 92/52 mm Hg.

ANS: D Hypotension is an adverse effect of bromocriptine, and the nurse should check with the health care provider before giving the medication. Diarrhea, cough, and deep vein thrombosis are not associated with bromocriptine use. DIF: Cognitive Level: Apply (application) REF: 1435 TOP: Nursing Process: Evaluation MSC: NCLEX: Physiological Integrity

A patient who has non-Hodgkin's lymphoma is receiving combination treatment with rituximab (Rituxan) and chemotherapy. Which patient assessment finding requires the most rapid action by the nurse? a. Anorexia b. Vomiting c. Oral ulcers d. Lip swelling

ANS: D Lip swelling in angioedema may indicate a hypersensitivity reaction to the rituximab. The nurse should stop the infusion and further assess for anaphylaxis. The other findings may occur with chemotherapy, but are not immediately life threatening

44. A patient who has non-Hodgkin's lymphoma is receiving combination treatment with rituximab (Rituxan) and chemotherapy. Which patient assessment finding requires the most rapid action by the nurse? a. Anorexia b. Vomiting c. Oral ulcers d. Lip swelling

ANS: D Lip swelling in angioedema may indicate a hypersensitivity reaction to the rituximab. The nurse should stop the infusion and further assess for anaphylaxis. The other findings may occur with chemotherapy, but are not immediately life threatening.

Which laboratory result will the nurse expect to show a decreased value if a patient develops heparin-induced thrombocytopenia (HIT)? a. Prothrombin time b. Erythrocyte count c. Fibrinogen degradation products d. Activated partial thromboplastin time

ANS: D Platelet aggregation in HIT causes neutralization of heparin, so that the activated partial thromboplastin time will be shorter and more heparin will be needed to maintain therapeutic levels. The other data will not be affected by HIT.

Which information about a 72-year-old patient who has a new prescription for phenytoin (Dilantin) indicates that the nurse should consult with the health care provider before administration of the medication? a. Patient has generalized tonic-clonic seizures. b. Patient experiences an aura before seizures. c. Patient's most recent blood pressure is 156/92 mm Hg. d. Patient has minor elevations in the liver function tests.

ANS: D Many older patients (especially with compromised liver function) may not be able to metabolize phenytoin. The health care provider may need to choose another antiseizure medication. Phenytoin is an appropriate medication for patients with tonic-clonic seizures, with or without an aura. Hypertension is not a contraindication for phenytoin therapy. DIF: Cognitive Level: Apply (application) REF: 1424 TOP: Nursing Process: Implementation MSC: NCLEX: Physiological Integrity

A critical action by the nurse caring for a patient with an acute exacerbation of polycythemia vera is to a. place the patient on bed rest. b. administer iron supplements. c. avoid use of aspirin products. d. monitor fluid intake and output.

ANS: D Monitoring hydration status is important during an acute exacerbation because the patient is at risk for fluid overload or underhydration. Aspirin therapy is used to decrease risk for thrombosis. The patient should be encouraged to ambulate to prevent deep vein thrombosis (DVT). Iron is contraindicated in patients with polycythemia vera

13. A critical action by the nurse caring for a patient with an acute exacerbation of polycythemia vera is to a. place the patient on bed rest. b. administer iron supplements. c. avoid use of aspirin products. d. monitor fluid intake and output.

ANS: D Monitoring hydration status is important during an acute exacerbation because the patient is at risk for fluid overload or underhydration. Aspirin therapy is used to decrease risk for thrombosis. The patient should be encouraged to ambulate to prevent deep vein thrombosis (DVT). Iron is contraindicated in patients with polycythemia vera.

When assessing the patient with a multi-lumen central line, the nurse notices that the cap is off one of the lines. On assessment, the patient is in respiratory distress and the vital signs show hypotension and tachycardia. What is the nurse's priority action?

Administer oxygen

Which nursing action is of highest priority for a 68-year-old patient with renal calculi who is being admitted to the hospital with gross hematuria and severe colicky left flank pain?

Administer prescribed analgesics.

15. Which laboratory result will the nurse expect to show a decreased value if a patient develops heparin-induced thrombocytopenia (HIT)? a. Prothrombin time b. Erythrocyte count c. Fibrinogen degradation products d. Activated partial thromboplastin time

ANS: D Platelet aggregation in HIT causes neutralization of heparin, so that the activated partial thromboplastin time will be shorter and more heparin will be needed to maintain therapeutic levels. The other data will not be affected by HIT.

A 23-year-old patient is admitted with diabetes insipidus. Which action will be mostappropriate for the registered nurse (RN) to delegate to an experienced licensed practical/vocational nurse (LPN/LVN)?

Administer subcutaneous DDAVP.

When a patient with type 2 diabetes is admitted for a cholecystectomy, which nursing action can the nurse delegate to a licensed practical/vocational nurse (LPN/LVN)?

Administer the prescribed lispro (Humalog) insulin before transporting the patient to surgery.

The nurse notes that a patient who was admitted with diabetic ketoacidosis has rapid, deep respirations. Which action should the nurse take?

Administer the prescribed normal saline bolus and insulin.

The nurse reviews the electronic medical record for a patient scheduled for a total hip replacement. Which assessment data shown in the accompanying figure increase the patients risk for respiratory complications after surgery?

Albumin level and recent weight loss

43. Following successful treatment of Hodgkin's lymphoma for a 55-year-old woman, which topic will the nurse include in patient teaching? a. Potential impact of chemotherapy treatment on fertility b. Application of soothing lotions to treat residual pruritus c. Use of maintenance chemotherapy to maintain remission d. Need for follow-up appointments to screen for malignancy

ANS: D The chemotherapy used in treating Hodgkin's lymphoma results in a high incidence of secondary malignancies; follow-up screening is needed. The fertility of a 55-year-old woman will not be impacted by chemotherapy. Maintenance chemotherapy is not used for Hodgkin's lymphoma. Pruritus is a clinical manifestation of lymphoma, but should not be a concern after treatment.

Following successful treatment of Hodgkin's lymphoma for a 55-year-old woman, which topic will the nurse include in patient teaching? a. Potential impact of chemotherapy treatment on fertility b. Application of soothing lotions to treat residual pruritus c. Use of maintenance chemotherapy to maintain remission d. Need for follow-up appointments to screen for malignancy

ANS: D The chemotherapy used in treating Hodgkin's lymphoma results in a high incidence of secondary malignancies; follow-up screening is needed. The fertility of a 55-year-old woman will not be impacted by chemotherapy. Maintenance chemotherapy is not used for Hodgkin's lymphoma. Pruritus is a clinical manifestation of lymphoma, but should not be a concern after treatment.

A 20-year-old has a mandatory electrocardiogram (ECG) before participating on a college soccer team and is found to have sinus bradycardia, rate 52. Blood pressure (BP) is 114/54, and the student denies any health problems. What action by the nurse is most appropriate?

Allow the student to participate on the soccer team.

hich nursing diagnosis is of highest priority for a patient with Parkinson's disease who is unable to move the facial muscles? a. Activity intolerance b. Self-care deficit: toileting c. Ineffective self-health management d. Imbalanced nutrition: less than body requirements

ANS: D The data about the patient indicate that poor nutrition will be a concern because of decreased swallowing. The other diagnoses may also be appropriate for a patient with Parkinson's disease, but the data do not indicate that they are current problems for this patient. DIF: Cognitive Level: Apply (application) REF: 1436 OBJ: Special Questions: Prioritization TOP: Nursing Process: Analysis MSC: NCLEX: Physiological Integrity

A diabetic patient who has reported burning foot pain at night receives a new prescription. Which information should the nurse teach the patient about amitriptyline (Elavil)?

Amitriptyline helps prevent transmission of pain impulses to the brain.

A patient is hospitalized after a successful resuscitation of an episode of sudden cardiac death (SCD). During the care of the patient, what nursing intervention is most important?

Continuous ECG monitoring

1. A 62-year old man with chronic anemia is experiencing increased fatigue and occasional palpitations at rest. The nurse would expect the patient's laboratory findings to include a. a hematocrit (Hct) of 38%. b. an RBC count of 4,500,000/L. c. normal red blood cell (RBC) indices. d. a hemoglobin (Hgb) of 8.6 g/dL (86 g/L).

ANS: D The patient's clinical manifestations indicate moderate anemia, which is consistent with a Hgb of 6 to 10 g/dL. The other values are all within the range of normal.

A 62-year old man with chronic anemia is experiencing increased fatigue and occasional palpitations at rest. The nurse would expect the patient's laboratory findings to include a. a hematocrit (Hct) of 38%. b. an RBC count of 4,500,000/L. c. normal red blood cell (RBC) indices. d. a hemoglobin (Hgb) of 8.6 g/dL (86 g/L).

ANS: D The patient's clinical manifestations indicate moderate anemia, which is consistent with a Hgb of 6 to 10 g/dL. The other values are all within the range of normal.

30. A patient in the emergency department complains of back pain and difficulty breathing 15 minutes after a transfusion of packed red blood cells is started. The nurse's first action should be to a. administer oxygen therapy at a high flow rate. b. obtain a urine specimen to send to the laboratory. c. notify the health care provider about the symptoms. d. disconnect the transfusion and infuse normal saline.

ANS: D The patient's symptoms indicate a possible acute hemolytic reaction caused by the transfusion. The first action should be to disconnect the transfusion and infuse normal saline. The other actions also are needed but are not the highest priority.

A patient in the emergency department complains of back pain and difficulty breathing 15 minutes after a transfusion of packed red blood cells is started. The nurse's first action should be to a. administer oxygen therapy at a high flow rate. b. obtain a urine specimen to send to the laboratory. c. notify the health care provider about the symptoms. d. disconnect the transfusion and infuse normal saline.

ANS: D The patient's symptoms indicate a possible acute hemolytic reaction caused by the transfusion. The first action should be to disconnect the transfusion and infuse normal saline. The other actions also are needed but are not the highest priority.

A triage nurse in a busy ED assesses a patient who complains of 7/10 abdominal pain and states, "I had a temperature of 103F at home." The nurses first action should be to...

Assess the patients current vital signs

When a nurse is assessing a patient with a head injury, what would be the priority of nursing management?

Assessing the LOC

Which nursing action has the highest priority for a patient who was admitted 16 hours previously with a C5 spinal cord injury?

Assessment of respiratory rate and effort

Which prescribed intervention will the nurse implement first for a patient in the emergency department who is experiencing continuous tonic-clonic seizures? a. Give phenytoin (Dilantin) 100 mg IV. b. Monitor level of consciousness (LOC). c. Obtain computed tomography (CT) scan. d. Administer lorazepam (Ativan) 4 mg IV.

ANS: D To prevent ongoing seizures, the nurse should administer rapidly acting antiseizure medications such as the benzodiazepines. A CT scan is appropriate, but prevention of any seizure activity during the CT scan is necessary. Phenytoin will also be administered, but it is not rapidly acting. Patients who are experiencing tonic-clonic seizures are nonresponsive, although the nurse should assess LOC after the seizure. DIF: Cognitive Level: Apply (application) REF: 1424 OBJ: Special Questions: Prioritization TOP: Nursing Process: Implementation MSC: NCLEX: Physiological Integrity

A patient with rheumatic fever has subcutaneous nodules, erythema marginatum, and polyarthritis. Based on these findings, which nursing diagnosis would be most appropriate?

Activity intolerance related to arthralgia

The nurse notes thick, white secretions in the endotracheal tube (ET) of a patient who is receiving mechanical ventilation. Which intervention will be most effective in addressing this problem?

Add additional water to the patients enteral feedings.

A nurse is preping to give a client information about an ACTH stimulation test. The nurse should explain that the purpose of the test is to assess for which of the following disorders? A. Cushing's syndrome B. Hyperthyroidism C. Pheochromocytoma D. Addison's disease

Addison's Disease (ACTH stimulation test is standard test. Measures cortisol response to ACTH. Response is absent or very decreased when there is adrenal insufficiency.

A patient who had a subtotal thyroidectomy earlier today develops laryngeal stridor and a cramp in the right hand upon returning to the surgical nursing unit. Which collaborative action will the nurse anticipate next?

Administer IV calcium gluconate.

Which action by a new registered nurse (RN) caring for a patient with a goiter and possible hyperthyroidism indicates that the charge nurse needs to do more teaching? a. The RN checks the blood pressure on both arms. b. The RN palpates the neck thoroughly to check thyroid size. c. The RN lowers the thermostat to decrease the temperature in the room. d. The RN orders nonmedicated eye drops to lubricate the patient's bulging eyes.

B

Which information about a 30-year-old patient who is scheduled for an oral glucose tolerance test should be reported to the health care provider before starting the test? a. The patient reports having occasional orthostatic dizziness. b. The patient takes oral corticosteroids for rheumatoid arthritis. c. The patient has had a 10-pound weight gain in the last month. d. The patient drank several glasses of water an hour previously.

B

Which information obtained by the nurse interviewing a 30-year-old male patient is most important to communicate to the health care provider? a. The patient has a history of constipation. b. The patient has noticed blood in the stools. c. The patient had an appendectomy at age 27. d. The patient smokes a pack/day of cigarettes.

B

A 24-year-old woman with Crohn's disease develops a fever and symptoms of a urinary tract infection (UTI) with tan, fecal-smelling urine. What information will the nurse add to a general teaching plan about UTIs in order to individualize the teaching for this patient? a. Bacteria in the perianal area can enter the urethra. b. Fistulas can form between the bowel and bladder. c. Drink adequate fluids to maintain normal hydration. d. Empty the bladder before and after sexual intercourse.

B

Which question will provide the most useful information to a nurse who is interviewing a patient about a possible thyroid disorder? a. "What methods do you use to help cope with stress?" b. "Have you experienced any blurring or double vision?" c. "Have you had a recent unplanned weight gain or loss?" d. "Do you have to get up at night to empty your bladder?"

C

The nurse is caring for a patient who has had an ileal conduit for several years. Which nursing action could be delegated to unlicensed assistive personnel (UAP)?

Change the ostomy appliance.

Which statement by a nurse to a patient newly diagnosed with type 2 diabetes is correct?

Changes in diet and exercise may control blood glucose levels in type 2 diabetes.

A patient is admitted to the emergency department with possible renal trauma after an automobile accident. Which prescribed intervention will the nurse implement first?

Check blood pressure and heart rate.

A patient has arrived for a scheduled hemodialysis session. Which nursing action is most appropriate for the registered nurse (RN) to delegate to a dialysis technician?

Check blood pressure before starting dialysis.

The physician has ordered cooling measures for a child with fever who is likely to be discharged when the temperature comes down. Which of the following would be appropriate to delegate to the nursing assistant?

Assist the child to remove outer clothing

Which information will the nurse teach a 48-year-old patient who has been newly diagnosed with Graves' disease?

Antithyroid medications may take several months for full effect.

A 19 y/o pt with massive trauma and possible spinal cord injury is admitted to the ED. Which assessment finding by the nurse will help confirm a diagnosis of neurogenic shock?

Apical HR of 45 bpm

A 19-year-old patient with massive trauma and possible spinal cord injury is admitted to the emergency department (ED). Which assessment finding by the nurse will help confirm a diagnosis of neurogenic shock?

Apical heart rate 45 beats/min.

Which nursing assessment of a 69-year-old patient is most important to make during initiation of thyroid replacement with levothyroxine (Synthroid)?

Apical pulse rate

After the return of a spontaneous circulation following the resuscitation of a patient who had a cardiac arrest, therapeutic hypothermia is ordered. Which action will the nurse include in the plan of care?

Apply external cooling device

A patient develops sinus bradycardia at a rate of 32 beats/minute, has a blood pressure (BP) of 80/42 mm Hg, and is complaining of feeling faint. Which actions should the nurse take next?

Apply the transcutaneous pacemaker (TCP) pads.

An unresponsive 79 year old is admitted to the ED during a summer heat wave. The patients core temperature is 105.4F, BP 88/50, and pulse 112. The nurse initially will plan to...

Apply wet sheets and a fan to the patient

A patient has acute bronchitis with a nonproductive cough and wheezes. Which topic should the nurse plan to include in the teaching plan?

Appropriate use of cough suppressants

An alcoholic and homeless patient is diagnosed with active tuberculosis (TB). Which intervention by the nurse will be most effective in ensuring adherence with the treatment regimen?

Arrange for a daily noon meal at a community center where the drug will be administered.

When planning care for a patient hospitalized with a streptococcal infective endocarditis (IE), which intervention is a priority for the nurse to include?

Arrange for placement of a long-term IV catheter.

To evaluate the effectiveness of ordered interventions for a patient with ventilatory failure, which diagnostic test will be most useful to the nurse?

Arterial blood gas analysis

A patient has ICP monitoring with an intraventricular catheter. What is a priority nursing intervention for the patient?

Aseptic technique to prevent infection

Family members are in the patients room when the patient has a cardiac arrest and the staff start to resuscitation measures. Which action should the nurse take first?

Ask the family members about whether they would prefer to remain in the patients room or wait outside the room

A 19-year-old student comes to the student health center at the end of the semester complaining that, My heart is skipping beats. An electrocardiogram (ECG) shows occasional premature ventricular contractions (PVCs). What action should the nurse take next?

Ask the patient about current stress level and caffeine use.

After 2 months of tuberculosis (TB) treatment with isoniazid (INH), rifampin (Rifadin), pyrazinamide (PZA), and ethambutol, a patient continues to have positive sputum smears for acid-fast bacilli (AFB). Which action should the nurse take next?

Ask the patient whether medications have been taken as directed.

Which action is a priority for the nurse to take when the low pressure alarm sounds for a patient who has an arterial line in the left radial artery?

Assess for cardiac dysrhythmias.

Which action by the nurse will determine if the therapies ordered for a patient with chronic constrictive pericarditis are effective?

Assess for the presence of jugular venous distention (JVD).

When assessing a patient with chronic obstructive pulmonary disease (COPD), the nurse finds a new onset of agitation and confusion. Which action should the nurse take first?

Assess oxygenation using pulse oximetry.

A 25-year-old male patient calls the clinic complaining of diarrhea for 24 hours. Which action should the nurse take first? a. Inform the patient that laboratory testing of blood and stools will be necessary. b. Ask the patient to describe the character of the stools and any associated symptoms. c. Suggest that the patient drink clear liquid fluids with electrolytes, such as Gatorade or Pedialyte. d. Advise the patient to use over-the-counter loperamide (Imodium) to slow gastrointestinal (GI) motility.

B

A 30-year-old patient seen in the emergency department for severe headache and acute confusion is found to have a serum sodium level of 118 mEq/L. The nurse will anticipate the need for which diagnostic test? a. Urinary 17-ketosteroids b. Antidiuretic hormone level c. Growth hormone stimulation test d. Adrenocorticotropic hormone level

B

A 33-year-old male patient with a gunshot wound to the abdomen undergoes surgery, and a colostomy is formed as shown in the accompanying figure. Which information will be included in patient teaching? a. Stool will be expelled from both stomas. b. This type of colostomy is usually temporary. c. Soft, formed stool can be expected as drainage. d. Irrigations can regulate drainage from the stomas.

B

A 35-year-old female patient with a possible pituitary adenoma is scheduled for a computed tomography (CT) scan with contrast media. Which patient information is most important for the nurse to communicate to the health care provider before the test? a. Bilateral poor peripheral vision b. Allergies to iodine and shellfish c. Recent weight loss of 20 pounds d. Complaint of ongoing headaches

B

A 40-year-old male patient has been newly diagnosed with type 2 diabetes mellitus. Which information about the patient will be most useful to the nurse who is helping the patient develop strategies for successful adaptation to this disease? a. Ideal weight b. Value system c. Activity level d. Visual changes

B

A 54-year-old critically ill patient with sepsis is frequently incontinent of watery stools. What action by the nurse will prevent complications associated with ongoing incontinence? a. Apply incontinence briefs. b. Use a fecal management system c. Insert a rectal tube with a drainage bag. d. Assist the patient to a commode frequently.

B

A 58-year-old man with blunt abdominal trauma from a motor vehicle crash undergoes peritoneal lavage. If the lavage returns brown fecal drainage, which action will the nurse plan to take next? a. Auscultate the bowel sounds. b. Prepare the patient for surgery. c. Check the patient's oral temperature. d. Obtain information about the accident.

B

A 71-year-old male patient tells the nurse that growing old causes constipation so he has been using a suppository for constipation every morning. Which action should the nurse take first? a. Encourage the patient to increase oral fluid intake. b. Assess the patient about risk factors for constipation. c. Suggest that the patient increase intake of high-fiber foods. d. Teach the patient that a daily bowel movement is unnecessary.

B

A 74-year-old patient preparing to undergo a colon resection for cancer of the colon asks about the elevated carcinoembryonic antigen (CEA) test result. The nurse explains that the test is used to a. identify any metastasis of the cancer. b. monitor the tumor status after surgery. c. confirm the diagnosis of a specific type of cancer. d. determine the need for postoperative chemotherapy.

B

A nurse will teach a patient who is scheduled to complete a 24-hour urine collection for 17-ketosteroids to a. insert and maintain a retention catheter. b. keep the specimen refrigerated or on ice. c. drink at least 3 L of fluid during the 24 hours. d. void and save that specimen to start the collection.

B

A patient being admitted with an acute exacerbation of ulcerative colitis reports crampy abdominal pain and passing 15 or more bloody stools a day. The nurse will plan to a. administer IV metoclopramide (Reglan). b. discontinue the patient's oral food intake. c. administer cobalamin (vitamin B12) injections. d. teach the patient about total colectomy surgery.

B

A patient in the emergency department has just been diagnosed with peritonitis caused by a ruptured diverticulum. Which prescribed intervention will the nurse implement first? a. Insert a urinary catheter to drainage. b. Infuse metronidazole (Flagyl) 500 mg IV. c. Send the patient for a computerized tomography scan. d. Place a nasogastric (NG) tube to intermittent low suction.

B

During routine hemodialysis, the 68-year-old patient complains of nausea and dizziness. Which action should the nurse take first?

Check patients blood pressure (BP).

After a total proctocolectomy and permanent ileostomy, the patient tells the nurse, "I cannot manage all these changes. I don't want to look at the stoma." What is the best action by the nurse? a. Reassure the patient that ileostomy care will become easier. b. Ask the patient about the concerns with stoma management. c. Develop a detailed written list of ostomy care tasks for the patient. d. Postpone any teaching until the patient adjusts to the ileostomy.

B

After change-of-shift report, which patient should the nurse assess first? a. 40-year-old male with celiac disease who has frequent frothy diarrhea b. 30-year-old female with a femoral hernia who has abdominal pain and vomiting c. 30-year-old male with ulcerative colitis who has severe perianal skin breakdown d. 40-year-old female with a colostomy bag that is pulling away from the adhesive wafer

B

During the physical examination of a 36-year-old female, the nurse finds that the patient's thyroid gland cannot be palpated. The most appropriate action by the nurse is to a. palpate the patient's neck more deeply. b. document that the thyroid was nonpalpable. c. notify the health care provider immediately. d. teach the patient about thyroid hormone testing.

B

The nurse is assessing a 31-year-old female patient with abdominal pain. Th nurse,who notes that there is ecchymosis around the area of umbilicus, will document this finding as a. Cullen sign. b. Rovsing sign. c. McBurney sign. d. Grey-Turner's signt.

B

The nurse is caring for a 45-year-old male patient during a water deprivation test. Which finding is most important for the nurse to communicate to the health care provider? a. The patient complains of intense thirst. b. The patient has a 5-lb (2.3 kg) weight loss. c. The patient's urine osmolality does not increase. d. The patient feels dizzy when sitting on the edge of the bed.

B

The nurse preparing for the annual physical exam of a 50-year-old man will plan to teach the patient about a. endoscopy. b. colonoscopy. c. computerized tomography screening. d. carcinoembryonic antigen (CEA) testing.

B

The nurse will determine that teaching a 67-year-old man to irrigate his new colostomy has been effective if the patient a. inserts the irrigation tubing 4 to 6 inches into the stoma. b. hangs the irrigating container 18 inches above the stoma. c. stops the irrigation and removes the irrigating cone if cramping occurs. d. fills the irrigating container with 1000 to 2000 mL of lukewarm tap water.

B

To evaluate the effectiveness of the pantoprazole (Protonix) ordered for a patient with systemic inflammatory response syndrome (SIRS), which assessment will the nurse perform?

Check stools for occult blood.

Which information will the nurse include in teaching a patient who had a proctocolectomy and ileostomy for ulcerative colitis? a. Restrict fluid intake to prevent constant liquid drainage from the stoma. b. Use care when eating high-fiber foods to avoid obstruction of the ileum. c. Irrigate the ileostomy daily to avoid having to wear a drainage appliance. d. Change the pouch every day to prevent leakage of contents onto the skin.

B

Which information will the nurse teach a 23-year-old patient with lactose intolerance? a. Ice cream is relatively low in lactose. b. Live-culture yogurt is usually tolerated. c. Heating milk will break down the lactose. d. Nonfat milk is a better choice than whole milk.

B

Which nursing action will the nurse include in the plan of care for a 35-year-old male patient admitted with an exacerbation of inflammatory bowel disease (IBD)? a. Restrict oral fluid intake. b. Monitor stools for blood. c. Ambulate four times daily. d. Increase dietary fiber intake.

B

Which patient statement indicates that the nurse's teaching about sulfasalazine (Azulfidine) for ulcerative colitis has been effective? a. "The medication will be tapered if I need surgery." b. "I will need to use a sunscreen when I am outdoors." c. "I will need to avoid contact with people who are sick." d. "The medication will prevent infections that cause the diarrhea."

B

You respond to a call for help from the ED waiting room. There is an elderly patient lying on the floor. List the order for the actions that you must perform. a. Perform the chin lift or jaw thrust maneuver. b. Establish unresponsiveness. c. Initiate cardiopulmonary resuscitation (CPR). d. Call for help and activate the code team. e. Instruct a nursing assistant to get the crash cart. A. A B C E D B. B D A C E C. C A B E D D. D C B E A E. E C D B A

B

The health care provider suspects the Somogyi effect in a 50-year-old patient whose 6:00 AM blood glucose is 230 mg/dL. Which action will the nurse teach the patient to take?

Check the blood glucose during the night

Physiological Integrity 12. A 53-year-old patient with Stage D heart failure and type 2 diabetes asks the nurse whether heart transplant is a possible therapy. Which response by the nurse is most appropriate? a. "Because you have diabetes, you would not be a candidate for a heart transplant." b. "The choice of a patient for a heart transplant depends on many different factors." c. "Your heart failure has not reached the stage in which heart transplants are needed." d. "People who have heart transplants are at risk for multiple complications after surgery."

B Indications for a heart transplant include end-stage heart failure (Stage D), but other factors such as coping skills, family support, and patient motivation to follow the rigorous posttransplant regimen are also considered. Diabetic patients who have well-controlled blood glucose levels may be candidates for heart transplant. Although heart transplants can be associated with many complications, this response does not address the patient's question. DIF: Cognitive Level: Apply (application) REF: 783 TOP: Nursing Process: Implementation MSC:

Safe and Effective Care Environment 25. After receiving change-of-shift report on a heart failure unit, which patient should the nurse assess first? a. Patient who is taking carvedilol (Coreg) and has a heart rate of 58 b. Patient who is taking digoxin and has a potassium level of 3.1 mEq/L c. Patient who is taking isosorbide dinitrate/hydralazine (BiDil) and has a headache d. Patient who is taking captopril (Capoten) and has a frequent nonproductive cough

B The patient's low potassium level increases the risk for digoxin toxicity and potentially fatal dysrhythmias. The nurse should assess the patient for other signs of digoxin toxicity and then notify the health care provider about the potassium level. The other patients also have side effects of their medications, but their symptoms do not indicate potentially life-threatening complications. DIF: Cognitive Level: Analyze (analysis) REF: 777 OBJ: Special Questions: Prioritization; Multiple Patients TOP: Nursing Process: Assessment MSC:

The nurse is caring for a patient admitted with a history of hypertension. The patient's medication history includes hydrochlorothiazide (Hydrodiuril) daily for the past 10 years. Which parameter would indicate the optimal intended effect of this drug therapy? A Weight loss of 2 lb B Blood pressure 128/86 C Absence of ankle edema D Output of 600 mL per 8 hours

B Blood pressure 128/86 Hydrochlorothiazide may be used alone as monotherapy to manage hypertension or in combination with other medications if not effective alone. After the first few weeks of therapy, the diuretic effect diminishes, but the antihypertensive effect remains. Since the patient has been taking this medication for 10 years, the most direct measurement of its intended effect would be the blood pressure.

The nurse admits a 73-year-old male patient with dementia for treatment of uncontrolled hypertension. The nurse will closely monitor for hypokalemia if the patient receives which medication? A Clonidine (Catapres) B Bumetanide (Bumex) C Amiloride (Midamor) D Spironolactone (Aldactone)

B Bumetanide (Bumex) Bumetanide is a loop diuretic. Hypokalemia is a common adverse effect of this medication. Amiloride is a potassium-sparing diuretic. Spironolactone is an aldosterone-receptor blocker. Hyperkalemia is an adverse effect of both amiloride and spironolactone. Clonidine is a central-acting α-adrenergic antagonist and does not cause electrolyte abnormalities.

When providing dietary instruction to a patient with hypertension, the nurse would advise the patient to restrict intake of which meat? A Broiled fish B Roasted duck C Roasted turkey D Baked chicken breast

B Roasted duck Roasted duck is high in fat, which should be avoided by the patient with hypertension. Weight loss may slow the progress of atherosclerosis and overall CVD risk. The other meats are lower in fat and are therefore acceptable in the diet.

A 44-year-old patient is admitted with tetany. Which laboratory value should the nurse monitor? a. Total protein b. Blood glucose c. Ionized calcium d. Serum phosphate

C

A 60-year-old patient is taking spironolactone (Aldactone), a drug that blocks the action of aldosterone on the kidney, for hypertension. The nurse will monitor for a. increased serum sodium. b. decreased urinary output. c. elevated serum potassium. d. evidence of fluid overload.

C

Which action could the nurse delegate to unlicensed assistive personnel (UAP) trained as electrocardiogram (ECG) technicians working on the cardiac unit?

Check the heart monitor for changes in rhythm while a patient who had a valve replacement ambulates

A patient with acute kidney injury (AKI) has longer QRS intervals on the electrocardiogram (ECG) than were noted on the previous shift. Which action should the nurse take first?

Check the medical record for most recent potassium level.

Which information will the nurse include in teaching a female patient who has peripheral arterial disease, type 2 diabetes, and sensory neuropathy of the feet and legs?

Choose flat-soled leather shoes.

When teaching a patient about dietary management of stage 1 hypertension, which instruction is most appropriate? A Restrict all caffeine. B Restrict sodium intake. C Increase protein intake. D Use calcium supplements.

B Restrict sodium intake. The patient should decrease intake of sodium. This will help to control hypertension, which can be aggravated by excessive salt intake, which in turn leads to fluid retention. Caffeine and protein intake do not affect hypertension. Calcium supplements are not recommended to lower BP.

The nurse is assessing a patient with myocarditis before administering the scheduled dose of digoxin (Lanoxin). Which finding is most important for the nurse to communicate to the health care provider?

Irregular pulse

When assessing the patient for orthostatic hypotension, after taking the blood pressure (BP) and pulse (P) in the supine position, what should the nurse do next? A Repeat BP and P in this position. B Take BP and P with patient sitting. C Record the BP and P measurements. D Take BP and P with patient standing.

B Take BP and P with patient sitting. When assessing for orthostatic changes in BP after measuring BP in the supine position, the patient is placed in a sitting position and BP is measured within 1 to 2 minutes and then repositioned to the standing position with BP measured again, within 1 to 2 minutes. The results are then recorded with a decrease of 20 mm Hg or more in SBP, a decrease of 10 mm Hg or more in DBP, and/or an increase in pulse of greater than or equal to 20 beats/minute from supine to standing indicating orthostatic hypotension.

The nurse supervises an unlicensed assistant personnel (UAP) who is taking the blood pressure of 58-year-old female patient admitted with heart failure. The patient is obese. The nurse should intervene if what is observed? A The UAP waits 2 minutes after position changes to take orthostatic pressures. B The UAP deflates the blood pressure cuff at a rate of 8 to 10 mm Hg per second. C The UAP takes the blood pressure with the patient's arm at the level of the heart. D The UAP takes a forearm blood pressure because the largest cuff will not fit the patient's upper arm.

B The UAP deflates the blood pressure cuff at a rate of 8 to 10 mm Hg per second. The cuff should be deflated at a rate of 2 to 3 mm Hg per second. The arm should be supported at the level of the heart for accurate blood pressure measurements. If the maximum size blood pressure cuff does not fit the upper arm, the forearm may be used. Orthostatic blood pressures should be taken within 1 to 2 minutes of repositioning the patient.

The nurse is admitting a 67-year-old patient with new-onset steatorrhea. Which question is most important for the nurse to ask? a. "How much milk do you usually drink?" b. "Have you noticed a recent weight loss?" c. "What time of day do your bowels move?" d. "Do you eat meat or other animal products?"

B characterized by bulky stool that floats and is fat loss through the stools. There for weight loss would be highly suspected

A new 19-year-old male patient has familial adenomatous polyposis (FAP). Which action will the nurse in the gastrointestinal clinic include in the plan of care? a. Obtain blood samples for DNA analysis. b. Schedule the patient for yearly colonoscopy. c. Provide preoperative teaching about total colectomy. d. Discuss lifestyle modifications to decrease cancer risk.

B since this disorder runs in his family he she be screened yearly by colonoscopy for any new formation of polyposis

Which prescribed intervention for a 61-year-old female patient with chronic short bowel syndrome will the nurse question? a. Ferrous sulfate (Feosol) 325 mg daily b. Senna (Senokot) 1 tablet every day c. Psyllium (Metamucil) 2.1 grams 3 times daily d. Diphenoxylate with atropine (Lomotil) prn loose stools

B the primary manifestation for this client is diarrhea providing a stool softener would be dangerous

A 62-year-old patient has had a hemorrhoidectomy at an outpatient surgical center. Which instructions will the nurse include in discharge teaching? a. Maintain a low-residue diet until the surgical area is healed. b. Use ice packs on the perianal area to relieve pain and swelling. c. Take prescribed pain medications before a bowel movement is expected. d. Delay having a bowel movement for several days until healing has occurred.

C

When establishing a diagnosis of MS, the nurse should teach the patient about what diagnostic studies (select all that apply)? A. EEG B. CT scan C. Carotid duplex scan D. Evoked response testing E. Cerebrospinal fluid analysis

B, D, E There is no definitive diagnostic test for MS. CT scan, evoked response testing, cerebrospinal fluid analysis, and MRI along with history and physical examination are used to establish a diagnosis for MS. EEG and carotid duplex scan are not used for diagnosing MS.

The nurse provides information to the caregiver of a 68-year-old man with epilepsy who has tonic-clonic seizures. Which statement, if made by the caregiver, requires further teaching? A. "It is normal for a person to be sleepy after a seizure." B. "I should call 911 if breathing stops during the seizure." C. "The jerking movements may last for 30 to 40 seconds." D. "Objects should not be placed in the mouth during a seizure."

B. Caregivers do not need to call an ambulance or send a person to the hospital after a single seizure unless the seizure is prolonged, another seizure immediately follows, or extensive injury has occurred. Altered breathing is a clinical manifestation of a tonic-clonic seizure. Contact emergency medical services (or call 911) if breathing stops for more than 30 seconds. No objects (e.g., oral airway, padded tongue blade) should be placed in the mouth. Lethargy is common in the postictal phase of a seizure. Jerking of the extremities occurs during the clonic phase of a tonic-clonic seizure. The clonic phase may last 30 to 40 seconds.

A patient has been receiving scheduled doses of phenytoin (Dilantin) and begins to experience diplopia. The nurse immediately assesses the patient for A. an aura or focal seizure. B. nystagmus or confusion. C. abdominal pain or cramping. D. irregular pulse or palpitations.

B. Diplopia is a sign of phenytoin toxicity. The nurse should assess for other signs of toxicity, which include neurologic changes, such as nystagmus, ataxia, confusion, dizziness, or slurred speech. An aura, focal seizure, abdominal pain or cramping, irregular pulse, or palpitations are not associated with phenytoin toxicity.

The nurse observes a 74-year-old man with Parkinson's disease rocking side to side while sitting in the chair. Which action by the nurse is most appropriate? A. Provide the patient with diversional activities. B. Document the activity in the patient's health record. C. Take the patient's blood pressure sitting and standing. D. Ask if the patient is feeling either anxious or depressed.

B. Patients with Parkinson's disease are instructed to rock from side to side to stimulate balance mechanisms and decrease akinesia.

A 50-year-old male patient has been diagnosed with amyotrophic lateral sclerosis (ALS). What nursing intervention is most important to help prevent a common cause of death for patients with ALS? A. Reduce fat intake. B. Reduce the risk of aspiration. C. Decrease injury related to falls. D. Decrease pain secondary to muscle weakness.

B. Reducing the risk of aspiration can help prevent respiratory infections that are a common cause of death from deteriorating muscle function. Reducing fat intake may reduce cardiovascular disease, but this is not a common cause of death for patients with ALS. Decreasing injury related to falls and decreasing pain secondary to muscle weakness are important nursing interventions for patients with ALS but are unrelated to causes of death for these patients.

The home health registered nurse (RN) is planning care for a patient with a seizure disorder related to a recent head injury. Which nursing action can be delegated to a licensed practical/vocational nurse (LPN/LVN)? a. Make referrals to appropriate community agencies. b. Place medications in the home medication organizer. c. Teach the patient and family how to manage seizures. d. Assess for use of medications that may precipitate seizures.

B. LPN/LVN education includes administration of medications. The other activities require RN education and scope of practice.

Which intervention will the nurse include in the plan of care for a patient with primary restless legs syndrome (RLS) who is having difficulty sleeping? a. Teach about the use of antihistamines to improve sleep. b. Suggest that the patient exercise regularly during the day. c. Make a referral to a massage therapist for deep massage of the legs. d. Assure the patient that the problem is transient and likely to resolve.

B. Nondrug interventions such as getting regular exercise are initially suggested to improve sleep quality in patients with RLS. Antihistamines may aggravate RLS. Massage does not alleviate RLS symptoms and RLS is likely to progress in most patients.

A 73-year-old patient with Parkinson's disease has a nursing diagnosis of impaired physical mobility related to bradykinesia. Which action will the nurse include in the plan of care? a. Instruct the patient in activities that can be done while lying or sitting. b. Suggest that the patient rock from side to side to initiate leg movement. c. Have the patient take small steps in a straight line directly in front of the feet. d. Teach the patient to keep the feet in contact with the floor and slide them forward.

B. Rocking the body from side to side stimulates balance and improves mobility. The patient will be encouraged to continue exercising because this will maintain functional abilities. Maintaining a wide base of support will help with balance. The patient should lift the feet and avoid a shuffling gait.

Which drug treatment helps to decrease ICP by expanding plasma and the osmotic effect to move fluid?

Mannitol (Osmitrol) (25%)

The nurse teaches a 38-year-old woman who has migraine headaches about sumatriptan (Imitrex). Which statement by the patient requires an intervention by the nurse? A. "The injection might feel like a bee sting." B. "This medicine will prevent a migraine headache." C. "I can take another dose if the first does not work." D. "This drug for migraine headaches could cause birth defects."

B. Sumatriptan is given to abort an ongoing migraine headache and is not used to prevent migraine headaches. When given as a subcutaneous injection, this drug may cause transient pain and redness at the injection site. This drug may be repeated after a specified time period if the first dose is not effective. This drug should be avoided during pregnancy and is classified as an FDA Pregnancy Risk Category C drug.

Which action will the nurse plan to take for a 40-year-old patient with multiple sclerosis (MS) who has urinary retention caused by a flaccid bladder? a. Decrease the patient's evening fluid intake. b. Teach the patient how to use the Credé method. c. Suggest the use of adult incontinence briefs for nighttime only. d. Assist the patient to the commode every 2 hours during the day.

B. The Credé method can be used to improve bladder emptying. Decreasing fluid intake will not improve bladder emptying and may increase risk for urinary tract infection (UTI) and dehydration. The use of incontinence briefs and frequent toileting will not improve bladder emptying.

When a 74-year-old patient is seen in the health clinic with new development of a stooped posture, shuffling gait, and pill rolling-type tremor, the nurse will anticipate teaching the patient about a. oral corticosteroids. b. antiparkinsonian drugs. c. magnetic resonance imaging (MRI). d. electroencephalogram (EEG) testing.

B. The diagnosis of Parkinson's is made when two of the three characteristic manifestations of tremor, rigidity, and bradykinesia are present. The confirmation of the diagnosis is made on the basis of improvement when antiparkinsonian drugs are administered. This patient has symptoms of tremor and bradykinesia. The next anticipated step will be treatment with medications. MRI and EEG are not useful in diagnosing Parkinson's disease, and corticosteroid therapy is not used to treat it.

Which characteristic of a patient's recent seizure is consistent with a focal seizure? A. The patient lost consciousness during the seizure. B. The seizure involved lip smacking and repetitive movements. C. The patient fell to the ground and became stiff for 20 seconds. D. The etiology of the seizure involved both sides of the patient's brain.

B. The most common complex focal seizure involves lip smacking and automatisms (repetitive movements that may not be appropriate). Loss of consciousness, bilateral brain involvement, and a tonic phase are associated with generalized seizure activity.

A high school teacher who has just been diagnosed with epilepsy after having a generalized tonic-clonic seizure tells the nurse, "I cannot teach anymore, it will be too upsetting if I have a seizure at work." Which response by the nurse is best? a. "You might benefit from some psychologic counseling." b. "Epilepsy usually can be well controlled with medications." c. "You will want to contact the Epilepsy Foundation for assistance." d. "The Department of Vocational Rehabilitation can help with work retraining."

B. The nurse should inform the patient that most patients with seizure disorders are controlled with medication. The other information may be necessary if the seizures persist after treatment with antiseizure medications is implemented.

Following a thymectomy, a 62-year-old male patient with myasthenia gravis receives the usual dose of pyridostigmine (Mestinon). An hour later, the patient complains of nausea and severe abdominal cramps. Which action should the nurse take first? a. Auscultate the patient's bowel sounds. b. Notify the patient's health care provider. c. Administer the prescribed PRN antiemetic drug. d. Give the scheduled dose of prednisone (Deltasone).

B. The patient's history and symptoms indicate a possible cholinergic crisis. The health care provider should be notified immediately, and it is likely that atropine will be prescribed. The other actions will be appropriate if the patient is not experiencing a cholinergic crisis.

A hospitalized patient complains of a bilateral headache, 4/10 on the pain scale, that radiates from the base of the skull. Which prescribed PRN medications should the nurse administer initially? a. Lorazepam (Ativan) b. Acetaminophen (Tylenol) c. Morphine sulfate (Roxanol) d. Butalbital and aspirin (Fiorinal)

B. The patient's symptoms are consistent with a tension headache, and initial therapy usually involves a nonopioid analgesic such as acetaminophen, which is sometimes combined with a sedative or muscle relaxant. Lorazepam may be used in conjunction with acetaminophen but would not be appropriate as the initial monotherapy. Morphine sulfate and butalbital and aspirin would be more appropriate for a headache that did not respond to a nonopioid analgesic.

The health care provider is considering the use of sumatriptan (Imitrex) for a 54-year-old male patient with migraine headaches. Which information obtained by the nurse is most important to report to the health care provider? a. The patient drinks 1 to 2 cups of coffee daily. b. The patient had a recent acute myocardial infarction. c. The patient has had migraine headaches for 30 years. d. The patient has taken topiramate (Topamax) for 2 months.

B. The triptans cause coronary artery vasoconstriction and should be avoided in patients with coronary artery disease. The other information will be reported to the health care provider, but none of it indicates that sumatriptan would be an inappropriate treatment.

The nurse will assess a 67-year-old patient who is experiencing a cluster headache for a. nuchal rigidity. b. unilateral ptosis. c. projectile vomiting. d. throbbing, bilateral facial pain.

B. Unilateral eye edema, tearing, and ptosis are characteristic of cluster headaches. Nuchal rigidity suggests meningeal irritation, such as occurs with meningitis. Although nausea and vomiting may occur with migraine headaches, projectile vomiting is more consistent with increased intracranial pressure (ICP). Unilateral sharp, stabbing pain, rather than throbbing pain, is characteristic of cluster headaches.

When obtaining a health history and physical assessment for a 36-year-old female patient with possible multiple sclerosis (MS), the nurse should a. assess for the presence of chest pain. b. inquire about urinary tract problems. c. inspect the skin for rashes or discoloration. d. ask the patient about any increase in libido.

B. Urinary tract problems with incontinence or retention are common symptoms of MS. Chest pain and skin rashes are not symptoms of MS. A decrease in libido is common with MS.

Which nursing action can the nurse delegate to unlicensed assistive personnel (UAP) who are working in the diabetic clinic?

Measure the ankle-brachial index.

The nurse is caring for a patient with chronic obstructive pulmonary disorder (COPD) and pneumonia who has an order for arterial blood gases to be drawn. What is the minimum length of time the nurse should plan to hold pressure on the puncture site? A. 2 minutes B. 5 minutes C. 10 minutes D. 15 minutes

B. 5 minutes After obtaining blood for an arterial blood gas measurement, the nurse should hold pressure on the puncture site for 5 minutes by the clock to be sure that bleeding has stopped. An artery is an elastic vessel under much higher pressure than veins, and significant blood loss or hematoma formation could occur if the time is insufficient.

Which patient is exhibiting an early clinical manifestation of hypoxemia? A. A 48-year-old patient who is intoxicated and acutely disoriented to time and place B. A 72-year-old patient who has four new premature ventricular contractions per minute C. A 67-year-old patient who has dyspnea while resting in the bed or in a reclining chair D. A 94-year-old patient who has renal insufficiency, anemia, and decreased urine output

B. A 72-year-old patient who has four new premature ventricular contractions per minute Early clinical manifestations of hypoxemia include dysrhythmias (e.g., premature ventricular contractions), unexplained decreased level of consciousness (e.g., disorientation), dyspnea on exertion, and unexplained decreased urine output.

A patient with a possible urinary tract infection (UTI) gives the nurse in the clinic a urine specimen that is a red-orange color. Which action should the nurse take first? A. Notify the patient's health care provider. B. Ask the patient about use of any medications. C. Question the patient about any UTI risk factors. D. Teach about the correct procedure for midstream urine collection.

B. Ask the patient about use of any medications. A red-orange color in the urine is normal with some over-the-counter (OTC) medications such as phenazopyridine (Pyridium). The color would not be expected with urinary tract infection, is not a sign that poor technique was used in obtaining the specimen, & does not need to be communicated to the health care provider until further assessment is done.

When assessing the accessory nerve, what should the nurse do? A. Assess the gag reflex by stroking the posterior pharynx. B. Ask the patient to shrug the shoulders against resistance. C. Ask the patient to push the tongue to either side against resistance. D. Have the patient say "ah" while visualizing elevation of soft palate.

B. Ask the patient to shrug the shoulders against resistance. The spinal accessory nerve is tested by asking the patient to shrug the shoulders against resistance and to turn the head to either side against resistance while observing the sternocleidomastoid muscles and the trapezius muscles. Assessing the gag reflex and saying "ah" are used to assess the glossopharyngeal and vagus nerves. Asking the patient to push the tongue to either side against resistance and to stick out the tongue are used to assess the hypoglossal nerve.

How should the nurse most accurately assess the position sense of a patient with a recent traumatic brain injury? A. Ask the patient to close his or her eyes and slowly bring the tips of the index fingers together. B. Ask the patient to stand with the feet together and eyes closed and observe for balance maintenance. C. Ask the patient to close his or her eyes and identify the presence of a common object on the forearm. D. Place the two points of a calibrated compass on the tips of the fingers and toes and ask the patient to discriminate the points.

B. Ask the patient to stand with the feet together and eyes closed and observe for balance maintenance. The Romberg test is an assessment of position sense in which the patient stands with the feet together and then closes his or her eyes while attempting to maintain balance. The other cited tests of neurologic function do not directly assess position sense.

The patient had abdominal surgery yesterday. Today the lung sounds in the lower lobes have decreased. The nurse knows this could be due to what occurring? A. Pain B. Atelectasis C. Pneumonia D. Pleural effusion

B. Atelectasis Postoperatively there is an increased risk for atelectasis from anesthesia as well as restricted breathing from pain. Without deep breathing to stretch the alveoli, surfactant secretion to hold the alveoli open is not promoted. Pneumonia will occur later after surgery. Pleural effusion occurs because of blockage of lymphatic drainage or an imbalance between intravascular and oncotic fluid pressures, which is not expected in this case

While assessing a patient's urinary system, the nurse cannot palpate either kidney. Which action should the nurse take next? A. Obtain a urine specimen to check for hematuria. B. Document the information on the assessment form. C. Ask the patient about any history of recent sore throat. D. Ask the health care provider about scheduling a renal ultrasound.

B. Document the information on the assessment form. The kidneys are protected by the abdominal organs, ribs, and muscles of the back and may not be palpable under normal circumstances, so no action except to document the assessment information is needed.

A creatinine clearance test is ordered for a hospitalized patient with possible renal insufficiency. Which equipment will the nurse need to obtain? A. Sterile specimen cup B. Large container for urine C. Foley catheter and drainage bag D. Towelettes for perineal cleaning

B. Large container for urine Since creatinine clearance testing involves a 24-hour urine specimen, the nurse should obtain a large container for the urine collection. Catheterization, cleaning of the perineum with antiseptic towelettes, and a sterile specimen cup are not needed for this test.

An 82-year-old man has been admitted with benign prostatic hyperplasia. Which of the following is most appropriate to include in the nursing plan of care? A. Limit fluid intake to no more than 1500 mL/day. Incorrect B. Leave a light on in the bathroom during the night. C. Pad the patient's bed to accommodate overflow incontinence. D. Ask the patient to use a urinal so that all urine can be measured.

B. Leave a light on in the bathroom during the night The patient's age and diagnosis indicate a likelihood of nocturia, so leaving the light on in the bathroom is appropriate. Fluids should be encouraged because dehydration is more common in older patients. The information in the question does not indicate that measurement of the patient's output is necessary or that the patient has overflow incontinence.

After assisting at the bedside with a thoracentesis, the nurse should continue to assess the patient for signs and symptoms of what? A. Bronchospasm B. Pneumothorax C. Pulmonary edema D. Respiratory acidosis

B. Pneumothorax Because thoracentesis involves the introduction of a catheter into the pleural space, there is a risk of pneumothorax. Thoracentesis does not carry a significant potential for causing bronchospasm, pulmonary edema, or respiratory acidosis.

Which normal nervous system changes of aging put the geriatric person at higher risk of falls (select all that apply)? A. Memory deficit B. Sensory deficit C. Motor function deficit D. Cranial and spinal nerves E. Reticular activation system F. Central nervous system changes

B. Sensory deficit C. Motor function deficit F. Central nervous system changes An older person is at a higher risk for falls because the changes in the nervous system decrease the sensory function that leads to poor ability to maintain balance and a widened gait. The motor function deficit decreases muscle strength and agility. The central nervous system changes in the brain lead to a diminished kinesthetic sense or position sense. Memory deficits, normal changes of cranial and spinal nerves, and the reticular activation system do not contribute to the increased risk of falls.

Following an intravenous pyelogram (IVP), all of the following assessment data are obtained. Which one requires immediate action by the nurse? A. The heart rate is 58 beats/minute. B. The respiratory rate is 38 breaths/minute. D. The patient complains of a dry mouth. D. The urine output is 400 mL in the first 2 hours.

B. The respiratory rate is 38 breaths/minute. The increased respiratory rate indicates that the patient may be experiencing an allergic reaction to the contrast medium used during the procedure. The nurse should immediately assess the patient's oxygen saturation & breath sounds. The other data are not unusual findings following an IVP.

In assessment of the patient with acute respiratory distress, what should the nurse expect to observe (select all that apply)? A. Cyanosis B. Tripod position C. Kussmaul respirations D. Accessory muscle use E. Increased AP diameter

B. Tripod position D. Accessory muscle use Tripod position and accessory muscle use indicate moderate to severe respiratory distress. Cyanosis may be related to anemia, decreased oxygen transfer in the lungs, or decreased cardiac output. Therefore it is a nonspecific and unreliable indicator of only respiratory distress. Kussmaul respirations occur when the patient is in metabolic acidosis to increase CO2 excretion. Increased AP diameter occurs with lung hyperinflation from COPD, cystic fibrosis, or with advanced age.

To detect early signs of symptoms of inadequate oxygenation, the nurse would examine the patient for: A. dyspnea and hypotension. B. apprehension and restlessness. C. cyanosis and cool, clammy skin. D. increased urine output and diaphoresis.

B. apprehension and restlessness. Early symptoms of inadequate oxygenation include unexplained restlessness, apprehension, and irritability.

When auscultating the chest of an older patient in respiratory distress, it is best to: A. begin listening at the apices. B. begin listening at the lung bases. C. begin listening on the anterior chest. D. ask the patient to breathe through the nose with the mouth closed.

B. begin listening at the lung bases. Normally, auscultation should proceed from the lung apices to the bases, so that opposite areas of the chest are compared. If the patient is likely to tire easily or has respiratory distress, start at the bases.

The nurse is caring for a patient who has an intra aortic balloon pump in place. Which action should be included in the plan of care?

Measure the patients urinary output every hour.

The nurse is caring for a patient with peripheral neuropathy who is going to have EMG studies tomorrow morning. The nurse should: A. ensure the patient has an empty bladder. B. instruct the patient that there is no risk of electric shock. C. ensure the patient has no metallic jewelry or metal fragments. D. instruct the patient that she or he may experience pain during the study.

B. instruct the patient that there is no risk of electric shock. Electromyography (EMG) is used to assess electrical activity associated with nerves and skeletal muscles. Activity is recorded by insertion of needle electrodes to detect muscle and peripheral nerve disease. The nurse should inform the patient that pain and discomfort are associated with insertion of needles. There is no risk of electric shock with this procedure.

If patient had Peripheral Artery Disease (PAD), why would they have thin, shiny skin, and hair loss?

Because the rich healthy oxygenated blood cannot get to the legs

A nurse is caring for a client undergoing screening for primary Cushing's disease. The nurse should expect that which of the following laboratory findings to be elevated? A. Lymphocyte B. Serum Potassium C. Serum Calcium D. Blood Glucose

Blood Glucose

A patient has just been admitted with probable bacterial pneumonia and sepsis. Which order should the nurse implement first?

Blood cultures from two sites

A nurse is caring for a client who has type 2 DM and is admitted with hyperglycemic-hyperosmolar state (HHS). Which of the following laboratory findings should the nurse expect? A. Blood glucose of 496 mg/dL and serum pH of 7.32 B. Blood glucose of 550 mg/dL and serum pH of 7.02 C. Blood glucose of 702 mg/dL and serum pH of 6.11 D. Blood glucose of 846 mg/dL and serum pH of 7.40

Blood glucose of 846 mg/dL and serum pH of 7.40 (Enough insulin to prevent ketosis, but not enough to prevent hyperglycemia -> serum pH is normal, glucose is elevated)

An active 28-year-old male with type 1 diabetes is being seen in the endocrine clinic. Which finding may indicate the need for a change in therapy?

Blood pressure 146/88 mmHg

Which assessment finding for a patient who has just been admitted with acute pyelonephritis is most important for the nurse to report to the health care provider?

Blood pressure 90/48 mm Hg

Which assessment finding for a patient who has just been admitted with acute pyelonephritis ismost important for the nurse to report to the health care provider?

Blood pressure 90/48 mm Hg

Which is most important to respond to in a patient presenting with a T3 spinal injury?

Blood pressure of 88/60 mm Hg, pulse of 56 beats/minute

A nurse is providing discharge teaching for a client who has DI and has a new scrip for desmopressin nasal spray. Which of the following instructions should the nurse include in the teaching? A. Breath deeply while using the nasal spray B, Blow nose gently prior to using the nasal spray. C. Administer the spray while in a side-lying position. D. Instill the med 4x per day.

Blow the nose gently prior to using the nasal spray. (avoids dilution of meds by nasal secretions or improper absorption due to blockage)

The nurse is caring for a patient admitted with a subdural hematoma after a motor vehicle accident. What change in vital signs would the nurse interpret as a manifestation of increased intracranial pressure (ICP)?

Bradycardia

A patient is admitted to the ED after falling through the ice while ice skating. Which assessment will the nurse obtain first?

Breath sounds

A 54 year old patient arrives in the ED after exposure to powdered lime at work. Which action should the nurse take first?

Brush off any visible powder on the skin and clothing

Which assessment data reported by a 28-year-old male patient is consistent with a lower urinary tract infection (UTI)?

Burning on urination

A 22-year-old female patient with an exacerbation of ulcerative colitis is having 15 to 20 stools daily and has excoriated perianal skin. Which patient behavior indicates that teaching regarding maintenance of skin integrity has been effective? a. The patient uses incontinence briefs to contain loose stools. b. The patient asks for antidiarrheal medication after each stool. c. The patient uses witch hazel compresses to decrease irritation. d. The patient cleans the perianal area with soap after each stool.

C

A 26-year-old woman is being evaluated for vomiting and abdominal pain. Which question from the nurse will be most useful in determining the cause of the patient's symptoms? a. "What type of foods do you eat?" b. "Is it possible that you are pregnant?" c. "Can you tell me more about the pain?" d. "What is your usual elimination pattern?"

C

A 29-year-old patient in the outpatient clinic will be scheduled for blood cortisol testing. Which instruction will the nurse provide? a. "Avoid adding any salt to your foods for 24 hours before the test." b. "You will need to lie down for 30 minutes before the blood is drawn." c. "Come to the laboratory to have the blood drawn early in the morning." d. "Do not have anything to eat or drink before the blood test is obtained."

C

A 72-year-old male patient with dehydration caused by an exacerbation of ulcerative colitis is receiving 5% dextrose in normal saline at 125 mL/hour. Which assessment finding by the nurse is most important to report to the health care provider? a. Patient has not voided for the last 4 hours. b. Skin is dry with poor turgor on all extremities. c. Crackles are heard halfway up the posterior chest. d. Patient has had 5 loose stools over the last 6 hours.

C

A 76-year-old patient with obstipation has a fecal impaction and is incontinent of liquid stool. Which action should the nurse take first? a. Administer bulk-forming laxatives. b. Assist the patient to sit on the toilet. c. Manually remove the impacted stool. d. Increase the patient's oral fluid intake.

C

The nurse is providing preoperative teaching for a 61-year-old man scheduled for an abdominal-perineal resection. Which information will the nurse include? a. Another surgery in 8 to 12 weeks will be used to create an ileal-anal reservoir. b. The patient will begin sitting in a chair at the bedside on the first postoperative day. c. The patient will drink polyethylene glycol lavage solution (GoLYTELY) preoperatively. d. IV antibiotics will be started at least 24 hours before surgery to reduce the bowel bacteria.

C

The nurse reviews a patient's glycosylated hemoglobin (Hb A1C) results to evaluate a. fasting preprandial glucose levels. b. glucose levels 2 hours after a meal. c. glucose control over the past 90 days. d. hypoglycemic episodes in the past 3 months.

C

The nurse will teach a patient to plan to minimize physical and emotional stress while the patient is undergoing a. a water deprivation test. b. testing for serum T3 and T4 levels. c. a 24-hour urine test for free cortisol. d. a radioactive iodine (I-131) uptake test.

C

Which activity in the care of a 48-year-old female patient with a new colostomy could the nurse delegate to unlicensed assistive personnel (UAP)? a. Document the appearance of the stoma. b. Place a pouching system over the ostomy. c. Drain and measure the output from the ostomy. d. Check the skin around the stoma for breakdown.

C

Which additional information will the nurse need to consider when reviewing the laboratory results for a patient's total calcium level? a. The blood glucose is elevated. b. The phosphate level is normal. c. The serum albumin level is low. d. The magnesium level is normal.

C

Which care activity for a patient with a paralytic ileus is appropriate for the registered nurse (RN) to delegate to unlicensed assistive personnel (UAP)? a. Auscultation for bowel sounds b. Nasogastric (NG) tube irrigation c. Applying petroleum jelly to the lips d. Assessment of the nares for irritation

C

Which diet choice by the patient with an acute exacerbation of inflammatory bowel disease (IBD) indicates a need for more teaching? a. Scrambled eggs b. White toast and jam c. Oatmeal with cream d. Pancakes with syrup

C

Which laboratory value should the nurse review to determine whether a patient's hypothyroidism is caused by a problem with the anterior pituitary gland or with the thyroid gland? a. Thyroxine (T4) level b. Triiodothyronine (T3) level c. Thyroid-stimulating hormone (TSH) level d. Thyrotropin-releasing hormone (TRH) level

C

Physiological Integrity 24. A patient who is receiving dobutamine (Dobutrex) for the treatment of acute decompensated heart failure (ADHF) has the following nursing interventions included in the plan of care. Which action will be most appropriate for the registered nurse (RN) to delegate to an experienced licensed practical/vocational nurse (LPN/LVN)? a. Assess the IV insertion site for signs of extravasation. b. Teach the patient the reasons for remaining on bed rest. c. Monitor the patient's blood pressure and heart rate every hour. d. Titrate the rate to keep the systolic blood pressure >90 mm Hg.

C An experienced LPN/LVN would be able to monitor BP and heart rate and would know to report significant changes to the RN. Teaching patients, making adjustments to the drip rate for vasoactive medications, and monitoring for serious complications such as extravasation require RN level education and scope of practice. DIF: Cognitive Level: Apply (application) REF: 15-16 OBJ: Special Questions: Delegation TOP: Nursing Process: Planning MSC:

Physiological Integrity 13. Which diagnostic test will be most useful to the nurse in determining whether a patient admitted with acute shortness of breath has heart failure? a. Serum troponin b. Arterial blood gases c. B-type natriuretic peptide d. 12-lead electrocardiogram

C B-type natriuretic peptide (BNP) is secreted when ventricular pressures increase, as they do with heart failure. Elevated BNP indicates a probable or very probable diagnosis of heart failure. A twelve-lead electrocardiogram, arterial blood gases, and troponin may also be used in determining the causes or effects of heart failure but are not as clearly diagnostic of heart failure as BNP. DIF: Cognitive Level: Apply (application) REF: 772 TOP: Nursing Process: Assessment MSC:

Physiological Integrity 15. A patient with heart failure has a new order for captopril (Capoten) 12.5 mg PO. After administering the first dose and teaching the patient about the drug, which statement by the patient indicates that teaching has been effective? a. "I will be sure to take the medication with food." b. "I will need to eat more potassium-rich foods in my diet." c. "I will call for help when I need to get up to use the bathroom." d. "I will expect to feel more short of breath for the next few days."

C Captopril can cause hypotension, especially after the initial dose, so it is important that the patient not get up out of bed without assistance until the nurse has had a chance to evaluate the effect of the first dose. The angiotensin-converting enzyme (ACE) inhibitors are potassium sparing, and the nurse should not teach the patient to purposely increase sources of dietary potassium. Increased shortness of breath is expected with the initiation of -adrenergic blocker therapy for heart failure, not for ACE inhibitor therapy. ACE inhibitors are best absorbed when taken an hour before eating. DIF: Cognitive Level: Apply (application) REF: 776 TOP: Nursing Process: Evaluation MSC:

Physiological Integrity 22. A patient has recently started on digoxin (Lanoxin) in addition to furosemide (Lasix) and captopril (Capoten) for the management of heart failure. Which assessment finding by the home health nurse is a priority to communicate to the health care provider? a. Presence of 1 to 2+ edema in the feet and ankles b. Palpable liver edge 2 cm below the ribs on the right side c. Serum potassium level 3.0 mEq/L after 1 week of therapy d. Weight increase from 120 pounds to 122 pounds over 3 days

C Hypokalemia can predispose the patient to life-threatening dysrhythmias (e.g., premature ventricular contractions), and potentiate the actions of digoxin and increase the risk for digoxin toxicity, which can also cause life-threatening dysrhythmias. The other data indicate that the patient's heart failure requires more effective therapies, but they do not require nursing action as rapidly as the low serum potassium level. DIF: Cognitive Level: Apply (application) REF: 777 OBJ: Special Questions: Prioritization TOP: Nursing Process: Assessment MSC:

Physiological Integrity 9. The nurse plans discharge teaching for a patient with chronic heart failure who has prescriptions for digoxin (Lanoxin) and hydrochlorothiazide (HydroDIURIL). Appropriate instructions for the patient include a. limit dietary sources of potassium. b. take the hydrochlorothiazide before bedtime. c. notify the health care provider if nausea develops. d. skip the digoxin if the pulse is below 60 beats/minute.

C Nausea is an indication of digoxin toxicity and should be reported so that the provider can assess the patient for toxicity and adjust the digoxin dose, if necessary. The patient will need to include potassium-containing foods in the diet to avoid hypokalemia. Patients should be taught to check their pulse daily before taking the digoxin and if the pulse is less than 60, to call their provider before taking the digoxin. Diuretics should be taken early in the day to avoid sleep disruption. DIF: Cognitive Level: Apply (application) REF: 777 TOP: Nursing Process: Planning MSC:

Physiological Integrity 5. A patient who has chronic heart failure tells the nurse, "I was fine when I went to bed, but I woke up in the middle of the night feeling like I was suffocating!" The nurse will document this assessment finding as a. orthopnea. b. pulsus alternans. c. paroxysmal nocturnal dyspnea. d. acute bilateral pleural effusion.

C Paroxysmal nocturnal dyspnea is caused by the reabsorption of fluid from dependent body areas when the patient is sleeping and is characterized by waking up suddenly with the feeling of suffocation. Pulsus alternans is the alternation of strong and weak peripheral pulses during palpation. Orthopnea indicates that the patient is unable to lie flat because of dyspnea. Pleural effusions develop over a longer time period. DIF: Cognitive Level: Remember (knowledge) REF: 770-771 TOP: Nursing Process: Assessment MSC:

Physiological Integrity 4. IV sodium nitroprusside (Nipride) is ordered for a patient with acute pulmonary edema. During the first hours of administration, the nurse will need to titrate the nitroprusside rate if the patient develops a. ventricular ectopy. b. a dry, hacking cough. c. a systolic BP <90 mm Hg. d. a heart rate <50 beats/minute.

C Sodium nitroprusside is a potent vasodilator, and the major adverse effect is severe hypotension. Coughing and bradycardia are not adverse effects of this medication. Nitroprusside does not cause increased ventricular ectopy. DIF: Cognitive Level: Apply (application) REF: 774 TOP: Nursing Process: Evaluation MSC:

Physiological Integrity 7. The nurse working on the heart failure unit knows that teaching an older female patient with newly diagnosed heart failure is effective when the patient states that a. she will take furosemide (Lasix) every day at bedtime. b. the nitroglycerin patch is applied when any chest pain develops. c. she will call the clinic if her weight goes from 124 to 128 pounds in a week. d. an additional pillow can help her sleep if she is feeling short of breath at night.

C Teaching for a patient with heart failure includes information about the need to weigh daily and notify the health care provider about an increase of 3 pounds in 2 days or 3 to 5 pounds in a week. Nitroglycerin patches are used primarily to reduce preload (not to prevent chest pain) in patients with heart failure and should be used daily, not on an "as needed" basis. Diuretics should be taken earlier in the day to avoid nocturia and sleep disturbance. The patient should call the clinic if increased orthopnea develops, rather than just compensating by further elevating the head of the bed. DIF: Cognitive Level: Apply (application) REF: 771 TOP: Nursing Process: Evaluation MSC:

Physiological Integrity 18. A patient with chronic heart failure who is taking a diuretic and an angiotensin-converting enzyme (ACE) inhibitor and who is on a low-sodium diet tells the home health nurse about a 5-pound weight gain in the last 3 days. The nurse's priority action will be to a. have the patient recall the dietary intake for the last 3 days. b. ask the patient about the use of the prescribed medications. c. assess the patient for clinical manifestations of acute heart failure. d. teach the patient about the importance of restricting dietary sodium.

C The 5-pound weight gain over 3 days indicates that the patient's chronic heart failure may be worsening. It is important that the patient be assessed immediately for other clinical manifestations of decompensation, such as lung crackles. A dietary recall to detect hidden sodium in the diet, reinforcement of sodium restrictions, and assessment of medication compliance may be appropriate interventions but are not the first nursing actions indicated. DIF: Cognitive Level: Apply (application) REF: 771 OBJ: Special Questions: Prioritization TOP: Nursing Process: Assessment MSC:

Physiological Integrity 3. Which topic will the nurse plan to include in discharge teaching for a patient with systolic heart failure and an ejection fraction of 33%? a. Need to begin an aerobic exercise program several times weekly b. Use of salt substitutes to replace table salt when cooking and at the table c. Benefits and side effects of angiotensin-converting enzyme (ACE) inhibitors d. Importance of making an annual appointment with the primary care provider

C The core measures for the treatment of heart failure established by The Joint Commission indicate that patients with an ejection fraction (EF) <40% receive an ACE inhibitor to decrease the progression of heart failure. Aerobic exercise may not be appropriate for a patient with this level of heart failure, salt substitutes are not usually recommended because of the risk of hyperkalemia, and the patient will need to see the primary care provider more frequently than annually. DIF: Cognitive Level: Apply (application) REF: 767 | 776 | 779 TOP: Nursing Process: Planning MSC:

Physiological Integrity 10. While admitting an 82-year-old with acute decompensated heart failure to the hospital, the nurse learns that the patient lives alone and sometimes confuses the "water pill" with the "heart pill." When planning for the patient's discharge the nurse will facilitate a a. consult with a psychologist. b. transfer to a long-term care facility. c. referral to a home health care agency. d. arrangements for around-the-clock care.

C The data about the patient suggest that assistance in developing a system for taking medications correctly at home is needed. A home health nurse will assess the patient's home situation and help the patient develop a method for taking the two medications as directed. There is no evidence that the patient requires services such as a psychologist consult, long-term care, or around-the-clock home care. DIF: Cognitive Level: Apply (application) REF: 783 TOP: Nursing Process: Planning MSC:

1. While assessing a 68-year-old with ascites, the nurse also notes jugular venous distention (JVD) with the head of the patient's bed elevated 45 degrees. The nurse knows this finding indicates a. decreased fluid volume. b. jugular vein atherosclerosis. c. increased right atrial pressure. d. incompetent jugular vein valves.

C The jugular veins empty into the superior vena cava and then into the right atrium, so JVD with the patient sitting at a 45-degree angle reflects increased right atrial pressure. JVD is an indicator of excessive fluid volume (increased preload), not decreased fluid volume. JVD is not caused by incompetent jugular vein valves or atherosclerosis. DIF: Cognitive Level: Understand (comprehension) REF: 769 TOP: Nursing Process: Assessment MSC:

Physiological Integrity 23. An outpatient who has chronic heart failure returns to the clinic after 2 weeks of therapy with metoprolol (Toprol XL). Which assessment finding is most important for the nurse to report to the health care provider? a. 2+ pedal edema b. Heart rate of 56 beats/minute c. Blood pressure (BP) of 88/42 mm Hg d. Complaints of fatigue

C The patient's BP indicates that the dose of metoprolol may need to be decreased because of hypotension. Bradycardia is a frequent adverse effect of -adrenergic blockade, but the rate of 56 is not unusual with â-adrenergic blocker therapy. -Adrenergic blockade initially will worsen symptoms of heart failure in many patients, and patients should be taught that some increase in symptoms, such as fatigue and edema, is expected during the initiation of therapy with this class of drugs. DIF: Cognitive Level: Apply (application) REF: 777 OBJ: Special Questions: Prioritization TOP: Nursing Process: Assessment MSC:

Physiological Integrity 17. A patient with a history of chronic heart failure is admitted to the emergency department (ED) with severe dyspnea and a dry, hacking cough. Which action should the nurse do first? a. Auscultate the abdomen. b. Check the capillary refill. c. Auscultate the breath sounds. d. Assess the level of orientation.

C This patient's severe dyspnea and cough indicate that acute decompensated heart failure (ADHF) is occurring. ADHF usually manifests as pulmonary edema, which should be detected and treated immediately to prevent ongoing hypoxemia and cardiac/respiratory arrest. The other assessments will provide useful data about the patient's volume status and also should be accomplished rapidly, but detection (and treatment) of pulmonary complications is the priority. DIF: Cognitive Level: Apply (application) REF: 769-770 OBJ: Special Questions: Prioritization TOP: Nursing Process: Assessment MSC:

The nurse teaches a 28-year-old man newly diagnosed with hypertension about lifestyle modifications to reduce his blood pressure. Which statement by the patient requires an intervention by the nurse? A "I will avoid adding salt to my food during or after cooking." B "If I lose weight, I might not need to continue taking medications." C "I can lower my blood pressure by switching to smokeless tobacco." D "Diet changes can be as effective as taking blood pressure medications."

C "I can lower my blood pressure by switching to smokeless tobacco." Nicotine contained in tobacco products (smoking and chew) cause vasoconstriction and increase blood pressure. Persons with hypertension should restrict sodium to 1500 mg/day by avoiding foods high in sodium and not adding salt in preparation of food or at meals. Weight loss can decrease blood pressure between 5 to 20 mm Hg. Following dietary recommendations (such as the DASH diet) lowers blood pressure, and these decreases compare with those achieved with blood pressure-lowering medication.

A 19-year-old female is brought to the emergency department with a knife handle protruding from the abdomen. During the initial assessment of the patient, the nurse should a. remove the knife and assess the wound. b. determine the presence of Rovsing sign. c. check for circulation and tissue perfusion. d. insert a urinary catheter and assess for hematuria.

C AB C'SSSSS check the circulation before doing anything else

The nurse is caring for a patient admitted with chronic obstructive pulmonary disease (COPD), angina, and hypertension. Before administering the prescribed daily dose of atenolol 100 mg PO, the nurse assesses the patient carefully. Which adverse effect is this patient at risk for, given the patient's health history? A Hypocapnia B Tachycardia C Bronchospasm D Nausea and vomiting

C Bronchospasm Atenolol is a cardioselective β1-adrenergic blocker that reduces blood pressure and could affect the β2-receptors in the lungs with larger doses or with drug accumulation. Although the risk of bronchospasm is less with cardioselective β-blockers than nonselective β-blockers, atenolol should be used cautiously in patients with COPD.

The patient has chronic hypertension. Today she has gone to the ED, and her blood pressure has risen to 200/140. What is the priority assessment for the nurse to make? A Is the patient pregnant? B Does the patient need to urinate? C Does the patient have a headache or confusion? D Is the patient taking antiseizure medications as prescribed?

C Does the patient have a headache or confusion? The nurse's priority assessments include neurologic deficits, retinal damage, heart failure, pulmonary edema, and renal failure. The headache or confusion could be seen with hypertensive encephalopathy from increased cerebral capillary permeability leading to cerebral edema. Pregnancy can lead to secondary hypertension. Needing to urinate and taking antiseizure medication do not support a hypertensive emergency.

When assessing the body functions of a patient with increased ICP, what should the nurse assess first?

Circulatory and respiratory status

After being hit by a baseball, a patient arrives in the emergency department with a possible nasal fracture. Which finding by the nurse is most important to report to the health care provider?

Clear nasal drainage

Which nursing diagnosis is likely to be a priority in the care of a patient with myasthenia gravis (MG)? A. Acute confusion B. Bowel incontinence C. Activity intolerance D. Disturbed sleep pattern

C. The primary feature of MG is fluctuating weakness of skeletal muscle. Bowel incontinence and confusion are unlikely signs of MG, and although sleep disturbance is likely, activity intolerance is usually of primary concern.

When teaching how lisinopril (Zestril) will help lower the patient's blood pressure, which mechanism of action should the nurse use to explain it? A Blocks β-adrenergic effects. B Relaxes arterial and venous smooth muscle. C Inhibits conversion of angiotensin I to angiotensin II. D Reduces sympathetic outflow from central nervous system.

C Inhibits conversion of angiotensin I to angiotensin II. Lisinopril is an angiotensin-converting enzyme (ACE) inhibitor that inhibits the conversion of angiotensin I to angiotensin II, which reduces angiotensin II-mediated vasoconstriction and sodium and water retention. Beta blockers result in vasodilation and decreased heart rate. Direct vasodilators relax arterial and venous smooth muscle. Central acting α-adrenergic antagonists reduce sympathetic outflow from the CNS to produce vasodilation and decreased SVR and BP.

The nurse is teaching a women's group about prevention of hypertension. What information should be included in the teaching for all the women (select all that apply)? A Lose weight. B Limit nuts and seeds. C Limit sodium and fat intake. D Increase fruits and vegetables. E Exercise 30 minutes most days.

C Limit sodium and fat intake. D Increase fruits and vegetables. E Exercise 30 minutes most days. Primary prevention of hypertension is to make lifestyle modifications that prevent or delay the increase in BP. Along with exercise for 30 minutes on most days, the DASH eating plan is a healthy way to lower BP by limiting sodium and fat intake, increasing fruits and vegetables, and increasing nutrients that are associated with lowering BP. Nuts and seeds and dried beans are used for protein intake. Weight loss may or may not be necessary for the individual.

A 67-year-old woman with a history of coronary artery disease and prior myocardial infarction is admitted to the emergency department with a blood pressure of 234/148 mm Hg and started on IV nitroprusside (Nitropress). What should the nurse determine as an appropriate goal for the first hour of treatment? A Mean arterial pressure lower than 70 mm Hg B Mean arterial pressure no more than 120 mm Hg C Mean arterial pressure no lower than 133 mm Hg D Mean arterial pressure between 70 and 110 mm Hg

C Mean arterial pressure no lower than 133 mm Hg The initial treatment goal is to decrease mean arterial pressure by no more than 25% within minutes to 1 hour. If the patient is stable, the goal for BP is 160/100 to 110 mm Hg over the next 2 to 6 hours. Lowering the blood pressure too much may decrease cerebral, coronary, or renal perfusion and could precipitate a stroke, myocardial infarction, or renal failure. Additional gradual reductions toward a normal blood pressure should be implemented over the next 24 to 48 hours if the patient is clinically stable.

In caring for a patient admitted with poorly controlled hypertension, which laboratory test result should the nurse understand as indicating the presence of target organ damage? A BUN of 15 mg/dL B Serum uric acid of 3.8 mg/dL C Serum creatinine of 2.6 mg/dL D Serum potassium of 3.5 mEq/L

C Serum creatinine of 2.6 mg/dL The normal serum creatinine level is 0.6-1.3 mg/dL. This elevated level indicates target organ damage to the kidneys. The other lab results are within normal limits.

A pulmonary embolus (PE) is suspected in a patient with a DVT who develops dyspnea, tachycardia, and chest pain. Diagnostic testing is scheduled. Which test should the nurse plan to teach the patient about?

C Spiral (helical) CT scan

Four hours after a bowel resection, a 74-year-old male patient with a nasogastric tube to suction complains of nausea and abdominal distention. The first action by the nurse should be to a. auscultate for hypotonic bowel sounds. b. notify the patient's health care provider. c. reposition the tube and check for placement. d. remove the tube and replace it with a new one.

C the tube may have been misplaced and accumulation of fluid is causing the nausea and distension

A female patient complains of a throbbing headache. When her history is obtained, the nurse discovers that the patient has had this type of headache before and experienced photophobia before the headache occurred. The nurse should know that what is probably the cause of this patient's headache? A. Polycythemia vera B. A cluster headache C. A migraine headache D. A hemorrhagic stroke

C. Although a headache may occur with any of these options, a migraine headache is the only one that has a throbbing headache with an aura (the photophobia). Headache from polycythemia vera is from erythrocytosis. The cluster headache pain is sharp and stabbing, and the headache with a hemorrhagic stroke has a sudden onset and is not recurrent.

A male patient with a diagnosis of Parkinson's disease (PD) has been admitted recently to a long-term care facility. Which action should the health care team take in order to promote adequate nutrition for this patient? A. Provide multivitamins with each meal. B. Provide a diet that is low in complex carbohydrates and high in protein. C. Provide small, frequent meals throughout the day that are easy to chew and swallow. D. Provide the patient with a minced or pureed diet that is high in potassium and low in sodium.

C. Nutritional support is a priority in the care of individuals with PD. Such patients may benefit from meals that are smaller and more frequent than normal and that are easy to chew and swallow. Multivitamins are not necessary at each meal, and vitamin intake, along with protein intake, must be monitored to prevent contraindications with medications. It is likely premature to introduce a minced or pureed diet, and a low carbohydrate diet is not indicated.

The nurse provides dietary instructions to the in-home caregiver of a 45-year-old man who has Huntington's disease. The nurse is most concerned if the caregiver makes which statement? A. "Depression is common and may cause a decrease in appetite." B. "If swallowing becomes difficult, a feeding tube may be needed." C. "Calories should be restricted to prevent unnecessary weight gain." D. "Muscles in the face are affected, and chewing may become impossible."

C. Patients with Huntington's disease may require 4000 to 5000 calories per day to maintain body weight. Weight loss occurs in patients with Huntington's disease because of choreic movements, difficulty swallowing, depression, and mental deterioration.

The nurse is caring for a group of patients on a medical unit. After receiving report, which patient should the nurse see first? A. A 42-year-old patient with multiple sclerosis who was admitted with sepsis B. A 72-year-old patient with Parkinson's disease who has aspiration pneumonia C. A 38-year-old patient with myasthenia gravis who declined prescribed medications D. A 45-year-old patient with amyotrophic lateral sclerosis who refuses enteral feedings

C. Patients with myasthenia gravis who discontinue pyridostigmine (Mestinon) will develop a myasthenic crisis. Myasthenia crisis results in severe muscle weakness and can lead to a respiratory arrest.

Which assessment is most important for the nurse to make regarding a patient with myasthenia gravis? a. Pupil size b. Grip strength c. Respiratory effort d. Level of consciousness

C. Because respiratory insufficiency may be life threatening, it will be most important to monitor respiratory function. The other data also will be assessed but are not as critical.

While the nurse is transporting a patient on a stretcher to the radiology department, the patient begins having a tonic-clonic seizure. Which action should the nurse take? a. Insert an oral airway during the seizure to maintain a patent airway. b. Restrain the patient's arms and legs to prevent injury during the seizure. c. Time and observe and record the details of the seizure and postictal state. d. Avoid touching the patient to prevent further nervous system stimulation.

C. Because the diagnosis and treatment of seizures frequently are based on the description of the seizure, recording the length and details of the seizure is important. Insertion of an oral airway and restraining the patient during the seizure are contraindicated. The nurse may need to move the patient to decrease the risk of injury during the seizure.

A 49-year-old patient with multiple sclerosis (MS) is to begin treatment with glatiramer acetate (Copaxone). Which information will the nurse include in patient teaching? a. Recommendation to drink at least 4 L of fluid daily b. Need to avoid driving or operating heavy machinery c. How to draw up and administer injections of the medication d. Use of contraceptive methods other than oral contraceptives

C. Copaxone is administered by self-injection. Oral contraceptives are an appropriate choice for birth control. There is no need to avoid driving or drink large fluid volumes when taking glatiramer.

A 40-year-old patient is diagnosed with early Huntington's disease (HD). When teaching the patient, spouse, and children about this disorder, the nurse will provide information about the a. use of levodopa-carbidopa (Sinemet) to help reduce HD symptoms. b. prophylactic antibiotics to decrease the risk for aspiration pneumonia. c. option of genetic testing for the patient's children to determine their own HD risks. d. lifestyle changes of improved nutrition and exercise that delay disease progression.

C. Genetic testing is available to determine whether an asymptomatic individual has the HD gene. The patient and family should be informed of the benefits and problems associated with genetic testing. Sinemet will increase symptoms of HD because HD involves an increase in dopamine. Antibiotic therapy will not reduce the risk for aspiration. There are no effective treatments or lifestyle changes that delay the progression of symptoms in HD.

The nurse determines that teaching about management of migraine headaches has been effective when the patient says which of the following? a. "I can take the (Topamax) as soon as a headache starts." b. "A glass of wine might help me relax and prevent a headache." c. "I will lie down someplace dark and quiet when the headaches begin." d. "I should avoid taking aspirin and sumatriptan (Imitrex) at the same time."

C. It is recommended that the patient with a migraine rest in a dark, quiet area. Topiramate (Topamax) is used to prevent migraines and must be taken for several months to determine effectiveness. Aspirin or other nonsteroidal antiinflammatory medications can be taken with the triptans. Alcohol may precipitate migraine headaches.

A 22-year-old patient seen at the health clinic with a severe migraine headache tells the nurse about having other similar headaches recently. Which initial action should the nurse take? a. Teach about the use of triptan drugs. b. Refer the patient for stress counseling. c. Ask the patient to keep a headache diary. d. Suggest the use of muscle-relaxation techniques.

C. The initial nursing action should be further assessment of the precipitating causes of the headaches, quality, and location of pain, etc. Stress reduction, muscle relaxation, and the triptan drugs may be helpful, but more assessment is needed first.

When reading a patient's chart, the nurse notes that the patient has dysuria. To assess whether there is any improvement, which question will the nurse ask? A. "Do you have any blood in your urine?" B. "Do you have to urinate very frequently?" C. "Do you have any pain when you urinate?" D. "Do you have to get up at night to urinate?" Dysuria is painful urination. The alternate responses are used to assess other urinary tract symptoms: hematuria, nocturia, and frequency.

C. "Do you have any pain when you urinate?" Dysuria is painful urination. The alternate responses are used to assess other urinary tract symptoms: hematuria, nocturia, and frequency.

A 67-year-old male patient had a right total knee replacement 2 days ago. Upon auscultation of the patient's posterior chest, the nurse detects discontinuous, high-pitched breath sounds just before the end of inspiration in the lower portion of both lungs. Which statement most appropriately reflects how the nurse should document the breath sounds? A. "Bibasilar rhonchi present on inspiration." B. "Diminished breath sounds in the bases of both lungs." C. "Fine crackles posterior right and left lower lung fields." D. "Expiratory wheezing scattered throughout the lung fields."

C. "Fine crackles posterior right and left lower lung fields." Fine crackles are described as a series of short-duration, discontinuous, high-pitched sounds heard just before the end of inspiration.

A patient who had a cystoscopy the previous day calls the urology clinic and gives the nurse all the following information. Which statement by the patient should be reported immediately to the health care provider? A. "My urine still looks pink." B. "My IV site is still bruised." C. "I have a temperature of 101." D. "I did not sleep well last night."

C. "I have a temperature of 101." The patient's elevated temperature may indicate a bladder infection, a possible complication of cystoscopy. The health care provider should be notified so that antibiotic therapy can be started. Pink-tinged urine is expected after a cystoscopy. The insomnia and bruising should be discussed further with the patient but do not indicate a need to notify the health care provider.

The nurse is interpreting a tuberculin skin test (TST) for a 58-year-old female patient with end-stage kidney disease secondary to diabetes mellitus. Which finding would indicate a positive reaction? A. Acid-fast bacilli cultured at the injection site B. 15-mm area of redness at the TST injection site C. 11-mm area of induration at the TST injection site D. Wheal formed immediately after intradermal injection

C. 11-mm area of induration at the TST injection site An area of induration ≥ 10 mm would be a positive reaction in a person with end-stage kidney disease. Reddened, flat areas do not indicate a positive reaction. A wheal appears when the TST is administered that indicates correct administration of the intradermal antigen. Presence of acid-fast bacilli in the sputum indicates active tuberculosis.

The patient is hospitalized with pneumonia. Which diagnostic test should be used to measure the efficiency of gas transfer in the lung and tissue oxygenation? A. Thoracentesis B. Bronchoscopy C. Arterial blood gases D. Pulmonary function tests

C. Arterial blood gases Arterial blood gases are used to assess the efficiency of gas transfer in the lung and tissue oxygenation as is pulse oximetry. Thoracentesis is used to obtain specimens for diagnostic evaluation, remove pleural fluid, or instill medication into the pleural space. Bronchoscopy is used for diagnostic purposes, to obtain biopsy specimens, and to assess changes resulting from treatment. Pulmonary function tests measure lung volumes and airflow to diagnose pulmonary disease, monitor disease progression, evaluate disability, and evaluate response to bronchodilators

A patient's sudden onset of hemiplegia has necessitated a computed tomography (CT) of her head. Which assessment should the nurse complete prior to this diagnostic study? A. Assess the patient's immunization history. B. Screen the patient for any metal parts or pacemaker. C. Assess the patient for allergies to shellfish, iodine, or dyes. D. Assess the patient's need for tranquilizers or antiseizure medications.

C. Assess the patient for allergies to shellfish, iodine, or dyes. Allergies to shellfish, iodine, or dyes contraindicate the use of contrast media in CT. The patient's immunization history is not a central consideration, and the presence of metal in the body does not preclude the use of CT as a diagnostic tool. The need to assess for allergies supersedes the need for tranquilizers or antiseizure medications in the majority of patients.

The nurse, when auscultating the lower lungs of the patient, hears these breath sounds. How should the nurse document these sounds? A. Stridor B. Rhonchi C. Coarse crackles D. Bronchovesicular

C. Coarse crackles Coarse crackles are a series of long-duration, discontinuous, low-pitched sounds caused by air passing through an airway intermittently occluded by mucus, an unstable bronchial wall, or a fold of mucosa. Coarse crackles are evident on inspiration and at times expiration. Stridor is a continuous crowing sound of constant pitch from partial obstruction of larynx or trachea. Rhonchi are a continuous rumbling, snoring, or rattling sound from obstruction of large airways with secretions. Bronchovesicular sounds are normal sounds heard anteriorly over the mainstem bronchi on either side of the sternum and posteriorly between the scapulae with a medium pitch and intensity.

The new patient has a diagnosis of frontal lobe dementia. What functional difficulties should the nurse expect in this patient? A. A lack of reflexes. B. Endocrine problems. C. Higher cognitive function abnormalities. D. Respiratory, vasomotor, and cardiac dysfunction.

C. Higher cognitive function abnormalities. Because the frontal lobe is responsible for higher cognitive function, this patient may have difficulty with memory retention, voluntary eye movements, voluntary motor movement, and expressive speech. The lack of reflexes would occur if the patient had problems with the reflex arcs in the spinal cord. Endocrine problems would be evident if the hypothalamus or pituitary gland were affected. Respiratory, vasomotor, and cardiac dysfunction would occur if there were a problem in the medulla.

The nurse is caring for a group of well older people at a community day center. Which neurologic finding associated with aging would the nurse expect to find in older adults? A. Longer reaction time. B. Improved sense of taste C. Orthostatic hypotension D. Hyperactive deep tendon reflexes

C. Orthostatic hypotension Older adults are more likely to experience orthostatic hypotension related to altered coordination of neuromuscular activity. Other neurologic changes in the older adult include atrophy of taste buds with decreased sense of taste, below average reflex score (and diminished deep tendon reflexes), and slowed reaction times.

How will the nurse assess the flank area of a patient with pyelonephritis for tenderness? A. Push gently into the two lowest intercostal spaces. B. Palpate along both sides of the lumbar vertebral column. C. Position one hand flat at the costovertebral angle (CVA) & strike it with the other fist. D. Use two fingers to percuss the area between the iliac crest and ribs along the midaxillary line.

C. Position one hand flat at the costovertebral angle (CVA) and strike it with the other fist. Checking for flank pain is best performed by percussion of the CVA and asking about pain.

The nurse is preparing the patient for an electromyography (EMG). What should the nurse include in teaching the patient before the test? A. The patient will be tilted on a table during the test. B. It is noninvasive, and there is no risk of electric shock. C. The pain that occurs is from the insertion of the needles. D. The passive sensor does not make contact with the patient.

C. The pain that occurs is from the insertion of the needles. With an EMG, pain may occur when needles are inserted to record the electrical activity of nerve and skeletal muscle. The patient is tilted on a table during a myelogram. The electroencephalography (EEG) is noninvasive without a danger of electric shock. The magnetoencephalography (MEG) is done with a passive sensor that does not make contact with the patient.

When reviewing the results of a patient's urinalysis, which information indicates that the nurse should notify the health care provider? A. pH 6.2 B. Trace protein C. WBC: 20-26/hpf D. Specific gravity: 1.021

C. WBC: 20-26/hpf The increased number of white blood cells (WBCs) indicates the presence of urinary tract infection or inflammation. The other findings are normal.

The nurse can best determine adequate arterial oxygenation of the blood by assessing: A. heart rate. B. hemoglobin level. C. arterial oxygen tension. D. arterial carbon dioxide tension.

C. arterial oxygen tension. The ability of the lungs to oxygenate arterial blood adequately is determined by examination of the partial pressure of oxygen in arterial blood (PaO2) and arterial oxygen saturation (SaO2).

The nurse uses auscultation during assessment of the urinary system to A. check for ureteral peristalsis. B. assess for bladder distention. Incorrect C. identify renal artery or aortic bruits. D. determine the position of the kidneys.

C. identify renal artery or aortic bruits. The presence of a bruit may indicate problems such as renal artery tortuosity or abdominal aortic aneurysm. Auscultation would not be helpful in assessing for the other listed urinary tract information.

During the respiratory assessment of the older adult, the nurse would expect to find (select all that apply): A. a vigorous cough. B. increased chest expansion. C. increased residual volume. D. increased breath sounds in the lung apices. E. increased anteroposterior (AP) chest diameter.

C. increased residual volume. E. increased anteroposterior (AP) chest diameter. The anterior-posterior diameter of the thoracic cage and the residual volume increase in older adults. An older adult has a less forceful cough. The costal cartilages calcify with aging and interfere with chest expansion. Small airways in the lung bases close earlier during expiration. As a consequence, more inspired air is distributed to the lung apices, ventilation is less well matched to perfusion, and the PaO2 is lowered.

An obstruction of the anterior cerebral arteries will affect functions of A. visual imaging. B. balance and coordination. C. judgement, insight, and reasoning. D. visual and auditory integration for language comprehension.

C. judgement, insight, and reasoning. The anterior cerebral artery feeds the medial and anterior portions of the frontal lobes. The anterior portion of the frontal lobe controls higher order processes such as judgment and reasoning.

During neurologic testing, the patient is able to perceive pain elicited by pinprick. Based on this finding, the nurse may omit testing for: A. position sense. B patellar reflexes. C. temperature perception. D. heel-to-shin movements.

C. temperature perception. If pain sensation is intact, assessment of temperature sensation may be omitted because both sensations are transmitted by the same ascending pathways.

The nurse responds to a ventilator alarm and finds the patient lying in bed holding the endotracheal tube (ET). Which action should the nurse take next?

Call a Rapid Response

A patients cardiac monitor shows sinus rhythm, rate 64. The P-R interval is 0.18 seconds at 1:00AM, 0.22 seconds at 2:30 PM, and 0.28 seconds at 4:00 PM. Which action should the nurse take next?

Call the health care provider before giving the next dose of metoprolol (Lopressor).

After a ureterolithotomy, a female patient has a left ureteral catheter and a urethral catheter in place. Which action will the nurse include in the plan of care?

Call the health care provider if the ureteral catheter output drops suddenly.

A patient with newly diagnosed lung cancer tells the nurse, I dont think Im going to live to see my next birthday. Which response by the nurse is best?

Can you tell me what it is that makes you think you will die so soon?

A nurse is assessing a client who has DM and reports feeling anxious. Which of the following findings should the nurse expect if the client is hypoglycemic? A. Rapid, deep respirations B. Cool, clammy skin C. Abdominal cramping D. Orthostatic Hypotension

Cool, clammy skin (in addition to anxiety, nervousness, tachycardia, and confusion)

Which finding by the nurse will be most helpful in determining whether a 67-year-old patient with benign prostatic hyperplasia (BPH) has an upper urinary tract infection (UTI)?

Costovertebral tenderness

A 27-year-old female patient is admitted to the hospital for evaluation of right lower quadrant abdominal pain with nausea and vomiting. Which action should the nurse take? a. Encourage the patient to sip clear liquids. b. Assess the abdomen for rebound tenderness. c. Assist the patient to cough and deep breathe. d. Apply an ice pack to the right lower quadrant.

D

A 42-year-old male patient has had a herniorrhaphy to repair an incarcerated inguinal hernia. Which patient teaching will the nurse provide before discharge? a. Soak in sitz baths several times each day. b. Cough 5 times each hour for the next 48 hours. c. Avoid use of acetaminophen (Tylenol) for pain. d. Apply a scrotal support and ice to reduce swelling.

D

A 47-year-old female patient is transferred from the recovery room to a surgical unit after a transverse colostomy. The nurse observes the stoma to be deep pink with edema and a small amount of sanguineous drainage. The nurse should a. place ice packs around the stoma. b. notify the surgeon about the stoma. c. monitor the stoma every 30 minutes. d. document stoma assessment findings.

D

A 61-year-old female patient admitted with pneumonia has a total serum calcium level of 13.3 mg/dL (3.3 mmol/L). The nurse will anticipate the need to teach the patient about testing for _____ levels. a. calcitonin b. catecholamine c. thyroid hormone d. parathyroid hormone

D

A 64-year-old woman who has chronic constipation asks the nurse about the use of psyllium (Metamucil). Which information will the nurse include in the response? a. Absorption of fat-soluble vitamins may be reduced by fiber-containing laxatives. b. Dietary sources of fiber should be eliminated to prevent excessive gas formation. c. Use of this type of laxative to prevent constipation does not cause adverse effects. d. Large amounts of fluid should be taken to prevent impaction or bowel obstruction.

D

A 73-year-old patient with diverticulosis has a large bowel obstruction. The nurse will monitor for a. referred back pain. b. metabolic alkalosis. c. projectile vomiting. d. abdominal distention.

D

An 18-year-old male patient with a small stature is scheduled for a growth hormone stimulation test. In preparation for the test, the nurse will obtain a. ice in a basin. b. glargine insulin. c. a cardiac monitor. d. 50% dextrose solution.

D

The nurse will plan to teach a patient with Crohn's disease who has megaloblastic anemia about the need for a. oral ferrous sulfate tablets. b. regular blood transfusions. c. iron dextran (Imferon) infusions. d. cobalamin (B12) spray or injections.

D

Which breakfast choice indicates a patient's good understanding of information about a diet for celiac disease? a. Oatmeal with nonfat milk b. Whole wheat toast with butter c. Bagel with low-fat cream cheese d. Corn tortilla with scrambled eggs

D

Which patient should the nurse assess first after receiving change-of-shift report? a. 60-year-old patient whose new ileostomy has drained 800 mL over the previous 8 hours b. 50-year-old patient with familial adenomatous polyposis who has occult blood in the stool c. 40-year-old patient with ulcerative colitis who has had six liquid stools in the previous 4 hours d. 30-year-old patient who has abdominal distention and an apical heart rate of 136 beats/minute

D

Which question from the nurse would help determine if a patient's abdominal pain might indicate irritable bowel syndrome? a. "Have you been passing a lot of gas?" b. "What foods affect your bowel patterns?" c. "Do you have any abdominal distention?" d. "How long have you had abdominal pain?"

D

Which statement by a 50-year-old female patient indicates to the nurse that further assessment of thyroid function may be necessary? a. "I notice my breasts are tender lately." b. "I am so thirsty that I drink all day long." c. "I get up several times at night to urinate." d. "I feel a lump in my throat when I swallow."

D

Physiological Integrity 11. Following an acute myocardial infarction, a previously healthy 63-year-old develops clinical manifestations of heart failure. The nurse anticipates discharge teaching will include information about a. digitalis preparations. b. -adrenergic blockers. c. calcium channel blockers. d. angiotensin-converting enzyme (ACE) inhibitors.

D ACE inhibitor therapy is currently recommended to prevent the development of heart failure in patients who have had a myocardial infarction and as a first-line therapy for patients with chronic heart failure. Digoxin therapy for heart failure is no longer considered a first-line measure, and digoxin is added to the treatment protocol when therapy with other medications such as ACE-inhibitors, diuretics, and -adrenergic blockers is insufficient. Calcium channel blockers are not generally used in the treatment of heart failure. The -adrenergic blockers are not used as initial therapy for new onset heart failure. DIF: Cognitive Level: Apply (application) REF: 776 TOP: Nursing Process: Planning MSC:

Physiological Integrity 16. A patient who has just been admitted with pulmonary edema is scheduled to receive the following medications. Which medication should the nurse question before giving? a. Furosemide (Lasix) 60 mg b. Captopril (Capoten) 25 mg c. Digoxin (Lanoxin) 0.125 mg d. Carvedilol (Coreg) 3.125 mg

D Although carvedilol is appropriate for the treatment of chronic heart failure, it is not used for patients with acute decompensated heart failure (ADHF) because of the risk of worsening the heart failure. The other medications are appropriate for the patient with ADHF. DIF: Cognitive Level: Analyze (analysis) REF: 774 | 771 TOP: Nursing Process: Implementation MSC:

Physiological Integrity 2. The nurse is caring for a patient who is receiving IV furosemide (Lasix) and morphine for the treatment of acute decompensated heart failure (ADHF) with severe orthopnea. Which clinical finding is the best indicator that the treatment has been effective? a. Weight loss of 2 pounds in 24 hours b. Hourly urine output greater than 60 mL c. Reduction in patient complaints of chest pain d. Reduced dyspnea with the head of bed at 30 degrees

D Because the patient's major clinical manifestation of ADHF is orthopnea (caused by the presence of fluid in the alveoli), the best indicator that the medications are effective is a decrease in dyspnea with the head of the bed at 30 degrees. The other assessment data also may indicate that diuresis or improvement in cardiac output has occurred, but are not as specific to evaluating this patient's response. DIF: Cognitive Level: Apply (application) REF: 769-770 | 773-774 TOP: Nursing Process: Evaluation MSC:

Physiological Integrity 8. When teaching the patient with newly diagnosed heart failure about a 2000-mg sodium diet, the nurse explains that foods to be restricted include a. canned and frozen fruits. b. fresh or frozen vegetables. c. eggs and other high-protein foods. d. milk, yogurt, and other milk products.

D Milk and yogurt naturally contain a significant amount of sodium, and intake of these should be limited for patients on a diet that limits sodium to 2000 mg daily. Other milk products, such as processed cheeses, have very high levels of sodium and are not appropriate for a 2000-mg sodium diet. The other foods listed have minimal levels of sodium and can be eaten without restriction. DIF: Cognitive Level: Apply (application) REF: 778 TOP: Nursing Process: Implementation MSC:

Which action will the nurse anticipate taking for an otherwise healthy 50-year-old who has just been diagnosed with Stage 1 renal cell carcinoma?

Provide preoperative teaching about nephrectomy.

A 44-year-old man is diagnosed with hypertension and receives a prescription for benazepril (Lotensin). After the nurse teaches him about the medication, which statement by the patient indicates his correct understanding? A "If I take this medication, I will not need to follow a special diet." B "It is normal to have some swelling in my face while taking this medication." C "I will need to eat foods such as bananas and potatoes that are high in potassium." D "If I develop a dry cough while taking this medication, I should notify my doctor."

D "If I develop a dry cough while taking this medication, I should notify my doctor." Benazepril is an angiotensin-converting enzyme inhibitor. The medication inhibits breakdown of bradykinin, which may cause a dry, hacking cough. Other adverse effects include hyperkalemia. Swelling in the face could indicate angioedema and should be reported immediately to the prescriber. Patients taking drug therapy for hypertension should also attempt lifestyle modifications to lower blood pressure such as a reduced-sodium diet.

In reviewing medication instructions with a patient being discharged on antihypertensive medications, which statement would be most appropriate for the nurse to make when discussing guanethidine (Ismelin)? A "A fast heart rate is a side effect to watch for while taking guanethidine." B "Stop the drug and notify your doctor if you experience any nausea or vomiting." C "Because this drug may affect the lungs in large doses, it may also help your breathing." D "Make position changes slowly, especially when rising from lying down to a standing position."

D "Make position changes slowly, especially when rising from lying down to a standing position." Guanethidine is a peripheral-acting α-adrenergic antagonist and can cause marked orthostatic hypotension. For this reason, the patient should be instructed to rise slowly, especially when moving from a recumbent to a standing position. Support stockings may also be helpful. Tachycardia or lung effects are not evident with guanethidine.

A patient who was admitted with myxedema coma and diagnosed with hypothyroidism is improving and expected to be discharged in 2 days. Which teaching strategy will be best for the nurse to use?

Provide written reminders of self-care information.

When evaluating a patient with a central venous catheter, the nurse observes that the insertion site is red and tender to touch and the patients temperature is 101.8 F. What should the nurse plan to do next?

Discontinue the catheter and culture the tip.

The nurse is providing teaching to a patient recovering from an MI. How should resumption of sexual activity be discussed?

Discussed along with other physical activities.

A 28-year-old male patient is diagnosed with polycystic kidney disease. Which information is most appropriate for the nurse to include in teaching at this time?

Discussion of options for genetic counseling

What sign will you see if your patient has superior vena cava syndrome?

Distended jugular veins

When assessing an older patient admitted to the ED with a broken arm and facial bruises, the nurse observes several additional bruises in various stages of healing. Which statement or question by the nurse is most appropriate?

Do you feel safe at home?

The nurse is caring for a patient who has a right-sided chest tube after a right lower lobectomy. Which nursing action can the nurse delegate to the unlicensed assistive personnel (UAP)?

Document the amount of drainage every eight hours.

Your patient decerebrate posture, would have reaction with?

Rigidly extends arms

A patient with cardiogenic shock has the following vital signs: BP 102/50, pulse 128, respirations 28. The pulmonary artery wedge pressure (PAWP) is increased and cardiac output is low. The nurse will anticipate an order for which medication?

Furosemide (Lasix) IV

A patient is having a transsphenoidal hypophysectomy. The nurse should provide preoperative patient teaching about what potential deficit as a result of the surgery? A. Increased heart rate B. Loss of coordination C. Impaired swallowing D. Altered sense of smell

D. Altered sense of smell Using a transsphenoidal approach to remove the pituitary gland includes a risk of damage to the olfactory cranial nerve because the cell bodies of the olfactory nerve are located in the nasal epithelium. With damage to this nerve, the sense of smell would be altered. Increased heart rate, loss of coordination, and impaired swallowing will not be potential deficits from this surgery.

A 45-year-old patient is admitted to the emergency department with severe abdominal pain and rebound tenderness. Vital signs include temperature 102° F (38.3° C), pulse 120, respirations 32, and blood pressure (BP) 82/54. Which prescribed intervention should the nurse implement first? a. Administer IV ketorolac (Toradol) 15 mg. b. Draw blood for a complete blood count (CBC). c. Obtain a computed tomography (CT) scan of the abdomen. d. Infuse 1 liter of lactated Ringer's solution over 30 minutes.

D Give IV solution to increase the low blood pressure due to excesses fluid loss

The nurse is alerted to a possible acute subdural hematoma in the patient who?

Develops decreased LOC and a headache within 48 hours of a head injury

The nurse obtains the following assessment data on an older patient who has influenza. Which information will be most important for the nurse to communicate to the health care provider?

Diffuse crackles in the lungs

After several days of antibiotic therapy, an older hospitalized patient develops watery diarrhea. Which action should the nurse take first? a. Notify the health care provider. b. Obtain a stool specimen for analysis. c. Teach the patient about handwashing. d. Place the patient on contact precautions.

D this indicates that they may have developed C-Diff and need to be put on contact precautions

Which medication taken by a patient with restless legs syndrome should the nurse discuss with the patient? a. Multivitamin (Stresstabs) b. Acetaminophen (Tylenol) c. Ibuprofen (Motrin, Advil) d. Diphenhydramine (Benadryl)

D. Antihistamines can aggravate restless legs syndrome. The other medications will not contribute to restless legs syndrome.

Which information about a 60-year-old patient with MS indicates that the nurse should consult with the health care provider before giving the prescribed dose of dalfampridine (Ampyra)? a. The patient has relapsing-remitting MS. b. The patient walks a mile a day for exercise. c. The patient complains of pain with neck flexion. d. The patient has an increased serum creatinine level.

D. Dalfampridine should not be given to patients with impaired renal function. The other information will not impact whether the dalfampridine should be administered.

A 76-year-old patient is being treated with carbidopa/levodopa (Sinemet) for Parkinson's disease. Which information is most important for the nurse to report to the health care provider? a. Shuffling gait b. Tremor at rest c. Cogwheel rigidity of limbs d. Uncontrolled head movement

D. Dyskinesia is an adverse effect of the Sinemet, indicating a need for a change in medication or decrease in dose. The other findings are typical with Parkinson's disease.

A 62-year-old patient who has Parkinson's disease is taking bromocriptine (Parlodel). Which information obtained by the nurse may indicate a need for a decrease in the dose? a. The patient has a chronic dry cough. b. The patient has four loose stools in a day. c. The patient develops a deep vein thrombosis. d. The patient's blood pressure is 92/52 mm Hg.

D. Hypotension is an adverse effect of bromocriptine, and the nurse should check with the health care provider before giving the medication. Diarrhea, cough, and deep vein thrombosis are not associated with bromocriptine use.

Which information about a 72-year-old patient who has a new prescription for phenytoin (Dilantin) indicates that the nurse should consult with the health care provider before administration of the medication? a. Patient has generalized tonic-clonic seizures. b. Patient experiences an aura before seizures. c. Patient's most recent blood pressure is 156/92 mm Hg. d. Patient has minor elevations in the liver function tests.

D. Many older patients (especially with compromised liver function) may not be able to metabolize phenytoin. The health care provider may need to choose another antiseizure medication. Phenytoin is an appropriate medication for patients with tonic-clonic seizures, with or without an aura. Hypertension is not a contraindication for phenytoin therapy.

Which nursing diagnosis is of highest priority for a patient with Parkinson's disease who is unable to move the facial muscles? a. Activity intolerance b. Self-care deficit: toileting c. Ineffective self-health management d. Imbalanced nutrition: less than body requirements

D. The data about the patient indicate that poor nutrition will be a concern because of decreased swallowing. The other diagnoses may also be appropriate for a patient with Parkinson's disease, but the data do not indicate that they are current problems for this patient.

Which prescribed intervention will the nurse implement first for a patient in the emergency department who is experiencing continuous tonic-clonic seizures? a. Give phenytoin (Dilantin) 100 mg IV. b. Monitor level of consciousness (LOC). c. Obtain computed tomography (CT) scan. d. Administer lorazepam (Ativan) 4 mg IV.

D. To prevent ongoing seizures, the nurse should administer rapidly acting antiseizure medications such as the benzodiazepines. A CT scan is appropriate, but prevention of any seizure activity during the CT scan is necessary. Phenytoin will also be administered, but it is not rapidly acting. Patients who are experiencing tonic-clonic seizures are nonresponsive, although the nurse should assess LOC after the seizure.

A patient with diabetic nephropathy is admitted for a right renal biopsy. Immediately after the biopsy, which of these is an essential nursing action? A. Monitor the blood urea nitrogen (BUN) and creatinine to assess renal function. Incorrect B. Check blood glucose to assess for hyperglycemia or hypoglycemia. C. Insert a straight catheter to check for gross or microscopic hematuria. D. Apply a pressure dressing and keep the patient on the affected side for 30 to 60 minutes.

D. Apply a pressure dressing and keep the patient on the affected side for 30 to 60 minutes. A pressure dressing is applied and the patient is kept on the affected side for 30 to 60 minutes to put pressure on the biopsy side and decrease the risk for bleeding. The blood glucose and BUN/creatinine will not be affected by the biopsy. Although monitoring for hematuria is needed, there is no need for catheterization.

Which assessment finding of the respiratory system does the nurse interpret as abnormal? A. Inspiratory chest expansion of 1 in B. Percussion resonance over the lung bases C. Symmetric chest expansion and contraction D. Bronchial breath sounds in the lower lung fields

D. Bronchial breath sounds in the lower lung fields Bronchial or bronchovesicular sounds heard in the peripheral lung fields are abnormal breath sounds.

After swallowing, a 73-year-old patient is coughing and has a wet voice. What changes of aging could be contributing to this abnormality? A. Decreased response to hypercapnia B. Decreased number of functional alveoli C. Increased calcification of costal cartilage D. Decreased respiratory defense mechanisms

D. Decreased respiratory defense mechanisms These manifestations are associated with aspiration, which more easily occur in the right lung as the right mainstem bronchus is shorter, wider, and straighter than the left mainstem bronchus. Aspiration occurs more easily in the older patient related to decreased respiratory defense mechanisms (e.g., decreases in immunity, ciliary function, cough force, sensation in pharynx). Changes of aging include a decreased response to hypercapnia, decreased number of functional alveoli, and increased calcification of costal cartilage, but these do not increase the risk of aspiration.

What should the nurse inspect when assessing a patient with shortness of breath for evidence of long-standing hypoxemia? A. Chest excursion B. Spinal curvatures C. Respiratory pattern D. Fingernails and their base

D. Fingernails and their base Clubbing, a sign of long-standing hypoxemia, is evidenced by an increase in the angle between the base of the nail and the fingernail to 180 degrees or more, usually accompanied by an increase in the depth, bulk, and sponginess of the end of the finger.

A patient with a recent history of a dry cough has had a chest x-ray that revealed the presence of nodules. In an effort to determine whether the nodules are malignant or benign, what is the primary care provider likely to order? A. Thoracentesis B. Pulmonary angiogram C. CT scan of the patient's chest D. Positron emission tomography (PET)

D. Positron emission tomography (PET) PET is used to distinguish benign and malignant pulmonary nodules. Because malignant lung cells have an increased uptake of glucose, the PET scan (which uses an IV radioactive glucose preparation) can demonstrate increased uptake of glucose in malignant lung cells. This differentiation cannot be made using CT, a pulmonary angiogram, or thoracentesis.

When assessing motor function of a patient admitted with a stroke, the nurse notes mild weakness of the arm demonstrated by downward drifting of the arm. How should the nurse most accurately document this finding? A. Athetosis B. Hypotonia C. Hemiparesis D. Pronator drift

D. Pronator drift Downward drifting of the arm or pronation of the palm is identified as a pronator drift. Athetosis is a slow, writhing, involuntary movement of the extremities. Hypotonia is flaccid muscle tone, and hemiparesis is weakness of one side of the body.

When the patient is experiencing metabolic acidosis secondary to type 1 diabetes mellitus, what physiologic response should the nurse expect to assess in the patient? A. Vomiting B. Increased urination C. Decreased heart rate D. Rapid respiratory rate

D. Rapid respiratory rate When a patient with type 1 diabetes has hyperglycemia and ketonemia causing metabolic acidosis, the physiologic response is to increase the respiratory rate and tidal volume to blow off the excess CO2. Vomiting and increased urination may occur with hyperglycemia, but not as physiologic responses to metabolic acidosis. The heart rate will increase.

Paralysis of lateral gaze indicates a lesion of cranial nerve: A. II. B. III. C. IV. D. VI.

D. VI. Cranial nerves III (oculomotor), IV (trochlear), and VI (abducens) are responsible for eye movement. The lateral rectus eye muscle is innervated by cranial nerve VI, and it is the primary muscle that is responsible for lateral eye movement.

Assessment of muscle strength of older adults cannot be compared with that of younger adults because: A. stroke is more common in older adults. B. nutritional status is better in young adults. C. most young people exercise more than older people. D. aging leads to a decrease in muscle bulk and strength.

D. aging leads to a decrease in muscle bulk and strength. Changes associated with aging include decreases in muscle strength and agility in relation to decreased muscle bulk.

Drugs or diseases that impair the function of the extrapyramidal system may cause loss of: A. sensations of pain and temperature B. regulation of the autonomic nervous system. C. integration of somatic and special sensory inputs. D. automatic movements associated with skeletal muscle activity.

D. automatic movements associated with skeletal muscle activity. A group of descending motor tracts carries impulses from the extrapyramidal system, which includes all motor systems (except the pyramidal system) concerned with voluntary movement. It includes descending pathways originating in the brainstem, basal ganglia, and cerebellum. The motor output exits the spinal cord by way of the ventral roots of the spinal nerves.

The nurse observes unlicensed assistive personnel (UAP) taking the following actions when caring for a female patient with a urethral catheter. Which action requires that the nurse intervene?

Disconnecting the catheter from the drainage tube to obtain a specimen

A 26-year-old patient who is employed as a hairdresser and has a 10 pack-year history of cigarette smoking is scheduled for an annual physical examination. The nurse will plan to teach the patient about the increased risk for A. renal failure. B. kidney stones. C. pyelonephritis. D. bladder cancer.

D. bladder cancer. Exposure to the chemicals involved with working as a hairdresser and in smoking both increase the risk of bladder cancer, and the nurse should assess whether the patient understands this risk. The patient is not at increased risk for renal failure, pyelonephritis, or kidney stones.

When teaching a patient scheduled for a cystogram via a cystoscope about the procedure, the nurse tells the patient, A. "Your doctor will insert a lighted tube into the bladder, and little catheters will be inserted through the tube into your kidney." B. "Your doctor will place a catheter into an artery in your groin and inject a dye that will visualize the blood supply to the kidneys." C. "Your doctor will inject a radioactive solution into a vein in your arm and the distribution of the isotope in your kidneys and bladder will be checked." D. "Your doctor will insert a lighted tube into the bladder through your urethra, inspect the bladder, and instill a dye that will outline your bladder on x-ray."

D. bladder on x-ray." In a cystoscope and cystogram procedure, a cystoscope is inserted into the bladder for direct visualization, and then contrast solution is injected through the scope so that x-rays can be taken. The response beginning, "Your doctor will place a catheter" describes a renal arteriogram procedure. The response beginning, "Your doctor will inject a radioactive solution" describes a nuclear scan. The response beginning, "Your doctor will insert a lighted tube into the bladder, and little catheters will be inserted" describes a retrograde pyelogram.

A patient with a respiratory condition asks "how does air get into my lungs?". The nurse bases her answer on her knowledge that air moves into the lungs because of: A. contraction of the accessory abdominal muscles. B. increased carbon dioxide and decreased oxygen in the blood. C. stimulation of the respiratory muscles by the chemoreceptors. D. decrease in intrathoracic pressure relative to pressure at the airway.

D. decrease in intrathoracic pressure relative to pressure at the airway. During inspiration, the diaphragm contracts, increasing intrathoracic volume and pushing the abdominal contents downward. At the same time, the external intercostal muscles and scalene muscles contract, increasing the lateral and anteroposterior dimension of the chest. This causes the size of the thoracic cavity to increase and intrathoracic pressure to decrease, which enables air to enter the lungs.

The nurse informs the patient undergoing cystoscopy that following the procedure, the patient A. will be NPO for 8 hours to prevent nausea and vomiting. B. is expected to be on strict bed rest for about 4 to 6 hours. C. should ask for the ordered narcotics as necessary for pain. D. may experience blood-tinged urine and urinary frequency.

D. may experience blood-tinged urine and urinary frequency. Pink-tinged urine and urinary frequency are expected after cystoscopy. Burning on urination is common, but pain that requires opioids for relief is not expected. A good fluid intake is encouraged after this procedure. Bed rest is not required following cystoscopy.

In a patient with a disease that affects the myelin sheath of nerves, such as multiple sclerosis, the glial cells affected are the: A. microglia B. astrocytes C. ependymal cells D. oligodendrocytes

D. oligodendrocytes Glial cell types include oligodendrocytes, astrocytes, ependymal cells, and microglia, and each has specific functions. Oligodendrocytes are specialized cells that produce the myelin sheath of nerve fibers in the central nervous system (CNS), and they are located primarily in the white matter of the CNS.

A 22-yr-old man is admitted to the emergency department with a stab wound to the abdomen. The patient's vital signs are blood pressure 82/56 mm Hg, pulse 132 beats/min, respirations 28 breaths/min, and temperature 97.9° F (36.6° C). Which fluid, if ordered by the health care provider, should the nurse question?

D5W

The nurse is caring for a 78-year-old patient who was hospitalized 2 days earlier with community-acquired pneumonia. Which assessment information is most important to communicate to the health care provider?

Decreased oxygen saturation to 90% with 100% O2 by non-rebreather mask.

Which assessment finding is most important to report to the health care provider regarding a patient who has had left-sided extracorporeal shock wave lithotripsy?

Decreased urine output

An early sign of increased ICP that the nurse should assess for is?

Decreasing level of consciousness (LOC)

When caring for the patient with a pulmonary artery (PA) pressure catheter, the nurse observes that the PA waveform indicates that the catheter is in the wedged position. Which action should the nurse take next?

Deflate and reinflate the PA balloon

The family members of a patient who has just been admitted to the intensive care unit (ICU) with multiple traumatic injuries have just arrived in the ICU waiting room. Which action should the nurse take next?

Describe the patients injuries and the care that is being provided.

In order to assist an older diabetic patient to engage in moderate daily exercise, which action is most important for the nurse to take?

Determine what type of activities the patient enjoys.

A patient with acute respiratory distress syndrome (ARDS) and acute kidney injury has the following medications ordered. Which medication should the nurse discuss with the health care provider before giving?

Gentamicin (Garamycin) 60 mg IV

A nurse is prepping insulin for a client with DM. He is to receive evening doses of insulin glargine and regular insulin. Which of the following actions should the nurse take to administer these two medications safely? A. Draw up the insulin glargine into the syringe first, then the regular insulin. B. Draw up the regular insulin first, then the insulin glargine C. Draw up the insulin glargine and the regular insulin into two separate syringes. D. Draw up either insulin into the syringe first because both are clear.

Draw up the insulin glargine and the regular insulin into separate syringes.

The patient is admitted with pain, edema, and warm skin on her lower left leg. What test should the nurse expect to be ordered first?

Duplex ultrasound

Which action is most important for the nurse to take when caring for a patient with a subclavian triple-lumen catheter?

During removal of the catheter, have the patient perform the Valsalva maneuver.

A patient who has ovarian cancer is crying and tells the nurse, My husband rarely visits. He just doesn't care. The husband indicates to the nurse that he never knows what to say to help his wife. Which nursing diagnosis is most appropriate for the nurse to add to the plan of care?

Dysfunctional family processes related to effect of illness on family members

A 26-year-old female is transported by EMS to the ED because she experienced the sudden onset of a severe headache that began after she moved her bowels. She is 28 weeks pregnant. Her husband tells you that she is healthy, takes only prenatal vitamins, and has no allergies. On arrival in the ED, the patient is moaning and does not respond to voice. Emergency medical technicians (EMTs) tell you that she vomited about 5 minutes ago? What ESI level should she be?

ESI-1

A nurse is caring for a patient who is orally intubated and receiving mechanical ventilation. To decrease the risk for ventilator-associated pneumonia, which action will the nurse include in the plan of care?

Elevate head of bed to 30 to 45 degrees.

A patient with a possible pulmonary embolism complains of chest pain and difficulty breathing. The nurse finds a heart rate of 142 beats/minute, blood pressure of 100/60 mmHg, and respirations of 42 breaths/minute. Which action should the nurse take first?

Elevate the head of the bed to a semi-Fowlers position.

Which nursing action will be included in the plan of care for a 55-year-old patient with Graves' disease who has exophthalmos?

Elevate the head of the patient's bed to reduce periorbital fluid.

Patient unconscious with no spontaneous movement and in a deep coma, how would you get a response out of this patient?

Elicit painful stimuli

A 56-year-old patient presents in triage with left-sided chest pain, diaphoresis, and dizziness. This patient should be prioritized into which category?

Emergent

A 63-year-old patient with primary hyperparathyroidism has a serum phosphorus level of 1.7 mg/dL (0.55 mmol/L) and calcium of 14 mg/dL (3.5 mmol/L). Which nursing action should be included in the plan of care?

Encourage 4000 mL of fluids daily.

Patient has SCI at the T6, they are in stable condition. This injury places the patient at risk for serious complication. What is good nursing care for this patient?

Encourage patient to breathe as frequently as possible, and as deeply as possible.

A patient with respiratory failure has a respiratory rate of 6 breaths/minute and an oxygen saturation (SpO2) of 88%. The patient is increasingly lethargic. Which intervention will the nurse anticipate?

Endotracheal intubation and positive pressure ventilation

A 58-year-old male patient who is diagnosed with nephrotic syndrome has ascites and 4+ leg edema. Which nursing diagnosis is a priority for the patient?

Excess fluid volume related to low serum protein levels

A patient who is taking rifampin (Rifadin) for tuberculosis calls the clinic and reports having orange discolored urine and tears. Which is the best response by the nurse?

Explain that orange discolored urine and tears are normal while taking this medication.

A nurse is monitoring the lab values of a client who has DM and is taking insulin. Which of the following results indicates a therapeutic outcome of insulin therapy? A. Fasting blood glucose 96 mg/dL B. Postprandial blood glucose 195 mg/dL C. Casual blood glucose 210 mg/dL D. Preprandial blood glucose 60 mg/dL

Fasting blood glucose is 96mg/dL

A client is suspected of exposure to inhaled anthrax. The nurse assesses for which initial symptoms?

Fever, persistent cough, and dyspnea

When planning care for a patient with dehydration related to nausea and vomiting, the nurse would anticipate which fluid shift to occur because of the fluid volume deficit?

Fluid movement from the interstitial space into the blood vessels

The nurse provides preoperative instruction for a patient scheduled for a left pneumonectomy for cancer of the lung. Which information should the nurse include about the patients postoperative care?

Frequent use of an incentive spirometer

A 78-kg patient with septic shock has a urine output of 30 mL/hr for the past three hours. The pulse rate is 120/minute and the central venous pressure and pulmonary artery wedge pressure are low. Which order by the health care provider will the nurse question?

Give PRN furosemide (Lasix) 40 mg IV

A 78-kg patient with septic shock has a urine output of 30 mL/hr for the past 3 hours. The pulse rate is 120/minute and the central venous pressure and pulmonary artery wedge pressure are low. Which order by the health care provider will the nurse question?

Give PRN furosemide (Lasix) 40 mg IV.

Which action should the nurse perform when preparing a patient with supraventricular tachycardia for cardioversion who is alert and has a blood pressure of 110/66 mm Hg?

Give a sedative before cardio version is implemented.

The following interventions are ordered by the health care provider for a patient who has respiratory distress and syncope after eating strawberries. Which will the nurse complete first?

Give epinephrine (Adrenalin).

A 32-year-old patient with a history of polycystic kidney disease is admitted to the surgical unit after having shoulder surgery. Which of the routine postoperative orders is most important for the nurse to discuss with the health care provider?

Give ketorolac (Toradol) 10 mg PO PRN for pain.

A patient with septic shock has a BP of 70/46 mm Hg, pulse 136, respirations 32, temperature 104° F, and blood glucose 246 mg/dL. Which intervention ordered by the health care provider should the nurse implement first?

Give normal saline IV at 500 mL/hr.

A patient who is on the progressive care unit develops atrial flutter, rate 150, with associated dyspnea and chest pain. Which action that is included in the hospital dysrhythmia protocol should the nurse do first?

Give supplemental O2 at 2 to 3 L/min via nasal cannula.

The nurse has administered 4 oz of orange juice to an alert patient whose blood glucose was 62 mg/dL. Fifteen minutes later, the blood glucose is 67 mg/dL. Which action should the nurse take next?

Give the patient 4 to 6 oz more orange juice.

Which action should the nurse take after a 36-year-old patient treated with intramuscular glucagon for hypoglycemia regains consciousness?

Give the patient a snack of peanut butter and crackers.

A patient who had a transverse colectomy for diverticulosis 18 hours ago has nasogastric suction and is complaining of anxiety and incisional pain. The patients respiratory rate is 32 breaths/minute and the arterial blood gases (ABGs) indicate respiratory alkalosis. Which action should the nurse take first?

Give the patient the PRN IV morphine sulfate 4 mg.

A 37-year-old female patient is hospitalized with acute kidney injury (AKI). Which information will be most useful to the nurse in evaluating improvement in kidney function?

Glomerular filtration rate (GFR)

A few weeks after an 82-year-old with a new diagnosis of type 2 diabetes has been placed on metformin (Glucophage) therapy and taught about appropriate diet and exercise, the home health nurse makes a visit. Which finding by the nurse is most important to discuss with the health care provider?

Glomerular filtration rate is decreased.

Which information will the nurse include when teaching a 50-year-old patient who has type 2 diabetes about glyburide (Micronase, DiaBeta, Glynase)?

Glyburide stimulates insulin production and release from the pancreas.

A patient with type 2 diabetes is scheduled for a follow-up visit in the clinic several months from now. Which test will the nurse schedule to evaluate the effectiveness of treatment for the patient?

Glycosylated hemoglobin level

A patient arrives in the ear, nose, and throat clinic complaining of a piece of tissue being stuck up my nose and with foul-smelling nasal drainage from the right nare. Which action should the nurse take first?

Have the patient occlude the left nare and blow the nose.

A patient develops carpopedal spasms and tingling of the lips following a parathyroidectomy. Which action should the nurse take first?

Have the patient rebreathe from a paper bag.

The nurse assesses a 76 year old man with chronic myeloid leukemia receiving nilotinib (Tasigna). It is most important for the nurse to ask which question?

Have you had a fever?

The nurse is admitting a patient with possible rheumatic fever. Which question on the admission health history will be most pertinent to ask?

Have you had a recent sore throat?

The nurse obtains a health history from a 65-year-old patient with a prosthetic mitral valve who has symptoms of infective endocarditis (IE). Which question by the nurse is most appropriate?

Have you had dental work done recently?

The nurse is performing tuberculosis (TB) skin tests in a clinic that has many patients who have immigrated to the United States. Which question is most important for the nurse to ask before the skin test?

Have you received the bacille Calmette-Gurin (BCG) vaccine for TB?

Patient admitted to the ED with severe chest pain, on what basis will the nurse suspect an MI

He reports he has had no relief of pain with rest or position change

Which signs and symptoms in a patient with a T4 spinal cord injury should alert you to the possibility of autonomic dysreflexia?

Headache and rising blood pressure

A 63-year-old male patient had a cystectomy with an ileal conduit yesterday. Which new assessment data is most important for the nurse to communicate to the physician?

Heart rate 102 beats/minute

A 55-year-old patient with end-stage kidney disease (ESKD) is scheduled to receive a prescribed dose of epoetin alfa (Procrit). Which information should the nurse report to the health care provider before giving the medication?

Hemoglobin level 13 g/dL

A nurse is assessing a client who has a new diagnosis of Cushing's disease. Which of the following findings should the nurse expect? A. Decreased blood pressure B. Weight loss C. Hirsutism D. Increased skin thickness

Hirsutism (increased hair growth is an expected finding of Cushing's disease due to increased androgen production.

A nurse obtains a health history from a patient who has a 35 pack-year smoking history. The patient complains of hoarseness and tightness in the throat and difficulty swallowing. Which question is most important for the nurse to ask?

How much alcohol do you drink in an average week?

A patient with paraplegia has developed an irritable bladder with reflex emptying. What will be most helpful for the nurse to teach the patient?

How to perform intermittent self-catheterization

The nurse is caring for a patient receiving an initial dose of chemo to treat a rapidly growing metastatic colon cancer. The nurse is aware that this patient is at risk for tumor lysis syndrome and will monitor the pt closely for which abnormality associated with this oncologic emergency?

Hypocalcemia

The nurse discusses management of upper respiratory infections (URI) with a patient who has acute sinusitis. Which statement by the patient indicates that additional teaching is needed?

I can use my nasal decongestant spray until the congestion is all gone

The nurse completes discharge instructions for a patient with a total laryngectomy. Which statement by the patient indicates that additional instruction is needed?

I must keep the stoma covered with an occlusive dressing at all times.

A patient with hypotension and an elevated temperature after working outside on a hot day is treated in the ED. The nurse determines that discharge teaching has been effective when the patient makes which statement?

I should drink sports drinks when working outside in hot weather

The nurse establishes the nursing diagnosis of ineffective health maintenance related to lack of knowledge regarding long-term management of rheumatic fever when a 30-year-old recovering from rheumatic fever without carditis says which of the following?

I will be immune to further episodes of rheumatic fever after this infection.

The nurse completes discharge teaching for a patient who has had a lung transplant. The nurse evaluates that the teaching has been effective if the patient makes which statement?

I will call the health care provider right away if I develop a fever.

The home health nurse teaches a patient with a neurogenic bladder how to use intermittent catheterization for bladder emptying. Which patient statement indicates that the teaching has been effective?

I will clean the catheter carefully before and after each catheterization.

The nurse provides discharge instructions to a patient who was hospitalized for pneumonia. Which statement, if made by the patient, indicates a good understanding of the instructions?

I will continue to do the deep breathing and coughing exercises at home.

The nurse determines that instruction regarding prevention of future urinary tract infections (UTIs) has been effective for a 22-year-old female patient with cystitis when the patient states which of the following?

I will empty my bladder every 3 to 4 hours during the day.

The nurse teaches a patient about discharge instructions after a rhinoplasty. Which statement, if made by the patient, indicates that the teaching was successful?

I will keep my head elevated for 48 hours to minimize swelling and pain.

Which statement by a 62-year-old patient with stage 5 chronic kidney disease (CKD) indicates that the nurses teaching about management of CKD has been effective?

I will measure my urinary output each day to help calculate the amount I can drink.

Which statement by the patient indicates that the teaching has been effective for a patient scheduled for radiation therapy of the larynx?

I will need to buy a water bottle to carry with me.

The nurse determines that further instruction is needed for a patient with interstitial cystitis when the patient says which of the following?

I will start taking high potency multiple vitamins every morning.

Which statement by a patient with restrictive cardiomyopathy indicates that the nurses discharge teaching about self-management has been most effective?

I will take antibiotics when my teeth are cleaned at the dental office.

The nurse provides discharge teaching for a patient who has two fractured ribs from an automobile accident. Which statement, if made by the patient, would indicate that teaching has been effective?

I will use the incentive spirometer every hour or two during the day.

After providing a patient with discharge instructions on the management of a new permanent pacemaker, the nurse knows that teaching has been effective when the patient states

I wont lift the arm on the pacemaker side up very high until I see the doctor.

A patient has ST segment changes that support an acute inferior wall myocardial infarction. Which lead would be best for monitoring the patient?

II

A 48-year-old patient with stage 2 chronic kidney disease (CKD) is scheduled for an intravenous pyelogram (IVP). Which order for the patient will the nurse question?

Ibuprofen (Advil) 400 mg PO PRN for pain

The nurse plans to teach a patient how to manage allergic rhinitis. Which information should the nurse include in the teaching plan?

Identification and avoidance of environmental triggers are the best way to avoid symptoms.

A patient with a dissection of the arch of the aorta has a decreased LOC and weak carotid pulses. What should the nurse anticipate that initial treatment of the patient will include?

Immediate surgery to replace the torn area with a graft

A patient with pneumonia has a fever of 101.4 F (38.6 C), a nonproductive cough, and an oxygen saturation of 88%. The patient complains of weakness, fatigue, and needs assistance to get out of bed. Which nursing diagnosis should the nurse assign as the highest priority?

Impaired gas exchange related to respiratory congestion

After the nurse gives IV atropine to a patient with symptomatic type 1, second-degree atrioventricular (AV) block, which finding indicates that the medication has been effective?

Increase in the patients heart rate

Following surgery for an abdominal aortic aneurysm, a patients central venous pressure (CVP) monitor indicates low pressures. Which action is a priority for the nurse to take?

Increase the IV fluid infusion per protocol.

While caring for a patient who has been admitted with a pulmonary embolism, the nurse notes a change in the patients oxygen saturation (SpO2) from 94% to 88%. Which action should the nurse take next?

Increase the oxygen flow rate.

An older patient with cardiogenic shock is cool and clammy and hemodynamic monitoring indicates a high systemic vascular resistance (SVR). Which intervention should the nurse anticipate doing next?

Increase the rate for the sodium nitroprusside (Nipride) infusion.

A nurse is assessing a client who has diabetes insipidus. The nurse should expect which of the following findings? A. Decreased heart rate B. Increased Hematocrit C. High urine specific gravity D. Decreased BUN

Increased hematocrit (related to dehydration)

The nurse assesses the chest of a patient with pneumococcal pneumonia. Which finding would the nurse expect?

Increased tactile fremitus

Which finding for a patient who has hypothyroidism and hypertension indicates that the nurse should contact the health care provider before administering levothyroxine (Synthroid)?

Increased thyroxine (T4) level

Which finding indicates to the nurse that the current therapies are effective for a patient with acute adrenal insufficiency?

Increasing serum sodium levels

If you had a patient that had superior vena cava syndrome secondary to lung cancer, what is a potential nursing diagnosis?

Ineffective breathing pattern

What causes an initial incomplete spinal cord injury to result in complete cord damage?

Infarction and necrosis of the cord caused by edema, hemorrhage, and metabolites

A 54-year-old patient is admitted with diabetic ketoacidosis. Which admission order should the nurse implement first?

Infuse 1 liter of normal saline per hour.

After obtaining the information shown in the accompanying figure regarding a patient with Addison's disease, which prescribed action will the nurse take first?

Infuse 5% dextrose and 0.9% saline.

A nurse is caring for a patient with shock of unknown etiology whose hemodynamic monitoring indicates BP 92/54, pulse 64, and an elevated pulmonary artery wedge pressure. Which collaborative intervention ordered by the health care provider should the nurse question?

Infuse normal saline at 250 mL/hr.

A nurse is planning dietary teaching for a client who has type 1 diabetes mellitus. Which of the following information should the nurse include regarding alcohol consumption? A, Substitute two carbohydrate exchanges for every one alcoholic bev. B. Ingest alcohol with meals to reduce alcohol-induced hypoglycemia. C. Consuming alcohol decreases blood triglyceride levels. D. Expect to increase insulin dosage when consuming alcohol.

Ingest alcohol with meals to reduce alcohol-induced hypoglycemia. (Alcohol prevents liver production of glucose. Consuming carbs while drinking helps prevent hypoglycemia)

Which information will the nurse include when teaching a 50-year-old male patient about somatropin (Genotropin)?

Inject the medication subcutaneously every day.

Which action by a new registered nurse (RN) who is orienting to the progressive care unit indicates a good understanding of the treatment of cardiac dysrhythmias?

Injects IV adenosine (Adenocard) over 2 seconds to a patient with supraventricular tachycardia

A 76-year-old with benign prostatic hyperplasia (BPH) is agitated and confused, with a markedly distended bladder. Which intervention prescribed by the health care provider should the nurse implement first?

Insert a urinary retention catheter.

Which nursing interventions included in the care of a mechanically ventilated patient with acute respiratory failure can the registered nurse (RN) delegate to an experienced licensed practical/vocational nurse (LPN/LVN) working in the intensive care unit?

Insert an indwelling urinary catheter.

A 72-year-old patient with a history of benign prostatic hyperplasia (BPH) is admitted with acute urinary retention and elevated blood urea nitrogen (BUN) and creatinine levels. Which prescribed therapy should the nurse implement first?

Insert urethral catheter.

When assessing a patient who has just arrived after an automobile accident, the emergency department nurse notes tachycardia and absent breath sounds over the right lung. For which intervention will the nurse prepare the patient?

Insertion of a chest tube with a chest drainage system

Which action will the nurse include in the plan of care for a patient who was admitted with syncopal episodes of unknown origin?

Instruct the patient to call for assistance before getting out of bed.

Which assessment finding obtained by the nurse when assessing a patient with acute pericarditis should be reported immediately to the health care provider?

Jugular venous distention (JVD) to jaw level

A nurse is caring for a patient who has had a total laryngectomy and radical neck dissection. During the first 24 hours after surgery what is the priority nursing action?

Keep the patient in semi-Fowlers position.

Which assessment finding may indicate that a patient is experiencing adverse effects to a corticosteroid prescribed after kidney transplantation?

Knee and hip joint pain

A nurse is monitoring a client's status 24hr after a total thyroidectomy. Which of the following findings should the nurse report to the provider? A. Laryngeal Stridor. B. Productive cough C. Pain with hyperextension of the neck. D. Hoarse, weak voice

Laryngeal Stridor (indicates respiratory obstruction, nurse should take immediate action to preserve airway)

22-year-old female patient seen in the clinic for a bladder infection describes the following symptoms. Which information is most important for the nurse to report to the health care provider?

Left-sided flank pain

A 22-year-old female patient seen in the clinic for a bladder infection describes the following symptoms. Which information is most important for the nurse to report to the health care provider?

Left-sided flank pain

A 21-year-old woman is scheduled for percutaneous transluminal balloon valvuloplasty to treat mitral stenosis. Which information should the nurse include when explaining the advantages of valvuloplasty over valve replacement to the patient?

Lifelong anticoagulant therapy will be needed after mechanical valve replacement.

A 32-year-old patient with diabetes is starting on intensive insulin therapy. Which type of insulin will the nurse discuss using for mealtime coverage?

Lispro (Humalog)

You are caring for an older patient who is receiving IV fluids postoperatively. During the 8:00 AM assessment of this patient, you note that the IV solution, which was ordered to infuse at 125 mL/hr, has infused 950 mL since it was hung at 4:00 AM. What is the priority nursing intervention?

Listen to the patient's lung sounds and assess respiratory status.

The nurse notes that a patients endotracheal tube (ET), which was at the 22-cm mark, is now at the 25-cm mark and the patient is anxious and restless. Which action should the nurse take next?

Listen to the patients breath sounds.

You are caring for a patient admitted with heart failure. The morning laboratory results reveal a serum potassium level of 2.9 mEq/L. Which classification of medications should you withhold until consulting with the health care provider?

Loop diuretics

A patient with acute respiratory distress syndrome (ARDS) who is intubated and receiving mechanical ventilation develops a right pneumothorax. Which action will the nurse anticipate taking next?

Lower the positive end-expiratory pressure (PEEP).

The nurse in the dialysis clinic is reviewing the home medications of a patient with chronic kidney disease (CKD). Which medication reported by the patient indicates that patient teaching is required?

Magnesium hydroxide

A patient is admitted to the emergency department with a possible cervical spinal injury following an automobile crash. During admission of the patient, what is the highest priority for the nurse?

Maintaining a patent airway

When the nurse educator is evaluating the skills of a new registered nurse (RN) caring for patients experiencing shock, which action by the new RN indicates a need for more education?

Maintaining the room temperature at 66° to 68° F for a patient with neurogenic shock

What is the priority intervention in the ED for the patient with a stroke?

Maintenance of respiratory function with a patent airway and oxygen administration

A male patient suffered a diffuse axonal injury from a traumatic brain injury (TBI). He has been maintained on IV fluids for 2 days. The nurse seeks enteral feeding for this patient based on what rationale?

Malnutrition promotes continued cerebral edema.

The nurse is planning postoperative care for a patient who is being admitted to the surgical unit form the recovery room after transsphenoidal resection of a pituitary tumor. Which nursing action should be included?

Measure urine volume every hour.

The nurse notes that a patient has incisional pain, a poor cough effort, and scattered rhonchi after a thoracotomy. Which action should the nurse take first?

Medicate the patient with prescribed morphine.

A patient who is lethargic and exhibits deep, rapid respirations has the following arterial blood gas (ABG) results: pH 7.32, PaO2 88 mm Hg, PaCO2 37 mm Hg, and HCO3 16 mEq/L. How should the nurse interpret these results?

Metabolic acidosis

You are admitting a patient with complaints of abdominal pain, nausea, and vomiting. A proximal bowel obstruction is suspected. Which acid-base imbalance do you anticipate in this patient?

Metabolic alkalosis

Taking care of client with severe neutropenic precautions, how much time do you think the client should spend outside of the room?

Minimal time outside of the room

Which intervention will the nurse include in the plan of care for a patient who has cardiogenic shock?

Monitor breath sounds frequently.

Which nursing intervention is most appropriate when caring for a patient with dehydration?

Monitor daily weight and intake and output.

A 64-year-old male patient who has had progressive chronic kidney disease (CKD) for several years has just begun regular hemodialysis. Which information about diet will the nurse include in patient teaching?

More protein is allowed because urea and creatinine are removed by dialysis.

The nurse teaches a patient about the transmission of pulmonary tuberculosis (TB). Which statement, if made by the patient, indicates that teaching was effective?

My husband will be sleeping in the guest bedroom.

Which finding by the nurse when assessing a patient with a large pituitary adenoma is most important to report to the health care provider?

Nausea and projectile vomiting

A patient who is receiving positive pressure ventilation is scheduled for a spontaneous breathing trial (SBT). Which finding by the nurse is most important to discuss with the health care provider before starting the SBT?

New ST segment elevation is noted on the cardiac monitor.

During change-of-shift report, the nurse is told that a patient has been admitted with dehydration and hypotension after having vomiting and diarrhea for 4 days. Which finding ismost important for the nurse to report to the health care provider?

New onset of confusion

Which finding by the nurse when assessing a patient with Hashimoto's thyroiditis and a goiter will require the most immediate action?

New-onset changes in the patient's voice

Which information about a patient who has been receiving thrombolytic therapy for an acute myocardial infarction (AMI) is MOST important for a nurse to communicate to the HCP?

No change in the patient's chest pain.

Which laboratory value reported to the nurse by the unlicensed assistive personnel (UAP) indicates the most urgent need for the nurse's assessment of the patient?

Noon blood glucose of 52 mg/dL

A patient admitted with acute dyspnea is newly diagnosed with dilated cardiomyopathy. Which information will the nurse plan to teach the patient about managing this disorder?

Notify the doctor about any symptoms of heart failure such as shortness of breath.

Which nursing action can the registered nurse (RN) delegate to experienced unlicensed assistive personnel (UAP) working as a telemetry technician on the cardiac care unit?

Observe cardiac rhythms for multiple patients who have telemetry monitoring.

The nurse administers prescribed therapies for a patient with cor pulmonale and right-sided heart failure. Which assessment would best evaluate the effectiveness of the therapies?

Observe for distended neck veins.

During the primary assessment of a victim of a motor vehicle collision, the nurse determines that the pt is breathing and has an unobstructed airway. Which action should the nurse take next?

Observe the patients respiratory effort

A 67-year-old patient who has fallen from a ladder is transported to the emergency department by ambulance. The patient is unconscious on arrival and accompanied by family members. During the primary survey of the patient, the nurse should

Obtain a Glascow Coma Scale score

A patient who is unconscious after a fall from a ladder is transported to the emergency department by emergency medical personnel. During the primary survey of the patient, the nurse should...

Obtain a Glasgow Coma Scale score

A patient who was admitted with diabetic ketoacidosis secondary to a urinary tract infection has been weaned off an insulin drip 30 minutes ago. The patient reports feeling lightheaded and sweaty. Which action should the nurse take first?

Obtain a glucose reading using a finger stick.

A 46-year-old female patient returns to the clinic with recurrent dysuria after being treated with trimethoprim and sulfamethoxazole (Bactrim) for 3 days. Which action will the nurse plan to take?

Obtain a midstream urine specimen for culture and sensitivity testing.

The nurse documents the vital signs for a patient admitted 2 days ago with gram-negative sepsis: temperature 101.2 F, blood pressure 90/56 mm Hg, pulse 92, respirations 34. Which action should the nurse take next?

Obtain oxygen saturation using pulse oximetry.

A patient who is complaining of a racing heart and feeling anxious comes to the emergency department. The nurse places the patient on a heart monitor and obtains the following electrocardiographic (ECG) tracing.

Obtain the patients vital signs including oxygen saturation.

Which intervention will the nurse include in the plan of care for a 52-year-old male patient with syndrome of inappropriate antidiuretic hormone (SIADH)?

Offer patient hard candies to suck on.

A patient admitted with acute respiratory failure has a nursing diagnosis of ineffective airway clearance related to thick, secretions. Which action is a priority for the nurse to include in the plan of care?

Offer the patient fluids at frequent intervals.

A nurse is caring for an obese patient with right lower lobe pneumonia. Which position will be best to improve gas exchange?

On the left side

Which information about a patient who is receiving cisatracurium (Nimbex) to prevent asynchronous breathing with the positive pressure ventilator requires immediate action by the nurse?

Only continuous IV opioids have been ordered.

Which surgical therapy for AAA is most likely to have the postoperative complication of renal injury?

Open aneurysm repair (OAR) above the level of the renal arteries

The clinic nurse teaches a patient with a 42 pack-year history of cigarette smoking about lung disease. Which information will be most important for the nurse to include?

Options for smoking cessation

A patient recovering from heart surgery develops pericarditis and complains of level 6 (0 to 10 scale) chest pain with deep breathing. Which ordered PRN medication will be the most appropriate for the nurse to give?

Oral ibuprofen (Motrin) 600 mg

Which admission order written by the health care provider for a patient admitted with infective endocarditis (IE) and a fever would be a priority for the nurse to implement?

Order blood cultures drawn from two sites.

Which assessment information is most important for the nurse to obtain to evaluate whether treatment of a patient with anaphylactic shock has been effective?

Oxygen saturation

A nurse is caring for a patient with acute respiratory distress syndrome (ARDS) who is receiving mechanical ventilation using synchronized intermittent mandatory ventilation (SIMV). The settings include fraction of inspired oxygen (FIO2) 80%, tidal volume 450, rate 16/minute, and positive end-expiratory pressure (PEEP) 5 cm. Which assessment finding is most important for the nurse to report to the health care provider?

Oxygen saturation 99%

The nurse cares for a patient who has just had a thoracentesis. Which assessment information obtained by the nurse is a priority to communicate to the health care provider?

Oxygen saturation is 88%.

To determine whether there is a delay in impulse conduction through the atria, the nurse will measure the duration of the patients

P wave.

Which statement by the nurse when explaining the purpose of positive end-expiratory pressure (PEEP) to the family members of a patient with ARDS is accurate?

PEEP prevents the lung air sacs from collapsing during exhalation.

An unconscious patient with increased ICP is on ventilatory support. The nurse notifies the health care provider when arterial blood gas (ABG) measurement results reveal what?

PaO2 of 70 mm Hg

A patient experiences a chest wall contusion as a result of being struck in the chest with a baseball bat. The emergency department nurse would be most concerned if which finding is observed during the initial assessment?

Paradoxic chest movement

You are caring for a patient admitted with a diagnosis of chronic obstructive pulmonary disease (COPD) who has the following arterial blood gas results: pH 7.33, PaO2 47 mm Hg, PaCO2 60 mm Hg, HCO3 32 mEq/L, and O2 saturation of 92%. What is the correct interpretation of these results?

Partially compensated respiratory acidosis

Client who had a stroke and now has expressive aphasia, what would the nurse understand?

Patient is having difficulty expressing language and meaning

A patient with idiopathic pulmonary arterial hypertension (IPAH) is receiving nifedipine (Procardia). Which assessment would best indicate to the nurse that the patients condition is improving?

Patient reports decreased exertional dyspnea.

Which information obtained by the nurse in the endocrine clinic about a patient who has been taking prednisone (Deltasone) 40 mg daily for 3 weeks is most important to report to the health care provider?

Patient stopped taking the medication 2 days ago.

The nurse will plan discharge teaching about the need for prophylactic antibiotics when having dental procedures for which patient?

Patient who had a mitral valve replacement with a mechanical valve.

After receiving change-of-shift report, which patient should the nurse assess first?

Patient who has just returned from having hemodialysis and has a heart rate of 124/min

After receiving the following information about four patients during change-of-shift report, which patient should the nurse assess first?

Patient who has just returned to the unit after balloon valvuloplasty

After change-of-shift report, which patient should the progressive care nurse assess first?

Patient who is receiving IV heparin for a venous thromboembolism and has a partial thromboplastin time (PTT) of 98 sec

After change-of-shift report on a ventilator weaning unit, which patient should the nurse assess first?

Patient who was successfully weaned and extubated 4 hours ago and now has no urine output for the last 6 hours

After change-of-shift report, which patient should the nurse assess first?

Patient with a urethral stricture who has not voided for 12 hours

After receiving report on the following patients, which patient should the nurse assess first?

Patient with acute aortic regurgitation whose blood pressure is 86/54 mm Hg

After change-of-shift report in the progressive care unit, who should the nurse care for first?

Patient with suspected urosepsis who has new orders for urine and blood cultures and antibiotics

The cardiac telemetry unit charge nurse receives status reports from other nursing units about four patients who need cardiac monitoring. Which patient should be transferred to the cardiac unit first?

Patient with tetany who has a new order for IV calcium chloride

A patients cardiac monitor shows a pattern of undulations of varying contours and amplitude with no measurable ECG pattern. The patient is unconscious and pulseless. Which action should the nurse take first?

Perform immediate defibrillation.

While the nurse performs range of motion (ROM) on an unconscious patient with increased ICP, the patient experiences severe decerebrate posturing reflexes. What should the nurse do first?

Perform the exercises less frequently because posturing can increase ICP

Which information will the nurse monitor in order to determine the effectiveness of prescribed calcium carbonate (Caltrate) for a patient with chronic kidney disease (CKD)?

Phosphate level

You are caring for a patient receiving calcium carbonate for the treatment of osteopenia. Which serum laboratory result would you identify as an adverse effect related to this therapy?

Phosphorus falling to 2.1 mg/dL

Which action should the nurse take first when a patient develops a nosebleed?

Pinch the lower portion of the nose for 10 minutes.

A patient who has just been admitted with community-acquired pneumococcal pneumonia has a temperature of 101.6 F with a frequent cough and is complaining of severe pleuritic chest pain. Which prescribed medication should the nurse give first?

Piperacillin/tazobactam (Zosyn)

A patient admitted to the hospital with pneumonia has a history of functional urinary incontinence. Which nursing action will be included in the plan of care?

Place a bedside commode close to the patients bed.

The nurse develops a plan of care to prevent aspiration in a high-risk patient. Which nursing action will be most effective?

Place patients with altered consciousness in side-lying positions.

A 27-year-old patient admitted with diabetic ketoacidosis (DKA) has a serum glucose level of 732 mg/dL and serum potassium level of 3.1 mEq/L. Which action prescribed by the health care provider should the nurse take first?

Place the patient on a cardiac monitor

A 42-year-old patient admitted with acute kidney injury due to dehydration has oliguria, anemia, and hyperkalemia. Which prescribed actions should the nurse take first?

Place the patient on a cardiac monitor.

An experienced nurse instructs a new nurse about how to care for a patient with dyspnea caused by a pulmonary fungal infection. Which action by the new nurse indicates a need for further teaching?

Placing the patient on droplet precautions and in a private hospital room

After reviewing the information shown in the accompanying figure for a patient with pneumonia and sepsis, which information is most important to report to the health care provider?

Platelet count and presence of petechiae

Which menu choice by the patient who is receiving hemodialysis indicates that the nurses teaching has been successful?

Poached eggs, whole-wheat toast, and apple juice

A patient with a pleural effusion is scheduled for a thoracentesis. Which action should the nurse take to prepare the patient for the procedure?

Position the patient sitting upright on the edge of the bed and leaning forward.

A patient who was admitted with a myocardial infarction experiences a 45-second episode of ventricular tachycardia, then converts to sinus rhythm with a heart rate of 98 beats/minute. Which of the following actions should the nurse take next?

Prepare to give IV amiodarone (Cordarone) per agency dysrhythmia protocol.

Which prescribed medication should the nurse administer first to a 60-year-old patient admitted to the emergency department in thyroid storm?

Propranolol (Inderal)

A patient who has been involved in a motor vehicle crash arrives in the emergency department (ED) with cool, clammy skin; tachycardia; and hypotension. Which intervention ordered by the health care provider should the nurse implement first?

Provide oxygen at 100% per non-rebreather mask.

A nurse who is caring for patient with a tracheostomy tube in place has just auscultated rhonchi bilaterally. If the patient is unsuccessful in coughing up secretions, what action should the nurse take?

Put on sterile gloves and use a sterile catheter to suction.

Which information will the nurse include when teaching the patient with a urinary tract infection (UTI) about the use of phenazopyridine (Pyridium)?

Pyridium may change the urine color.

When analyzing the rhythm of a patients electrocardiogram (ECG), the nurse will need to investigate further upon finding a(n)

QRS interval of 0.14 second. (normal range: 0.06-0.12)

A 56-year-old female patient is admitted to the hospital with new onset nephrotic syndrome. Which assessment data will the nurse expect?

Recent weight gain

The nurse is caring for a patient who is intubated and receiving positive pressure ventilation to treat acute respiratory distress syndrome (ARDS). Which finding is most important to report to the health care provider?

Red-brown drainage from orogastric tube

A nurse is preparing to administer propranolol IV bolus to a client experiencing a thyroid storm. Which of the following findings indicates the client is having a therapeutic response? A. reduction of the effects of thyroid hormone on the heart B. Blockage of the release of thyroid hormone from the thyroid gland. C. Increase of the heart's sensitivity to thyroid hormone. D. Increase of the uptake of thyroid hormone by the thyroid glad.

Reduction of the effects of thyroid hormone on the heart.

a nurse is caring for an adolescent client who has a long history of diabetes and is being admitted to the ER confused, flushed ans with acetone odor on breath. Diabetic ketoacidosis is suspected. The nurse should anticipate using which of the following types of insulin to treat this client?

Regular (Humulin R)

A 44-year-old female patient with Cushing syndrome is admitted for adrenalectomy. Which intervention by the nurse will be most helpful for a nursing diagnosis of disturbed body image related to changes in appearance?

Remind the patient that most of the physical changes caused by Cushing syndrome will resolve after surgery.

The following interventions are part of the ED protocol for a patient who has been admitted with multiple bee stings to the hand. Which action should the nurse take first?

Remove the patients rings

You are assigned to telephone triage. A patient who was stung by a common honey bee calls for advice, reports pain and localized swelling, but denies any respiratory distress or other systemic signs of anaphylaxis. What is the action that you should direct the caller to perform?

Remove the stinger by scraping

A patient is admitted with metabolic acidosis. Which system is not functioning normally?

Renal system

After the insertion of an arteriovenous graft (AVG) in the right forearm, a 54-year-old patient complains of pain and coldness of the right fingers. Which action should the nurse take?

Report the patients symptoms to the health care provider.

An occupational health nurse works at a manufacturing plant where there is potential exposure to inhaled dust. Which action, if recommended by the nurse, will be most helpful in reducing the incidence of lung disease?

Require the use of protective equipment.

The nurse assesses a patient for signs of meningeal irritation. Which finding indicates nuchal rigidity is present?

Resistance to flexion of the neck

A patient with a spinal cord injury has spinal shock. The nurse plans care for the patient based on what knowledge?

Resolution of spinal shock is manifested by spasticity, hyperreflexia, and reflex emptying of the bladder.

A patient who was involved in a motor vehicle crash has had a tracheostomy placed to allow for continued mechanical ventilation. How should the nurse interpret the following arterial blood gas results: pH 7.48, PaO 2 85 mm Hg, PaCO2 32 mm Hg, and HCO3 25 mEq/L?

Respiratory alkalosis

The patient was in a traffic collision and is experiencing loss of function below C4. Which effect must the nurse be aware of to provide priority care for the patient?

Respiratory diaphragmatic breathing

For each of the following body systems, identify 3 physical manifestations that the nurse would expect to see in a patient approaching death. Respiratory a. b. c. Skin a. b. c. Gastrointestinal a. b. c. Musculoskeletal a. b. c.

Respiratory: a. Cheyne-Stokes respiration b. Death rattle (inability to cough and clear secretions) c. Increased, then slowing respiratory rate (Also: irregular breathing, terminal gasping) Skin: a. Mottling on hands, feet, and legs that progresses to the torso. b. Cold, clammy skin. c. Cyanosis on nose, nail beds, and knees. (Also: waxlike skin when very near death) Gastrointestinal: a. Slowing of the gastrointestinal tract with accumulation of gas and abdominal distention. b. Loss of sphincter control with incontinence. c. Bowel movement before imminent death or at time of death. Musculoskeletal: a. Loss of muscle tone with sagging jaw. b. Difficulty speaking. c. Difficulty swallowing. (Also: loss of ability to move or maintain body position, loss of gag reflex)

When assessing a patient admitted with nausea and vomiting, which finding best supports the nursing diagnosis of deficient fluid volume?

Restlessness

Which intervention will be included in the plan of care for a male patient with acute kidney injury (AKI) who has a temporary vascular access catheter in the left femoral vein?

Restrict physical activity to bed rest.

The nurse is taking a health history from a 29-year-old pregnant patient at the first prenatal visit. The patient reports no personal history of diabetes but has a parent who is diabetic. Which action will the nurse plan to take first?

Schedule the patient for a fasting blood glucose level.

A nurse is assessing a client who has adrenal insufficiency. Which of the following findings should the nurse expect? A. Moon face B. Weight gain C. Serum calcium 12.8 mg/dL D. Serum sodium 150 mEq/L

Serum calcium 12.8 mg/dL (adrenal insufficiency will raise calcium levels)

Which laboratory result for a patient with multifocal premature ventricular contractions (PVCs) is most important for the nurse to communicate to the health care provider?

Serum potassium 2.9 mEq/L

A 25-year-old male patient has been admitted with a severe crushing injury after an industrial accident. Which laboratory result will be most important to report to the health care provider?

Serum potassium level 6.5 mEq/L

A nurse is performing an assessment on a client who has SIADH. Which of the following assessment data should the nurse report? A. Serum sodium 110 mEq/L B. 2+ DTRs C. Serum K+ 3.7 mEq/L D. Urine specific gravity 1.025

Serum sodium 110 mEq/L (SIADH retains fluids, causing dilutional hyponatremia)

A 70 year old man who has end stage lung cancer is admitted to the hospital with confusion and oliguria for 2 days. Which finding would the nurse report immediately to the health care provider?

Serum sodium level of 118mEq/L

An hour after a thoracotomy, a patient complains of incisional pain at a level 7 (based on 0 to 10 scale) and has decreased left-sided breath sounds. The pleural drainage system has 100 mL of bloody drainage and a large air leak. Which action is best for the nurse to take next?

Set up the patient controlled analgesia (PCA) and administer the loading dose of morphine.

The nurse is caring for a patient who has septic shock. Which assessment finding is mostimportant for the nurse to report to the health care provider?

Skin cool and clammy

You receive a physician's order to change a patient's IV from D5½ NS with 40 mEq KCl/L to D5NS with 20 mEq KCl/L. Which serum laboratory values on this same patient best support the rationale for this IV order change?

Sodium, 135 mEq/L; potassium, 4.5 mEq/L

A 50-yr-old woman with hypertension has a serum potassium level that has acutely risen to 6.2 mEq/L. Which type of order, if written by the health care provider, should the nurse question?

Spironolactone (Aldactone) daily

A patient whose heart monitor shows sinus tachycardia, rate 132, is apneic and has no palpable pulses. What is the first action that the nurse should take?

Start cardiopulmonary resuscitation (CPR).

During the primary survey of a patient with severe leg trauma, the nurse observes that the patients left pedal pulse is absent and the leg is swollen. Which action will the nurse take next?

Start normal saline fluid infusion with a large-bore IV line

The nurse notes premature ventricular contractions (PVCs) while suctioning a patients endotracheal tube. Which action by the nurse is a priority?

Stop and ventilate the patient with 100% oxygen.

Which nursing action could the registered nurse (RN) working in a skilled care hospital unit delegate to an experienced licensed practical/vocational nurse (LPN/LVN) caring for a patient with a permanent tracheostomy?

Suction the tracheostomy when needed.

Which assessment finding in a patient who is admitted with infective endocarditis (IE) ismost important to communicate to the health care provider?

Sudden onset right flank pain

Which hemodynamic parameter is most appropriate for the nurse to monitor to determine the effectiveness of medications given to a patient to reduce left ventricular after load?

Systemic vascular resistance (SVR)

The nurse is caring for a patient receiving a continuous norepinephrine (Levophed) IV infusion. Which patient assessment finding indicates that the infusion rate may need to be adjusted?

Systemic vascular resistance (SVR) is elevated.

A patient has sustained Spinal Cord Injury, which level of injury is at risk for decreased respiratory reserve? (select all that apply)

T6 or above

A female patient is scheduled for an oral glucose tolerance test. Which information from the patient's health history is most important for the nurse to communicate to the health care provider?

The patient uses oral contraceptives.

A patient with a C7 spinal cord injury undergoing rehabilitation tells you he must have the flu because he has a bad headache and nausea. What is your initial action?

Take blood pressure

A patient who has a right-sided chest tube following a thoracotomy has continuous bubbling in the suction-control chamber of the collection device. Which action by the nurse is most appropriate?

Take no further action with the collection device.

a nurse is educating a client who is prescribed metformin (Glucophage) for type 2 diabetes. Which if the following information should the nurse include?

Take the medication with your meal

A nurse is developing a teaching plan for a client who had a thyroidectomy and takes a thyroid hormone replacement. Which of the following instructions should the nurse plan to include? A. Take this med on an empty stomach B Take this med with an antacid C. Change position slowly while taking this med D. Limit your fluid intake while taking this med

Take this medication on an empty stomach (not eat or drink anything for 30 to 60 mins after taking it)

A patient is admitted to the emergency department with an open stab wound to the left chest. What is the first action that the nurse should take?

Tape a nonporous dressing on three sides over the chest wound.

Which intervention will the nurse include in the plan of care for a patient who is diagnosed with a lung abscess?

Teach about the need for prolonged antibiotic therapy after discharge from the hospital.

The health care provider writes an order for bacteriologic testing for a patient who has a positive tuberculosis skin test. Which action should the nurse take?

Teach about the need to get sputum specimens for 2 to 3 consecutive days.

When developing a community health program to decrease the incidence of rheumatic fever, which action would be most important for the community health nurse to include?

Teach community members to seek treatment for streptococcal pharyngitis.

The nurse is caring for a patient who has acute pharyngitis caused by Candida albicans. Which action is appropriate for the nurse to include in the plan of care?

Teach patient to swish and swallow prescribed oral nystatin (Mycostatin).

A patient in the urology clinic is diagnosed with monilial urethritis. Which action will the nurse include in the plan of care?

Teach the patient about the use of antifungal medications.

A 55-year-old woman admitted for shoulder surgery asks the nurse for a perineal pad, stating that laughing or coughing causes leakage of urine. Which intervention is most appropriate to include in the care plan?

Teach the patient how to perform Kegel exercises.

A 28 year old patient who has deep human bite wounds on the left hand is being treated in the urgent care center. Which action will the nurse plan to take?

Teach the patient the reason for the use of prophylactic antibiotics

Which action by the nurse will be most effective in decreasing the spread of pertussis in a community setting?

Teaching patients about the need for adult pertussis immunizations

A 74-year-old who is progressing to stage 5 chronic kidney disease asks the nurse, Do you think I should go on dialysis? Which initial response by the nurse is best?

Tell me more about what you are thinking regarding dialysis.

A lobectomy is scheduled for a patient with stage I nonsmall cell lung cancer. The patient tells the nurse, I would rather have chemotherapy than surgery. Which response by the nurse is most appropriate?

Tell me what you know about the various treatments available.

Which assessment finding for a 33-year-old female patient admitted with Graves' disease requires the most rapid intervention by the nurse?

Temperature 103.8° F (40.4° C)

How is cranial nerve (CN) III, originating in the midbrain, assessed by the nurse for an early indication of pressure on the brainstem?

Test papillary reaction to light

A patient with a head injury has bloody drainage from the ear. What should the nurse do to determine if CSF is present in the drainage?

Test the fluid for a halo sign on a white dressing

A licensed practical/vocational nurse (LPN/LVN) is caring for a patient with stage 2 chronic kidney disease. Which observation by the RN requires an intervention?

The LPN/LVN administers the iron supplement and phosphate binder with lunch.

The nurse educator is evaluating the care that a new registered nurse (RN) provides to a patient receiving mechanical ventilation. Which action by the new RN indicates the need for more education?

The RN positions the patient with the head of bed at 10 degrees.

The nurse educator is evaluating the performance of a new registered nurse (RN) who is providing care to a patient who is receiving mechanical ventilation with 15 cm H2O of peak end-expiratory pressure (PEEP). Which action indicates that the new RN is safe?

The RN uses a closed-suction technique to suction the patient.

Which assessment finding of a 42-year-old patient who had a bilateral adrenalectomy requires the most rapid action by the nurse?

The blood pressure (BP) is 88/50 mm Hg.

When rewarming a patient who arrive in the ED with a temperature of 87F, which assessment indicates that the nurse should discontinue active rewarming?

The core temperature is 94F

A patient is admitted to the hospital with Guillain-Barre syndrome. She had weakness in her feet and ankles that has progressed to weakness with numbness and tingling in both legs. During the acute phase of the illness, what should the nurse know about Guillain-Barres syndrome?

The most important aspect of care is to monitor the pt's respiratory and depth and vital capacity

Which intervention by a new nurse who is caring for a patient who has just had an implantable cardioverter-defibrillator (ICD) inserted indicates a need for more education about care of patients with ICDs?

The nurse assists the patient to do active range of motion exercises for all extremities.

The nurse is caring for a hospitalized older patient who has nasal packing in place to treat a nosebleed. Which assessment finding will require the most immediate action by the nurse?

The oxygen saturation is 89%.

A patient who had a total laryngectomy has a nursing diagnosis of hopelessness related to loss of control of personal care. Which information obtained by the nurse is the best indicator that this identified problem is resolving?

The patient asks how to clean the tracheostomy stoma and tube.

Which action by a 70-year-old patient who is using peritoneal dialysis (PD) indicates that the nurse should provide more teaching about PD?

The patient cleans the catheter while taking a bath each day.

Which patient action indicates good understanding of the nurse's teaching about administration of aspart (NovoLog) insulin?

The patient cleans the skin with soap and water before insulin administration.

The nurse is caring for a 78-year-old patient with aortic stenosis. Which assessment data obtained by the nurse would be most important to report to the health care provider?

The patient complains of chest pressure when ambulating.

Which action by a patient indicates that the home health nurse's teaching about glargine and regular insulin has been successful?

The patient disposes of the open vials of glargine and regular insulin after 4 weeks.

Which information is most important for the nurse to communicate rapidly to the health care provider about a patient admitted with possible syndrome of inappropriate antidiuretic hormone (SIADH)?

The patient has a serum sodium level of 118 mEq/L.

The nurse is caring for a 64-year-old patient admitted with mitral valve regurgitation. Which information obtained by the nurse when assessing the patient should be communicated to the health care provider immediately?

The patient has bilateral crackles.

The nurse is interviewing a new patient with diabetes who receives rosiglitazone (Avandia) through a restricted access medication program. What is most important for the nurse to report immediately to the health care provider?

The patient has chest pressure when walking.

A 37-year-old patient has just arrived in the postanesthesia recovery unit (PACU) after a thyroidectomy. Which information is most important to communicate to the surgeon?

The patient has increasing neck swelling.

Which information in a patients history indicates to the nurse that the patient is not an appropriate candidate for kidney transplantation?

The patient has metastatic lung cancer.

Which information from a patient who had a transurethral resection with fulguration for bladder cancer 3 days ago is most important to report to the health care provider?

The patient has seen clots in the urine.

A nurse is caring for a patient with ARDS who is being treated with mechanical ventilation and high levels of positive end-expiratory pressure (PEEP). Which assessment finding by the nurse indicates that the PEEP may need to be reduced?

The patient has subcutaneous emphysema on the upper thorax.

A 28-year-old male patient with type 1 diabetes reports how he manages his exercise and glucose control. Which behavior indicates that the nurse should implement additional teaching?

The patient increases daily exercise when ketones are present in the urine.

A patient is diagnosed with both human immunodeficiency virus (HIV) and active tuberculosis (TB) disease. Which information obtained by the nurse is most important to communicate to the health care provider?

The patient is being treated with antiretrovirals for HIV infection.

Which finding about a patient who is receiving vasopressin (Pitressin) to treat septic shock ismost important for the nurse to communicate to the health care provider?

The patient is complaining of chest pain.

The nurse is caring for a patient admitted with diabetes insipidus (DI). Which information ismost important to report to the health care provider?

The patient is confused and lethargic.

When admitting a patient with possible respiratory failure with a high PaCO2, which assessment information should be immediately reported to the health care provider?

The patient is somnolent.

Which patient action indicates a good understanding of the nurse's teaching about the use of an insulin pump?

The patient programs the pump for an insulin bolus after eating.

A nurse is weaning a 68-kg male patient who has chronic obstructive pulmonary disease (COPD) from mechanical ventilation. Which patient assessment finding indicates that the weaning protocol should be stopped?

The patient respiratory rate is 32 breaths/min.

Which information is most important for the nurse to report to the health care provider before a patient with type 2 diabetes is prepared for a coronary angiogram?

The patient took the prescribed metformin (Glucophage) today.

A 55-year-old female patient with type 2 diabetes has a nursing diagnosis of imbalanced nutrition: more than body requirements. Which goal is most important for this patient?

The patient will reach a glycosylated hemoglobin level of less than 7%.

The patient with neurogenic shock is receiving a phenylephrine (Neo-Synephrine) infusion through a right forearm IV. Which assessment finding obtained by the nurse indicates a need forimmediate action?

The patient's IV infusion site is cool and pale

Which finding indicates a need to contact the health care provider before the nurse administers metformin (Glucophage)?

The patient's blood urea nitrogen (BUN) level is 52 mg/dL.

Norepinephrine (Levophed) has been prescribed for a patient who was admitted with dehydration and hypotension. Which patient data indicate that the nurse should consult with the health care provider before starting the norepinephrine?

The patient's central venous pressure is 3 mm Hg.

Which data collected by the nurse caring for a patient who has cardiogenic shock indicate that the patient may be developing multiple organ dysfunction syndrome (MODS)?

The patient's serum creatinine level is elevated.

The nurse is obtaining a health history from a 24-year-old patient with hypertrophic cardiomyopathy (HC). Which information obtained by the nurse is most important?

The patients 29-year-old brother died from a sudden cardiac arrest.

The nurse is caring for a 33-year-old patient who arrived in the emergency department with acute respiratory distress. Which assessment finding by the nurse requires the most rapid action?

The patients PaO2 is 45 mm Hg.

A patient with acute respiratory distress syndrome (ARDS) is placed in the prone position. When prone positioning is used, which information obtained by the nurse indicates that the positioning is effective?

The patients PaO2 is 89 mm Hg, and the SaO2 is 91%.

The nurse is caring for a patient with idiopathic pulmonary arterial hypertension (IPAH) who is receiving epoprostenol (Flolan). Which assessment information requires the most immediate action by the nurse?

The patients central IV line is disconnected.

The nurse is assessing a patient 4 hours after a kidney transplant. Which information is most important to communicate to the health care provider?

The patients central venous pressure (CVP) is decreased.

A female patient with chronic kidney disease (CKD) is receiving peritoneal dialysis with 2 L inflows. Which information should the nurse report immediately to the health care provider?

The patients peritoneal effluent appears cloudy.

A patient with chronic obstructive pulmonary disease (COPD) arrives in the emergency department complaining of shortness of breath and dyspnea on minimal exertion. Which assessment finding by the nurse is most important to report to the health care provider?

The patients pulse oximetry indicates an O2 saturation of 91%.

Which assessment finding obtained by the nurse when caring for a patient receiving mechanical ventilation indicates the need for suctioning?

The patients respiratory rate is 32 breaths/minute.

A patient with right lower-lobe pneumonia has been treated with IV antibiotics for 3 days. Which assessment data obtained by the nurse indicates that the treatment has been effective?

The patients white blood cell (WBC) count is 9000/L.

Which finding for a patient admitted with glomerulonephritis indicates to the nurse that treatment has been effective?

The periorbital and peripheral edema is resolved.

Which information will the nurse include when teaching a patient who is scheduled for a radiofrequency catheter ablation for treatment of atrial flutter?

The procedure will use electrical energy to destroy areas of the conduction system.

Pt with neurogenic shock is receiving a phenylphrine (Neo-Syncephrine) infusion through a right forearm IV. Which assessment finding obtained by the nurse indicates a need for immediate action?

The pts IV infusion site is cool and pale

Which assessment finding obtained by the nurse when caring for a patient with a right radial arterial line indicates a need for the nurse to take immediate action?

The right hand is cooler than the left hand.

Patient in thrombolytic therapy, but patient still complains of chest pain?

Therapy doesn't work, reperfusion therapy is not effective

A 38-year-old patient who had a kidney transplant 8 years ago is receiving the immunosuppressants tacrolimus (Prograf), cyclosporine (Sandimmune), and prednisone (Deltasone). Which assessment data will be of mostconcern to the nurse?

There is a nontender axillary lump.

When caring for a patient who has an arterial catheter in the left radial artery for arterial pressure based cardiac output (APCO) monitoring, which information obtained by the nurse is most important to report to the health care provider?

There is redness at the catheter insertion site.

You have a client who had a CVA, they now have expressive aphasia. What should the nurse know about this patient?

They have difficulty producing language and expressing need

A patient is admitted with active tuberculosis (TB). The nurse should question a health care providers order to discontinue airborne precautions unless which assessment finding is documented?

Three sputum smears for acid-fast bacilli are negative.

A nurse is caring for a client following a thyroidectomy. The nurse should assess for which of the following findings as an indication of hypocalcemia? A. Strong, bounding pulse B. Decreased bowel sounds. C. Tingling and numbness of the hands and feet. D. Diminished deep tendon reflexes.

Tingling and numbness of the hands and feet. (Hypocalcemia causes paresthesias, usually starting in hands and feet)

Why is the Glasgow Coma scale (GCS) used?

To quickly assess the LOC

The nurse supervises unlicensed assistive personnel (UAP) who are providing care for a patient with right lower lobe pneumonia. The nurse should intervene if which action by UAP is observed?

UAP lower the head of the patients bed to 15 degrees.

The nurse is titrating the IV fluid infusion rate immediately after a patient has had kidney transplantation. Which parameter will be most important for the nurse to consider?

Urine output

A 58-year-old male patient who weighs 242 lb (110 kg) undergoes a nephrectomy for massive kidney trauma due to a motor vehicle crash. Which postoperative assessment finding is most important to communicate to the surgeon?

Urine output is 20 mL/hr for 2 hours.

Which finding is the best indicator that the fluid resuscitation for a patient with hypovolemic shock has been effective?

Urine output is 60 mL over the last hour.

The nurse identifies the nursing diagnosis of decreased cardiac output related to valvular insufficiency for the patient with infective endocarditis (IE) based on which assessment finding(s)?

Urine output less than 30 mL/hr

A 62-year-old female patient has been hospitalized for 8 days with acute kidney injury (AKI) caused by dehydration. Which information will be most important for the nurse to report to the health care provider?

Urine output over an 8-hour period is 2500 mL.

The nurse is caring for a mechanically ventilated patient with a cuffed tracheostomy tube. Which action by the nurse would best determine if the cuff has been properly inflated?

Use a manometer to ensure cuff pressure is at an appropriate level.

When assessing a patient with a sore throat, the nurse notes anterior cervical lymph node swelling, a temperature of 101.6 F (38.7 C), and yellow patches on the tonsils. Which action will the nurse anticipate taking?

Use a swab to obtain a sample for a rapid strep antigen test.

Following rectal surgery, a patient voids about 50 mL of urine every 30 to 60 minutes for the first 4 hours. Which nursing action is most appropriate?

Use an ultrasound scanner to check the postvoiding residual volume.

Employee health test results reveal a tuberculosis (TB) skin test of 16-mm induration and a negative chest x-ray for a staff nurse working on the pulmonary unit. The nurse has no symptoms of TB. Which information should the occupational health nurse plan to teach the staff nurse?

Use and side effects of isoniazid (INH)

During change-of-shift report on a medical unit, the nurse learns that a patient with aspiration pneumonia who was admitted with respiratory distress has become increasingly agitated. Which action should the nurse take first?

Use pulse oximetry to check the oxygen saturation.

Which intervention is most appropriate when communicating with a patient with aphasia after a stroke?

Use simple, short sentences accompanied by visual cues to enhance comprehension

The nurse needs to quickly estimate the heart rate for a patient with a regular heart rhythm. Which method will be best to use?

Use the 3-second markers to count the number of QRS complexes in 6 seconds and multiply by 10.

The dying patient and family have many interrelated psychosocial and physical care needs. Which ones can the nurse begin to manage with the patient and family (select all that apply)? a. Anxiety b. Fear of pain c. The dying process d. Care being provided e. Anger towards the nurse f. Feeling powerless and hopeless

a, b, c, d, e, f. Teaching, along with support and encouragement, can decrease some of the anxiety. Teaching about pain relief, the dying process, and the care provided will help the patient and family know what to expect. Allowing the patient to make decisions will help decrease feelings of powerlessness and hopelessness. The nurse who is the target of anger needs to not react to this anger on a personal level.

15. Priority Decision: The nurse finds a patient in bed having a generalized tonic-clonic seizure. During the seizure activity, what actions should the nurse take (select all that apply)? a. Loosen restrictive clothing. b. Turn the patient to the side. c. Protect the patient's head from injury. d. Place a padded tongue blade between the patient's teeth. e. Restrain the patient's extremities to prevent soft tissue and bone injury.

a, b, c. The focus is on maintaining a patent airway and preventing patient injury. The nurse should not place objects in the patient's mouth or restrain the patient.

According to the World Health Organization, palliative care is an approach that improves quality of life for patients and their families who face problems associated with life-threatening illnesses. From the list below, identify the specific goals of palliative care (select all that apply). a. Regard dying as a normal process. b. Minimize the financial burden on the family. c. Provide relief from symptoms, including pain. d. Affirm life and neither hasten nor postpone death. e. Prolong the patient's life with aggressive new therapies. f. Support holistic patient care and enhance quality of life. g. Offer support to patients to live as actively as possible until death. h. Assist the patient and family to identify and access pastoral care services. i. Offer support to family during the patient's illness and their own bereavement.

a, c, d, f, g, i. Table 10-1 lists the goals of palliative care. Overall, goals of palliative care are to prevent and relieve suffering and to improve the quality of life for the patient.

A patient is admitted to the hospital in hypertensive emergency (BP 244/142 mmHg). Sodium nitroprusside is started to treat the elevated BP. Which management strategy(ies) would be appropriate for this patient (select all that apply)? a. Measuring hourly urine output b. Decreasing the MAP by 50% within the first hour c. Continuous BP monitoring with an intraarterial line d. Maintaining bed rest and providing tranquilizers to lower the BP e. Assessing the patient for signs and symptoms of heart failure and changes in mental status

a, c, e

Which BP-regulating mechanism(s) can result in the development of hypertension if defective (select all that apply)? a. Release of norepinephrine b. Secretion of prostaglandins c. Stimulation of the sympathetic nervous system d. Stimulation of the parasympathetic nervous system e. Activation of the renin-angiotensin-aldosterone system

a, c, e

A 20 year old patient with a massive head injury is on life support, including a ventilator to maintain respiration. What 3 criteria for brain death are necessary to discontinue life support? a.________ b.________ c.________

a. Coma b. Absent brain-stem reflexes. c. Apnea

A patient who has experienced blunt abdominal trauma during a motor vehicle collision is complaining of increased abdominal pain. The nurse will plan to teach the patient about the purpose of...

abdominal ultrasonography

A patient with newly discovered high BP has an average reading of 158/98 mmHg after 3 months of exercise and diet modifications. Which management strategy will be a priority for this patient? a. Medication will be required because the BP is still not at goal b. BP monitoring should continue for another 3 months to confirm a diagnosis of hypertension c. Lifestyle changes are less important, since they were not effective, and medications will be started d. More vigorous changes in the patient's lifestyle are needed for a longer time before starting medications

a. Medication will be required because the BP is still not at goal

In most states, directives to physicians, durable power of attorney for health care, and medical power of attorney are included in what legal documents? a. Natural Death Acts b. Allow natural death c. Supportive care d. Do Not Resuscitate order

a. Natural death acts in each state have their own requirements. Allow natural death is the new term being used for the Do Not Resuscitate order. Advance care planning is the process of having patients and their families think through their values and goals for treatment and document those wishes as advance directives.

List the 2 criteria for admission to a hospice program: a. ____________ b._____________

a. Patient must desire services and agree in writing that only hospice care can be used to treat the terminal illness (palliative care). b. Patient must meet eligibility, which is less than 6 months to live, certified initially by 2 physicians.

a nurse is caring for a client diagnosed with diabetes and observes mild hand tremors. Which of the following actions should the nurse take after obtaining a glucometer reading of 60 mg/dl?

administer 10-15 g of carbohydrates

A terminal ill man tells the nurse, "I have never believed there is a God or an afterlife, but now it is too terrible to imagine that I will not exist. Why was I here in the first place?" What does this comment help the nurse recognize about the patient needs? a. He is experiencing spiritual distress. b. This man most likely will not have a peaceful death. c. He needs to be reassured that his feelings are normal. d. This patient should be referred to a clergyman for a discussion of his beliefs.

a. Spiritual distress may surface when an individual is faced with a terminal illness and it is characterized by verbalization of inner conflicts about beliefs and questioning the meaning of one's own existence. Individuals in spiritual distress may be able to resolve the problem and die peacefully with effective grief work but referral to spiritual leaders should be the patient's choice.

A patient is admitted to the emergency department (ED) for shock of unknown etiology. Thefirst action by the nurse should be to

administer oxygen.

The husband and daughter of a Hispanic woman dying from pancreatic cancer refuse to consider using hospice care. What is the first thing the nurse should do? a. Assess their understanding of what hospice care services are. b. Ask them how they will care for the patient without hospice care. c. Talk directly to the patient and family to see if she can change their minds. d. Accept their decision since they are Hispanic and prefer to care for their own.

a. The family may not understand what hospice care is and may need information. Some cultures and ethnic groups may underuse hospice care because lack of awareness of the services offered, a desire to continue with potentially curative therapies, and concerns about a lack of minority hospice workers.

A 26-year-old patient with diabetes rides a bicycle to and from work every day. Which site should the nurse teach the patient to administer the morning insulin?

abdomen.

An expected outcome during the acute phase of a client with a stroke

airway

A patient with dilated cardiomyopathy has new onset atrial fibrillation that has been unresponsive to drug therapy for several days. The priority teaching needed for this patient would include information about

anticoagulant therapy.

The nurse will anticipate teaching a patient with nephrotic syndrome who develops flank pain about treatment with

anticoagulants.

A 34-year-old has a new diagnosis of type 2 diabetes. The nurse will discuss the need to schedule a dilated eye exam

as soon as possible.

A female patient who had a stroke 24 hours ago has expressive aphasia. The nurse identifies the nursing diagnosis of impaired verbal communication. An appropriate nursing intervention to help the patient communicate is to

ask questions that the patient can answer with "yes" or "no."

A patient in the clinic reports a recent episode of dysphasia and left-sided weakness at home that resolved after 2 hours. The nurse will anticipate teaching the patient about

aspirin

You have a patient and they have a recent episode with aphasia they had left sided weakness but resolved after 2 hours. They had a TIA. What medication should they take?

aspirin

To assess the patient with pericarditis for evidence of a pericardial friction rub, the nurse should

auscultate by placing the diaphragm of the stethoscope on the lower left sternal border.

A 42-year-old female patient is scheduled for transsphenoidal hypophysectomy to treat a pituitary adenoma. During preoperative teaching, the nurse instructs the patient about the need to

avoid brushing teeth for at least 10 days after the surgery.

While caring for a 23-year-old patient with mitral valve prolapse (MVP) without valvular regurgitation, the nurse determines that discharge teaching has been effective when the patient states that it will be necessary to

avoid use of over-the-counter (OTC) medications that contain stimulant drugs.

2. A patient is diagnosed with cluster headaches. The nurse knows that which characteristics are associated with this type of headache (select all that apply)? a. Family history b. Alcohol is the only dietary trigger c. Abrupt onset lasting 5 to 180 minutes d. Severe, sharp, penetrating head pain e. Bilateral pressure or tightness sensation f. May be accompanied by unilateral ptosis or lacrimation

b, c, d, f. Cluster headaches have only alcohol as a dietary trigger and have an abrupt onset lasting 5 minutes to 3 hours with severe, sharp, penetrating pain. Cluster headaches may be accompanied by unilateral ptosis, lacrimation, rhinitis, facial flushing or pallor and commonly recur several times each day for several weeks, with months or years between clustered attacks. Family history and nausea, vomiting, or irritability may be seen with migraine headaches. Bilateral pressure occurring between migraine headaches and intermittent occurrence over long periods of time are characteristics of tension-type headaches.

When providing care for a patient with ALS, the nurse recognizes what as one of the most distressing problems experienced by the patient? a. Painful spasticity of the face and extremities b. Retention of cognitive function with total degeneration of motor function c. Uncontrollable writhing and twisting movements of the face, limbs, and body d. Knowledge that there is a 50% chance the disease has been passed to any offspring

b. In ALS there is gradual degeneration of motor neurons with extreme muscle wasting from lack of stimulation and use. However, cognitive function is not impaired and patients feel trapped in a dying body. Chorea manifested by writhing, involuntary movements is characteristic of HD. As an autosomal dominant genetic disease, HD also has a 50% chance of being passed to each offspring.

A patient with myasthenia gravis is admitted to the hospital with respiratory insufficiency and severe weakness. When is a diagnosis of cholinergic crisis made? a. The patient's respiration is impaired because of muscle weakness. b. Administration of edrophonium (Tensilon) increases muscle weakness. c. Administration of edrophonium (Tensilon) results in improved muscle contractility. d. EMG reveals decreased response to repeated stimulation of muscles.

b. The reduction of the acetylcholine (ACh) effect in myasthenia gravis (MG) is treated with anticholinesterase drugs, which prolong the action of ACh at the neuromuscular synapse, but too much of these drugs will cause a cholinergic crisis with symptoms very similar to those of MG. To determine whether the patient's manifestations are due to a deficiency of ACh or to too much anticholinesterase drug, the anticholinesterase drug edrophonium chloride (Tensilon) is administered. If the patient is in cholinergic crisis, the patient's symptoms will worsen; if the patient is in a myasthenic crisis, the patient will improve.

A patient with end-stage liver failure tells the nurse,"If I can just live to see my first grandchild who is expected in 5 months, then I can die happy." The nurse recognizes that the patient is demonstrating which of the following stages of grieving? a. Prolonged grief disorder. b. Kubler-Ross's stage of bargaining c. Kubler-Ross's stage of depression d. The new normal stage of the Grief Wheel

b. Bargaining

A terminally ill patient is unresponsive and has cold, clammy skin with mottling on the extremities. The patient's husband and two grown children are arguing at the bedside about where the patient's funeral should be held. What should the nurse do first? a. Ask the family members to leave the room if they are going to argue. b. Take the family members aside and explain the patient may be able to hear them. c. Tell the family members that this decision is premature because the patient is not yet dead. d. Remind the family that this should be the patient's decision and to ask her if she regains consciousness.

b. Hearing is often the last sense to disappear with declining consciousness and conversations can distress patients even when they appear unresponsive. Conversation around unresponsive patients should never be other than that which one would maintain if the patients were alert.

Which type of headache is suspected when the headaches are unilateral and throbbing, preceded by a prodrome of photophobia, and associated with a family history of this type of headache? a. Cluster b. Migraine c. Frontal-type d. Tension-type

b. Migraine headaches are frequently unilateral and usually throbbing. They may be preceded by a prodrome and frequently there is a family history. Cluster headaches are also unilateral with severe bone-crushing pain but there is no prodrome or family history. Frontal-type headache is not a functional type of headache. Tension-type headaches are bilateral with constant, squeezing tightness without prodrome or family history.

A patient is receiving care to manage symptoms of a terminal illness when the disease no longer responds to treatment. What is this type of care known as? a. Terminal care b. Palliative care c. Supportive care d. Maintenance care

b. Palliative care is aimed at symptom management rather than curative treatment for diseases that no longer respond to treatment and is focused on caring interventions rather than curative treatments. "Palliative care" and "Hospice" are frequently used interchangeably.

While obtaining subjective assessment date from a patient with hypertension, the nurse recognizes that a modifiable risk factor for the development of hypertension is: a. a low-calcium diet b. excessive alcohol consumption c. a family history of hypertension d. consumption of a high-protein diet

b. excessive alcohol consumption

The nurse is caring for a patient following an adrenalectomy. The highest priority in the immediate postoperative period is to

balance fluids and electrolytes.

The nurse will plan to teach a 27-year-old female who smokes 2 packs of cigarettes daily about the increased risk for

bladder cancer.

A nurse is providing teaching or a client who has DM. Which of the following findings associated with DKA should the nurse include? A. Decreased urine output B. Weight gain of 0.45kg (1lb) in 24 hrs C. Rapid, shallow respirations D. Blood glucose levels greater than 300 mg/dL

blood glucose greater than 300 mg/dL

Sodium polystyrene sulfonate (Kayexalate) is ordered for a patient with hyperkalemia. Before administering the medication, the nurse should assess the

bowel sounds.

A 65-year-old woman was just diagnosed with Parkinson's disease. The priority nursing intervention is a. searching the Internet for educational videos. b. evaluating the home for environmental safety. c. promoting physical exercise and a well-balanced diet. d. designing an exercise program to strengthen and stretch specific muscles.

c Rationale: Promotion of physical exercise and a well-balanced diet are major concerns of nursing care for patients with Parkinson's disease.

In providing care for patients with chronic, progressive neurologic disease, what is the major goal of treatment that the nurse works toward? a. Meet the patient's personal care needs. b. Return the patient to normal neurologic function. c. Maximize neurologic functioning for as long as possible. d. Prevent the development of additional chronic diseases.

c. Many chronic neurologic diseases involve progressive deterioration in physical or mental capabilities and have no cure, with devastating results for patients and families. Health care providers can only attempt to alleviate physical symptoms, prevent complications, and assist patients in maximizing function and self-care abilities for as long as possible.

During care of a patient in myasthenic crisis, maintenance of what is the nurse's first priority for the patient? a. Mobility b. Nutrition c. Respiratory function d. Verbal communication

c. The patient in myasthenic crisis has severe weakness and fatigability of all skeletal muscles, affecting the patient's ability to breathe, swallow, talk, and move. However, the priority of nursing care is monitoring and maintaining adequate ventilation.

During discharge teaching with a 68-year-old patient who had a mitral valve replacement with a mechanical valve, the nurse instructs the patient on the

need for frequent laboratory blood testing.

A patient has a normal cardiac rhythm and a heart rate of 72 beats/minute. The nurse determines that the P-R interval is 0.24 seconds. The most appropriate intervention by the nurse would be to

document the finding and continue to monitor the patient.

A nurse is caring for a client who was exposed to anthrax. Which of the following antibiotics should the nurse plan to administer?

doxycycline

30. To reduce the risk for falls in the patient with Parkinson's disease, what should the nurse teach the patient to do? a. Use an elevated toilet seat. b. Use a walker or cane for support. c. Consciously lift the toes when stepping. d. Rock side to side to initiate leg movements.

c. The shuffling gait of PD causes the patient to be off balance and at risk for falling. Teaching the patient to use a wide stance with the feet apart, to lift the toes when walking, and to look ahead helps to promote a more balanced gait. Use of an elevated toilet seat and rocking from side to side will enable a patient to initiate movement. Canes and walkers are difficult for patients with PD to maneuver and may make the patient more prone to injury.

Which type of seizure occurs in children, is also known as a petit mal seizure, and consists of a staring spell that lasts for a few seconds? a. Atonic b. Simple focal c. Typical absence d. Atypical absence

c. The typical absence seizure is also known as petit mal and the child has staring spells that last for a few seconds. Atonic seizures occur when the patient falls from loss of muscle tone accompanied by brief unconsciousness. Simple focal seizures have focal motor, sensory, or autonomic symptoms related to the area of the brain involved without loss of consciousness. Staring spells in atypical absence seizures last longer than those in typical absence seizures and are accompanied by peculiar behavior during the seizure or confusion after the seizure.

The nurse teaches a 64-year-old woman to prevent the recurrence of renal calculi by

drinking 2000 to 3000 mL of fluid a day.

A patient is admitted to the hospital with possible acute pericarditis. The nurse should plan to teach the patient about the purpose of

echocardiography.

When preparing a female patient with bladder cancer for intravesical chemotherapy, the nurse will teach about

emptying the bladder before the medication.

A patient who had radical neck surgery to remove a malignant tumor developed hypoparathyroidism. The nurse should plan to teach the patient about

calcium supplements to normalize serum calcium levels.

A 38-year-old patient who has type 1 diabetes plans to swim laps daily at 1:00 PM. The clinic nurse will plan to teach the patient to

check glucose level before, during, and after swimming.

The nurse identifies a need for additional teaching when the patient who is self-monitoring blood glucose

chooses a puncture site in the center of the finger pad.

The nurse recognizes that thrombolytic therapy for the treatment of an MI has not been successful when the patient displays which manifestations?

continues to have chest pain

The nurse is reinforcing teaching with a newly diagnosed patient with amyotrophic lateral sclerosis. Which statement would be appropriate to include in the teaching? a. "ALS results from an excess chemical in the brain, and the symptoms can be controlled with medication." b. "Even though the symptoms you are experiencing are severe, most people recover with treatment." c. "You need to consider advance directives now, since you will lose cognitive function as the disease progresses." d. "This is a progressing disease that eventually results in permanent paralysis, though you will not lose any cognitive function."

d Rationale: The disease results in destruction of the motor neurons in the brainstem and spinal cord, causing gradual paralysis. Cognitive function is maintained. Because there is no cure for amyotrophic lateral sclerosis (ALS), collaborative care is palliative and based on symptom relief. Death usually occurs within 3-6 years after diagnosis.

The nurse finds that an 87-year-old woman with Alzheimer's disease is continually rubbing, flexing, and kicking out her legs throughout the day. The night shift reports that this same behavior escalates at night, preventing her from obtaining her required sleep. The next step the nurse should take is to a. ask the physician for a daytime sedative for the patient. b. request soft restraints to prevent her from falling out of her bed. c. ask the physician for a nighttime sleep medication for the patient. d. assess the patient more closely, suspecting a disorder such as restless legs syndrome.

d Rationale: The severity of sensory symptoms of restless legs syndrome (RLS) ranges from infrequent, minor discomfort (paresthesias, including numbness, tingling, and "pins and needles" sensation) to severe pain. The discomfort occurs when the patient is sedentary and is most common in the evening or at night. The pain at night can disrupt sleep and is often relieved by physical activity, such as walking, stretching, rocking, or kicking. In the most severe cases, patients sleep only a few hours at night, which results in daytime fatigue and disruption of the daily routine. The motor abnormalities associated with RLS consist of voluntary restlessness and stereotyped, periodic, involuntary movements. The involuntary movements usually occur during sleep. Symptoms are aggravated by fatigue.

a nurse is caring for a client who is at risk of diabetes mellitus. Which of the following laboratory results indicates the client meets the criteria for diagnosis with DM?

fasting plasma glucose 155 mg/dL

End-of-Life palliative nursing care involves a. constant assessment for changes in physiologic functioning b. administering large doses of analgesics to keep patient sedated c. providing as little physical care as possible to prevent disturbing the patient d. encouraging the patient and family members to verbalize their feelings of sadness, loss, and forgiveness

d. In assisting patients with dying, end-of-life care promotes the grieving process, which involves saying goodbye. Physical care is very important for physical comfort, but assessment should be limited to essential data related to the patient's symptoms. Analgesics should be administered for pain, but patients who are sedated cannot participate in the grieving process.

A 34-year-old male patient seen at the primary care clinic complains of feeling continued fullness after voiding and a split, spraying urine stream. The nurse will ask about a history of

gonococcal urethritis.

3. What is the most important method of diagnosing functional headaches? a. CT scan b. Electromyography (EMG) c. Cerebral blood flow studies d. Thorough history of the headache

d. The primary way to diagnose and differentiate between headaches is with a careful history of the headaches, requiring assessment of specific details related to the headache. Electromyelography (EMG) may reveal contraction of the neck, scalp, or facial muscles in tensiontype headaches but this is not seen in all patients. CT scans and cerebral angiography are used to rule out organic causes of the headaches.

A patient in the last stages of life is experiencing shortness of breath and air hunger. Based on practice guidelines, what is the most appropriate action by the nurse? a. Administer oxygen b. Administer bronchodilators c. Administer antianxiety agents d. Use any methods that make the patient more comfortable

d. There currently are no clinical practice guidelines to relieve the shortness of breath and air hunger that often occur at the end of life. The principle of beneficence would encourage any of the options to be tried, based on knowing that whatever gives the patient the most relief should be used.

22. The nurse explains to a patient newly diagnosed with MS that the diagnosis is made primarily by a. spinal x-ray findings. b. T-cell analysis of the blood. c. analysis of cerebrospinal fluid. d. history and clinical manifestations.

d. There is no specific diagnostic test for MS. A diagnosis is made primarily by history and clinical manifestations. Certain diagnostic tests may be used to help establish a diagnosis of MS. Positive findings on MRI include evidence of at least two inflammatory demyelinating lesions in at least two different locations within the central nervous system (CNS). Cerebrospinal fluid (CSF) may have increased immunoglobulin G and the presence of oligoclonal banding. Evoked potential responses are often delayed in persons with MS.

A deathly ill patient from a culture different than the nurse's is admitted. Which question is appropriate to help the nurse provide culturally competent care? a. "If you die, will you want an autopsy?" b. "Are you interested in learning about palliative or hospice care?" c. "Do you have any preferences for what happens if you are dying?" d. "Tell me about your expectations of care during this hospitalization."

d. Using an open-ended statement to seek information related to the patient's and family's perspective and expectations will best guide the plan of care for this patient. This will open the discussion about palliative or hospice care and preferences for end-of-life care.

The nurse will plan to monitor a patient diagnosed with a pheochromocytoma for

headache.

In teaching a patient with hypertension about controlling the condition, the nurse recognizes that: a. all patients with elevated BP require medication b. obese persons must achieve a normal weight to lower BP c. It is not necessary to limit salt in the diet if taking a diuretic d. lifestyle modifications are indicated for all persons with elevated BP

d. lifestyle modifications are indicated for all persons with elevated BP

A major consideration in the management of the other adult with hypertension is to: a. prevent primary hypertension from converting to secondary hypertension b. recognize that the older adult is less likely to adhere to the drug therapy regimen than a younger adult c. ensure that the patient receives larger initial doses of antihypertensive drugs because of impaired absorption d. use careful technique in assessing the BP of the patient because of the possible presence of ab auscultatory gap

d. use careful technique in assessing the BP of the patient because of the possible presence of ab auscultatory gap

When a brain-injured patient responds to nail bed pressure with internal rotation, adduction, and flexion of the arms, the nurse reports the response as?

decorticate posturing.

Four hours after mechanical ventilation is initiated for a patient with chronic obstructive pulmonary disease (COPD), the patients arterial blood gas (ABG) results include a pH of 7.51, PaO2 of 82 mm Hg, PaCO2 of 26 mm Hg, and HCO3 of 23 mEq/L (23 mmol/L). The nurse will anticipate the need to

decrease the respiratory rate.

The nurse is assessing a 41-year-old African American male patient diagnosed with a pituitary tumor causing panhypopituitarism. Assessment findings consistent with panhypopituitarism include

decreased facial hair.

A 56-year-old patient who is disoriented and reports a headache and muscle cramps is hospitalized with possible syndrome of inappropriate antidiuretic hormone (SIADH). The nurse would expect the initial laboratory results to include a(n)

decreased serum sodium.

38-year-old patient has returned home following rehabilitation for a spinal cord injury. The home care nurse notes that the spouse is performing many of the activities that the patient had been managing unassisted during rehabilitation. The most appropriate action by the nurse at this time is to

develop a plan to increase the patient's independence in consultation with the patient and the spouse

An 82-year-old patient in a long-term care facility has several medications prescribed. After the patient is newly diagnosed with hypothyroidism, the nurse will need to consult with the health care provider before administering

diazepam (Valium).

A patient who had surgery for creation of an ileal conduit 3 days ago will not look at the stoma and requests that only the ostomy nurse specialist does the stoma care. The nurse identifies a nursing diagnosis of

disturbed body image related to change in function.

a nurse is caring for a client with type 1 diabetes mellitus. The nurse finds the client lying in bed, sweating, tachycardic, reporting lightheadedness and palpations. Which of the following complications should the nurse suspect?

hypoglycemia

a nurse is teaching a client who has diabetes about which dietary source should provide the greatest percentage of her calories. Which of the following statements indicates the client understands the teaching?

i should eat more calories from complex carbohydrates than anything else

When caring for a patient with head and neck trauma after a motorcycle accident, the emergency department nurse's first action should be to

immobilize the cervical spine

A nurse is caring for a client who returns to the nursing unit from the recovery room after a sigmoid colon resection for adenocarcinoma. The client had an episode of intraoperative bleeding. Which finding indicates to the nurse that the client may be developing hypovolemic shock?

increase hr from 88 to 110

The nurse is caring for a patient with a subarachnoid hemorrhage who is intubated and placed on a mechanical ventilator with 10 cm H2O of peak end-expiratory pressure (PEEP). When monitoring the patient, the nurse will need to notify the health care provider immediately if the patient develops

increased jugular venous distention.

An 81-year-old patient who has been in the intensive care unit (ICU) for a week is now stable and transfer to the progressive care unit is planned. On rounds, the nurse notices that the patient has new onset confusion. The nurse will plan to

inform the receiving nurse and then transfer the patient.

A patient complains of leg cramps during hemodialysis. The nurse should first

infuse a bolus of normal saline.

A nurse is caring for a client who has chronic kidney failure and the following laboratory results: BUN 196 mg/dL, sodium 152 mEq/L, and potassium 7.3 mEq/L. Which of the following interventions should the nurse implement?

infuse regular insulin in dextrose 10% in water

During the primary assessment of a patient with multiple trauma, the nurse observes that the patient's right pedal pulses are absent and the leg is swollen. The nurse's first action should be to

initiate isotonic fluid infusion through two large-bore IV lines

To maintain proper cuff pressure of an endotracheal tube (ET) when the patient is on mechanical ventilation, the nurse should

inject air into the cuff until a slight leak is heard only at peak inflation.

a nurse is preparing to inject insulin lispro to a client who has type 1 diabetes mellitus. which of the following nursing actions is appropriate?

inject the insulin 15 minutes before a meal

An unresponsive patient with type 2 diabetes is brought to the emergency department and diagnosed with hyperosmolar hyperglycemic syndrome (HHS). The nurse will anticipate the need to

insert a large-bore IV catheter.

A patient develops increasing dyspnea and hypoxemia 2 days after heart surgery. To determine whether the patient has acute respiratory distress syndrome (ARDS) or pulmonary edema caused by heart failure, the nurse will plan to assist with

insertion of a pulmonary artery catheter

A 48-year-old male patient screened for diabetes at a clinic has a fasting plasma glucose level of 120 mg/dL (6.7 mmol/L). The nurse will plan to teach the patient about

lifestyle changes to lower blood glucose.

The nurse is planning care for a patient with severe heart failure who has developed elevated blood urea nitrogen (BUN) and creatinine levels. The primary collaborative treatment goal in the plan will be

maintaining cardiac output.

A 26-year-old female with type 1 diabetes develops a sore throat and runny nose after caring for her sick toddler. The patient calls the clinic for advice about her symptoms and a blood glucose level of 210 mg/dL despite taking her usual glargine (Lantus) and lispro (Humalog) insulin. The nurse advises the patient to

monitor blood glucose every 4 hours and notify the clinic if it continues to rise.

A 56-year-old female patient has an adrenocortical adenoma, causing hyperaldosteronism. The nurse providing care should

monitor the blood pressure every 4 hours.

When planning teaching for a 59-year-old male patient with benign nephrosclerosis the nurse should include instructions regarding

monitoring and recording blood pressure.

While waiting for cardiac transplantation, a patient with severe cardiomyopathy has a ventricular assist device (VAD) implanted. When planning care for this patient, the nurse should anticipate

monitoring the surgical incision for signs of infection.

A patient with Guillain-Barre syndrome asks whether he is going to die as the paralysis spreads toward his chest. In responding to the patient, the nurse knows that

most patient with Guillain-Barre syndrome make a complete recovery

After receiving 2 L of normal saline, the central venous pressure for a patient who has septic shock is 10 mm Hg, but the blood pressure is still 82/40 mm Hg. The nurse will anticipate an order for

norepinephrine (Levophed).

The nurse suspects cardiac tamponade in a patient who has acute pericarditis. To assess for the presence of pulsus paradoxus, the nurse should

note when Korotkoff sounds are auscultated during both inspiration and expiration.

During the primary assessment of a trauma victim, the nurse determines that the patient has a patent airway. The next assessment by the nurse should be to

observe the patient's respiratory effort

A nurse is caring for a client who has hypovolemic shock. Which of the following should the nurse recognize as an expected finding?

oliguria

After a 22-year-old female patient with a pituitary adenoma has had a hypophysectomy, the nurse will teach about the need for

oral corticosteroids to replace endogenous cortisol.

The nurse is caring for a 76-yr-old woman admitted to the medical unit with hypernatremia and dehydration after prolonged fever. The best beverage to offer the patient is

orange juice.

During routine assessment of a patient with Guillain-Barre syndrome, the nurse finds the patient is short of breath. The patient's respiratory distress is caused by

paralysis ascending to the nerves that stimulate the thoracic area

Patient with type A aortic dissection has decreased loc and weak carotid pulse, what should the nurse anticipate?

patient is going to surgery

The nurse has identified a nursing diagnosis of acute pain related to inflammatory process for a patient with acute pericarditis. The priority intervention by the nurse for this problem is to

place the patient in Fowlers position, leaning forward on the overbed table.

The intensive care unit (ICU) nurse educator will determine that teaching about arterial pressure monitoring for a new staff nurse has been effective when the nurse

positions the zero-reference stopcock line level with the phlebostatic axis.

Before administration of captopril (Capoten) to a patient with stage 2 chronic kidney disease (CKD), the nurse will check the patients

potassium.

a nurse is talking with a client who has type 2 diabetes that has responded well to hypoglycemic medication. The client reports morning fasting levels above 180 for the past week. In reviewing the clients medication history, the nurse should identify which of the following medications as a possible contributing factor to the recent change in glycemic control?

prednisone

While caring for a patient with aortic stenosis, the nurse identifies a nursing diagnosis of acute pain related to decreased coronary blood flow. A priority nursing intervention for this patient would be to

promote rest to decrease myocardial oxygen demand.

A nurse is creating a plan of care for a client who is in the late stage of inhalation anthrax. Which of the following is appropriate to include in the plan of care?

provide respiratory support

When monitoring for the effectiveness of treatment for a patient with a large anterior wall myocardial infarction, the most important information for the nurse to obtain is

pulmonary artery wedge pressure (PAWP).

You're patient has hypercalcemia with a level of 13. Doctor wants to put patient on a fluid restriction. What do you do?

question the doctor

When a patient with acute kidney injury (AKI) has an arterial blood pH of 7.30, the nurse will expect an assessment finding of

rapid, deep respirations.

It is most important that the nurse ask a patient admitted with acute glomerulonephritis about

recent sore throat and fever.

A hospitalized diabetic patient received 38 U of NPH insulin at 7:00 AM. At 1:00 PM, the patient has been away from the nursing unit for 2 hours, missing the lunch delivery while awaiting a chest x-ray. To prevent hypoglycemia, the best action by the nurse is to

request that if testing is further delayed, the patient be returned to the unit to eat.

When a patient who takes metformin (Glucophage) to manage type 2 diabetes develops an allergic rash from an unknown cause, the health care provider prescribes prednisone (Deltasone). The nurse will anticipate that the patient may

require administration of insulin while taking prednisone.

T6 is good for diaphragm - look for affected area below the diaphragm

results in paralysis below the chest

To prevent recurrence of uric acid renal calculi, the nurse teaches the patient to avoid eating

sardines and liver.

A patient with diabetes who has bacterial pneumonia is being treated with IV gentamicin (Garamycin) 60 mg IV BID. The nurse will monitor for adverse effects of the medication by evaluating the patients

serum creatinine.

Before administration of calcium carbonate (Caltrate) to a patient with chronic kidney disease (CKD), the nurse should check laboratory results for

serum phosphate.

When caring for a patient with mitral valve stenosis, it is most important that the nurse assess for

shortness of breath on exertion.

When caring for a patient with infective endocarditis of the tricuspid valve, the nurse should monitor the patient for the development of

shortness of breath.

A nurse is caring for a client involved in a suspected bioterrorism event involving exposure to cutaneous anthrax. Which of the following manifestations should the nurse anticipate?

skin lesions with pruritis

An expected nursing diagnosis for a 30-year-old patient admitted to the hospital with symptoms of diabetes insipidus is

sleep pattern disturbance related to frequent waking to void.

Which nursing action could the registered (RN) working in a skilled care hospital unit delegate to an experienced licensed practical/vocational nurse (LPN/LVN) caring for a patient with a permanent tracheostomy?

suction the tracheostomy when needed

A patient with cancer decides to stop all treatment including tube feeding and antibiotics. The patient is of sound body and mind. As the nurse what would you do to advocate for your patient?

support clients wishes and encourage client to share wishes with family members

The central venous oxygen saturation (ScvO2) is decreasing in a patient who has severe pancreatitis. To determine the possible cause of the decreased ScvO2, the nurse assesses the patients

temperature.

a school age child who has type 1 diabetes has been vomiting for the past 24 hrs. Which of the following should be the nurse's first action?

test the urine for ketones

A 62-year-old patient with hyperthyroidism is to be treated with radioactive iodine (RAI). The nurse instructs the patient

that symptoms of hypothyroidism may occur as the RAI therapy takes effect.

A surgical repair is planned for a patient who has a 5.5-cm abdominal aortic aneurysm (AAA). On physical assessment of the patient, the nurse would expect to find?

the presence of a bruit in the periumbilical area

A patient reports dizziness and shortness of breath for several days. During cardiac monitoring in the emergency department (ED), the nurse obtains the following electrocardiographic (ECG) tracing.

third-degree atrioventricular (AV) block.

The emergency department (ED) nurse receives report that a patient involved in a motor vehicle crash is being transported to the facility with an estimated arrival in 1 minute. In preparation for the patient's arrival, the nurse will obtain

two 14-gauge IV catheters.

When assisting with the placement of a pulmonary artery (PA) catheter, the nurse notes that the catheter is correctly placed when the monitor shows a

typical PA wedge pressure (PAWP) tracing.

The nurse determines that demeclocycline (Declomycin) is effective for a patient with syndrome of inappropriate antidiuretic hormone (SIADH) based on finding that the patient's

urinary output is increased.

A nurse is caring for a client immediately following a kidney transplant. The nurse should identify which of the following client findings as a possible indication of a delay in functioning of the transplanted kidney? -Blood pressure 110/58 mm Hg -Incisional tenderness -Pink and bloody urine -Urine output 30 mL/2 hr

urine output 30 mL/2 hr

To verify the correct placement of an oral endotracheal tube (ET) after insertion, the best initial action by the nurse is to

use an end-tidal CO2 monitor to check for placement in the trachea.

When planning the response to the potential use of smallpox as an agent of terrorism, the ED nurse manager will plan to obtain adequate quantities of ...

vaccine

The nurse obtains a rhythm strip on a patient who has had a myocardial infarction and makes the following analysis: no visible P waves, P-R interval not measurable, ventricular rate 162, R-R interval regular, and QRS complex wide and distorted, QRS duration 0.18 second. The nurse interprets the patients cardiac rhythm as

ventricular tachycardia.

You are caring for a patient admitted with an exacerbation of asthma. After several treatments, the ABG results are pH 7.40, PaCO2 40 mm Hg, HCO3 24 mEq/L, PaO2 92 mm Hg, and O2 saturation of 99%. You interpret these results as

within normal limits.


Ensembles d'études connexes

Chapter 22: Nursing Management of the Postpartum Woman at Risk

View Set

مراجعة قوانين مساحات الأشكال الهندسية

View Set

West Coast EMT Chapter 17 -- Neurologic Emergencies

View Set